civil procedure ii - uchicago blsa - the university of ...blsa.uchicago.edu/first...

103
Ryan Preston Dahl; 917 687 7146 Civil Procedure II RPD: Keep in mind that International Shoe comes out in 1945. There are pre- Shoe cases and everything else. Introduction 1) Jurisdiction of Federal Court is articulated by Article III, Section 2 of the Constitution, granting Federal jurisdiction in: a) Constitutional and Federal law b) Cases involving ambassadors, ministers, and public counsels c) Admiralty/maritime cases d) Cases in which U.S. is a party e) Cases b/t two or mores states f) Between a state and citizen of a different state g) Citizens of different states (diversity) h) Between states/its citizens and foreign subjects or nations 2) Capron v. Van Noorden. Capron brought action of Case against Van Noorden in the Circuit Court of NC (note: This type of Federal Court no longer exists). Record did not establish the citizenship of the plaintiff. Plaintiff, lost and appealed on an error of the court. a) Van Noorden had won on the merits in the lower decision i) Note: Since this was basically a tort case it could have been prosecuted in state court b) Supreme Court held that the Circuit court failed in its responsibility to determine its jurisdiction: “the [consent] of the parties could not give jurisdiction.” i) There was no diversity of citizenship c) The Court has a duty to determine jurisdiction, its failure to do so constituted an “error of the court” sufficient to support an appeal i) RPD: See Fed.R.Civ.P. R.12(b)(1); see also R.12(h)(3), if there is a question regarding subject matter jurisdiction the court is obliged to raise the question sua sponte d) The burden is on the party seeking diversity jurisdiction to be able to point to something in the record affirming such jurisdiction on the record i) Note: Personal jurisdiction, unlike subject matter jurisdiction, can be waived e) Since defendant in error (Van Noorden) did not show for the appeal, P won judgment 3) Subject-matter jurisdiction of Federal Courts: Applies to many – but not all – matters of Federal law SUBJECT MATTER JURISDICTION Diversity Jurisdiction (a type of subject matter jurisdiction) 1) Diversity of citizenship a) Requirements: 1

Upload: lamnhan

Post on 16-Apr-2018

220 views

Category:

Documents


2 download

TRANSCRIPT

Ryan Preston Dahl; 917 687 7146

Civil Procedure II

RPD: Keep in mind that International Shoe comes out in 1945. There are pre-Shoe cases and everything else.

Introduction1) Jurisdiction of Federal Court is articulated by Article III, Section 2 of the Constitution, granting

Federal jurisdiction in:a) Constitutional and Federal lawb) Cases involving ambassadors, ministers, and public counselsc) Admiralty/maritime casesd) Cases in which U.S. is a partye) Cases b/t two or mores statesf) Between a state and citizen of a different stateg) Citizens of different states (diversity)h) Between states/its citizens and foreign subjects or nations

2) Capron v. Van Noorden. Capron brought action of Case against Van Noorden in the Circuit Court of NC (note: This type of Federal Court no longer exists). Record did not establish the citizenship of the plaintiff. Plaintiff, lost and appealed on an error of the court. a) Van Noorden had won on the merits in the lower decision

i) Note: Since this was basically a tort case it could have been prosecuted in state courtb) Supreme Court held that the Circuit court failed in its responsibility to determine its jurisdiction:

“the [consent] of the parties could not give jurisdiction.” i) There was no diversity of citizenship

c) The Court has a duty to determine jurisdiction, its failure to do so constituted an “error of the court” sufficient to support an appeali) RPD: See Fed.R.Civ.P. R.12(b)(1); see also R.12(h)(3), if there is a question regarding subject

matter jurisdiction the court is obliged to raise the question sua sponted) The burden is on the party seeking diversity jurisdiction to be able to point to something in the

record affirming such jurisdiction on the recordi) Note: Personal jurisdiction, unlike subject matter jurisdiction, can be waived

e) Since defendant in error (Van Noorden) did not show for the appeal, P won judgment3) Subject-matter jurisdiction of Federal Courts: Applies to many – but not all – matters of Federal law

SUBJECT MATTER JURISDICTIONDiversity Jurisdiction (a type of subject matter jurisdiction)1) Diversity of citizenship

a) Requirements:i) Diversity of citizenship and ii) Amount in controversy > $75,000iii) Diversity of citizenship codified in 28 U.S.C. § 1332, though rooted in the Constitution (see

above)(1) § 1332 holds that in Federal cases brought solely on the basis of diversity jurisdiction the

amount in controversy must be over the statutory amount(a) Note: If the ruling for P turns out to be less than $75,000, plaintiff may be taxed for

costs(2) But see 28 U.S.C. § 1359: A court does not have jurisdiction over an action where a

party is improperly joined in order to get jurisdiction(a) See Kramer v. Caribbean Mills, Inc., 394 U.S. 823 (1969).

(i) Kramer had been assigned his interest in the case by a Panamanian company, for an agreement that any recover would go back to the company

(ii) Court held that this assignment was improperly made as defined by § 1359b) There is no diversity jurisdiction if any plaintiff is a citizen of the same state as any defendant, no

matter how many parties are involved in the litigation. Strawbridge v. Curtiss, 7 U.S. (3 Cranch) 267 L.Ed. 435 (1806)

1

Ryan Preston Dahl; 917 687 7146

c) Similarly, if P decides to sue in federal court based on diversity of citizenship, she must bring suit in which D resides or in a district where a substantial part of the events giving rise to the claim occurred

d) Note: Jurisdiction protections are for D which the D may waive4) Exceptions to diversity jurisdiction:

a) Probate i) Federal court may not probate a will nor undertake administration of estateii) But Federal court can here actions against administrators, executors, etc. so long as the federal

court does not interfere with the state proceedings. (1) Federal decision will be binding(2) But federal court cannot order actual distribution of property

iii) This area is highly technicalb) Domestic relations

i) However, the exception is limited to cases involving issuance of a divorce, alimony, or child custody. Ankenbrandt v. Richards, 504 U.S. 689 (1992)

ii) Ankenbrandt used this to uphold district court’s jurisdiction over a tort action related to familial abuse

5) Determining citizenshipa) Party asserting diversity jurisdiction carries the burden of proof to prove its existence

i) Even as a state-less person, the P must still demonstrate diversity of citizenship. Blair Holdings Corp. v. Rubinstein, 144 F. Supp. 496 (S.D.N.Y. 1955)

b) To be a citizen of a State as contemplated by 28 U.S.C. § 1332 one must be both a citizen of the United States and a domiciliary of that State. Mas v. Perry, 489 F.2d 1396 (5th Cir. 1974)i) Case arose from Ps, H and W, who sued D for spying on them. H was a French national, W

claimed to be a resident of MS though she was married and the acts in question occurred when H and W lived in Ds apt building in LA(1) RPD: The problem would have been if W was deemed to be a resident of LA for

purposes of diversity citizenship → there would be no diversityii) Citizenship determined from the time of the institution of the suitiii) Domicile (which is not the same as a residence)

(1) Domicile defined as the true, fixed, and permanent home and principle establishment to which one returns when absent

(2) Change in domicile effected by:(a) Taking up residence in a difference domicile(b) With intent to remain(c) Note: A woman does not have her citizenship changed for diversity jurisdiction

purposes by marrying an alien. (3) Mas’ testimony that she intended to return to her birthplace of MS was sufficient since

she was a student when living in other states(4) Filip: Default rule is that you keep your old citizenship until you get a new one.

c) 28 U.S.C. § 1332(a) (1988): an alien admitted to the U.S. is deemed a citizen of his state in which he is domiciled for the purposes of diversity of citizenship

d) RPD: Could have had a different outcome in Mas if this law were in effect at the timee) Problem of alien plaintiff and alien defendant:

i) 28 U.S.C. § 1332(a) does not permit an alien corporation to sue a partnership made up of both U.S. citizens and permanent resident aliens. China Nuclear Energy Indus. Corp. v. Andersen, LLP, 11 F.Supp.2d 1256 (D.Colo.1998)(1) P had sued D for alleged misrepresentations made about Ps clients, causing P to lose

money(2) Court noted that a partnership is a citizen of every state of each of its partners, so

Andersen was a citizen of any state or country in which it did business(3) Court used this fact to hold that despite the fact that § 1332(a) might allow for such a

claim, this would clearly stand against Article III’s grant of Constitutional authority to the Federal Courts

ii) See also Ruhrgas AG v. Marathon Oil Co., 526 U.S. 574, 580 n.2 (1999): aliens on both sides of the action “rendered diversity incomplete”

2

Ryan Preston Dahl; 917 687 7146

6) Corporations and Diversity citizenshipa) 28 U.S.C. § 1332(c), a corporation is a citizen of:

i) The state in which it is incorporated; andii) The state in which it has its principal place of business

b) Principal place of business determined by:i) Nerve center test: “Locus of corporate decision making” determines the principal place of

businessii) Corporate activities test (the “muscle center” test)iii) Total activity (hybrid of the first two)

c) An unincorporated association is not treated as a citizen for purposes of diversity jurisdiction, but courts instead consider the citizenship of each of its members. United Steelworkers of America v. R.H. Bouligny, Inc. 382 U.S. 145 (1965

d) A limited partnership is not a citizen of the state under whose laws it was created for purposes of diversity jurisdiction; citizenship of an LP is determined by the citizenship of each of its partners. Carden v. Arkoma Assoc., 494 U.S. 185 (1990)

7) 28 U.S.C. § 1332(c): legal representative of an estate only has citizenship of the state of the decedent/infant etc. that is represented. Hence the appointment of administrators cannot create or destroy jurisdiction.

8) Nominal parties intended to create or destroy jurisdiction will be disregarded by the court. Rose v. Giamatti, 721 F. Supp. 906 (S.D. Ohio 1989)a) See also Abels v. State Farm Fire & Cas. Co., 770 F.2d 26 (3d Cir. 1985): inclusion of

unidentified parties (“John Does”) are insufficient to destroy diversity of citizenshipi) P, a CA resident, brought suit against D to compel payment on a fire insurance claim.ii) P had included 10 John Does, allegedly employees of D responsible for processing the policy

and, they claimed, were also residents of California9) Policy problems created by diversity jurisdiction

a) Congestion of diversity cases in Federal Courtb) The Erie problem: federal courts are not/should not be ruling on state lawc) Judicial and legislative authority should be co-extensive, so it makes no sense to have a Federal

Court rule on state lawd) Could retard development of state lawe) Diminishes incentives for reforms of state law

Diversity Jurisdiction and Amount in Controversy1) Codified in 28 U.S.C. § 1332(a); original jurisdiction granted over diversity suits when amount in

controversy is > $75,000a) Note: Prior to 1980, there was an amount in controversy requirement for Federal question cases as

well. This was removed in 1980 except for rare exceptions (e.g., Consumer Product Safety Act)b) Amount in Controversy requirement was the result of a compromise b/t people who wanted to

remove diversity jurisdiction and those who wanted to keep itc) Costs may be imposed on P (or at least denied to P) if actual recovery is less than $75,000. 28

U.S.C. § 1332(b).2) The sum claimed by the P determines the amount in controversy question assuming it is made in good

faith. If allowed by law, punitive damages are included in the calculation. A good faith claim by P regarding the amount in controversy cannot be dismissed on jurisdictional grounds without giving P the opportunity to make a showing. A.F.A. Tours, Inc. v. Whitchurch, 937 F.2d 82 (2d Cir. 1991)a) Claim related to a non-compete between P, a tour operator, and D, a former employee, who was

attempting to start his own tour company with P’s client listb) P claimed actual damages of $50,000 and punitive damages of $75,000. District court dismissed

on jurisdictional grounds. c) Court held that punitive damages are to be included as a matter of law, and “[b]efore making a

determination that the plaintiff’s claim does not meet the jurisdictional minimum, the court must afford the plaintiff an ‘appropriate and reasonable opportunity to show good faith in believing that a recovery in excess of [the jurisdictional amount] is reasonably possible.”

d) See Arnold v. Troccoli, 344 F.2d 842 (2d Cir. 1965)

3

Ryan Preston Dahl; 917 687 7146

i) P had originally brought a personal injury claim in state court for $6,000, at a time when the amount in controversy requirement was $10,000. He later tried to bring the same claim in Federal court claiming damages of $15,000

ii) Court dismissed on jurisdictional grounds. Court relied on statistics which said that 97% of all such accidents recovered amounts less than $10,000

3) “It must appear to a legal certainty that the claim is really for less than the jurisdictional amount to justify dismissal” St. Paul Mercury Indem. Co. v. Red Cab Co., 303 U.S. 283 (1938)

4) Aggregationa) In general, single Ps can aggregate claims against singles Ds. b) Two Ps may not aggregate if they have separate and distinct claims. c) If there is a single and indivisible harm, Ps may aggregate.

5) Determining amount in controversy when injunctive relief is soughta) If the value of the right in question is worth the statutory amount to the plaintiff, than there is

jurisdiction. “That the jurisdictional amount is to be tested by the value of the object to be gained by complainant.” Glenwood Light & Water Co. v. Mutual Light, Heat & Power, 239 U.S. 121 (1915). i) “It was said: 'The want of a sufficient amount of damage having been sustained to give the

Federal courts jurisdiction will not defeat the remedy, as the removal of the obstruction is the matter of controversy, and the value of the object must govern.' The same rule has been applied in numerous cases, and under varying circumstances.” Id.

b) Other courts have attempted to measure the amount in controversy from the point of view of the party attempting to assert Federal jurisdiction. While this makes it easier from a burden of proof point of view, there are problemsi) Anomalous results: failure to meet the amount in controversy from Ps p.o.v. might still get

you into Federal courtc) The “either viewpoint rule”; if the amount in controversy exceeds the minimum for either party

than there is Federal jurisdiction. Ronzio v. Denver R.G.W.R. Co., 116 F.2d 604 (10th Cir. 1940)Federal Questions1) Glannon: Think of Federal jurisdiction cases as “arising under” cases. i.e., cases “arising under”

federal law2) Federal courts did not have general jurisdiction over Federal questions until 18753) Federally created entity, with statutory right to sue and be sued in Federal court, confers subject matter

jurisdiction to Federal court over that entity to all its subsequent disputes. Osborn v. Bank of the United States, 22 U.S. (9 Wheat.) 738 (1824) (Marshall, J.)a) Congressional act chartering the Bank authorized it so “sue and be sued” in Federal court. Ohio

attempted to tax the bank. Bank resisted. Ohio challenged the Federal court’s jurisdiction over the matter

b) Even though (as a Federally-created entity) the case was clearly within the “arising under” language of Art. III, § 2, Marshall based his opinion on the validity of the statutory grant of jurisdiction to all cases in which the bank was a partyi) RPD: Seems to set the stage for the rise of the modern Federal Agency

c) Osborn is viewed as the “any ingredient test” – i.e., if “any ingredient” of the claim involves a federal issue, than the Constitutional issue is satisfiedi) Note: This is not the same as satisfying the statutory requirement

d) Osborn has been said to allow Congress to confer jurisdiction “whenever there exists . . . some federal proposition that might be challenged, despite the remoteness of the likelihood of actual presentation of such a federal question.” Textile Workers Union v. Lincoln Mills, 353 U.S. 448 (1957)

4) A Federal question does not arise from one party claiming that a potential defense or claim made by the other might involve a question of Federal law. Louisville & Nashville Co. v. Motley, 211 U.S. 149 (1908)a) Case arose out of a contract dispute between RR and Motley. Motley had settled his personal

injury suit against the RR in exchange for a lifetime rail pass. Such a pass was subsequently prohibited by Congress.

4

Ryan Preston Dahl; 917 687 7146

b) “It is settled interpretation of [statutory jurisdiction] conferring jurisdiction, that a suit arising under the Constitution and laws of the United States only when the plaintiff’s statement of his own cause of action shows that it is based upon those laws or that Constitution.”i) A federal question is not attained by anticipating a defenseii) “A suggestion of one party, that the other will or may set up a claim under the Constitution or

the laws of the United States, does not make the suit one arising under the Constitution or those laws.”

c) This is the “well-pleaded complaint rule.” In terms of Federal question jurisdiction, the P is confined to a statement of the cause of action. If the cause of action does not bring up a Federal question, than there is no jurisdiction on that ground. i) Note: This rule would not have affected Osborn.ii) “The plaintiff's claim itself must present a federal question 'unaided by anything alleged in

anticipation of avoidance of defenses which it is thought the defendant may interpose.” Skelly Oil Co. v. Phillips Petr. Co., 339 U.S. 667 (1950).

5) A suit for declaratory judgment under the Federal Declaratory Judgment Act (28 U.S.C. § 2201) does not confer Federal jurisdiction absent diversity of citizenship or some other underlying Federal question. Skelly Oil, supra. a) D had brought suit for declaratory judgment that its contracts with P were still valid. D won on

the merits and appealed.b) Court held that since this was a state claim, there was no jurisdiction. The availability of

declaratory judgment was “procedural only” and applied only to those claims which would otherwise have gotten into Federal court. i) “Congress enlarged the range of remedies available in the federal courts but did not extend

their jurisdiction. When concerned as we are with the power of the inferior federal courts to entertain litigation within the restricted area to which the Constitution and Acts of Congress confine them, 'jurisdiction' means the kinds of issues which give right of entrance to federal courts. Jurisdiction in this sense was not altered by the Declaratory Judgment Act.”

ii) “The limited subject matters which alone Congress had authorized the District Courts to adjudicate--were not impliedly repealed or modified.”

c) See also Franchise Tax Board v. Construction Laborers Vacation Trust, 463 U.S. 1, (1983)i) Trust was sued under problems related to CA state tax law. Trust attempted to remove to

Federal court on the grounds that since it was regulated by ERISA, it was not bound by state regulations.

ii) Court invoked the well-pleaded complaint rule to deny district court’s jurisdiction, since the federal question did not arise in the original complaint. (1) Also, court refused to take jurisdiction over state declaratory judgment claims if the issue

were not raised in the original complaint, citing Skelly Oil.(2) Also, refused to allow removal on the basis that Trust’s right to seek injunctive relief in

Federal court also allowed it to bring declaratory actions in Federal court6) For a complaint to state a Federal question, the complaint must require the application or interpretation

of Federal law. T.B. Harms Co. v. Eliscu, 339 F.2d 823 (2d Cir. 1964) (Friendly, J.)a) Case arose over the dispute regarding ownership of a Copyright. b) Court held that there was no Federal question simply by virtue of the subject of the dispute being a

copyright. i) “Even though the claim is created by state law, a case may ‘arise under’ a law of the United

States if the complaint discloses a neeed for determining the meaning or application of such a law.”

ii) Since the issue in Eliscu was regarding the validity of certain assignments, no such analysis was required

c) See also, American x Co. v. Layne & Bowler Co., 217 U.S. 257 (1916) (Holmes, J.)i) “A suit arises under the law that creates the cause of action.”ii) Case related to the slander of a patent. Holmes held that the fact that the slander related to a

patent was irrelevant to the Federal question analysis, since the action was slander. JoinderRule 131) Counterclaims. Important is the distinction between compulsory and permissive counterclaims.

5

Ryan Preston Dahl; 917 687 7146

a) Why the distinction between compulsory and permissive counterclaims? i) If D fails to raise a compulsory counterclaim he is barred from raising it again. The court will

not force people to litigate claimsii) Exception: An omitted counterclaim can be brought in if there is some negligence excusing it

(Rule 13(f))2) Compulsory counterclaims arises out of the same subject matter of the opposing party’s claim and it

doesn’t require the presence of third parties to be fairly adjudicated over whom the trial court can’t get jurisdictiona) How do you test this?b) Note: Necessary counterclaims are estopped/barred if they are not included in the

Answer/response to the intial actionc) Same issue of law or fact?d) Would res judicata bar a counterclaim?e) Would same evidence support either/both claim? (Bose Corp. v. Consumers Union of the United

States, cited in Wigglesworth)f) Any logical relation?

i) Mohr v. New York Cotton Exchange: “transaction is a word of flexible meaning”. This creates a broader test than that found in the actual Rule (defendant in Wigglesworth relies on this broad construction in its motion)

3) Denials of counterclaims are interlocutory and hence not appealable until final appeal. a) Exception: When a counterclaim falls under 28 U.S.C. § 1292(a) (e.g., a counterclaim for

injunctive relief), is certified under 54(b) or §1292(b), etc. 4) Permissive counterclaims do not arise out of the same transaction. Permissive counterclaims are ones

which a) Are already the subject of another action before a courtb) Court has no independent jurisdictional ability to hear the claim as such. Some other jurisdictional

reason must be found in order to c) E.g., When jurisdiction over the original claim stems from the presence of a federal question, the

permissive counterclaim, which by definition does not arise out of the same transaction or occurrence as the original claim, will need its own basis of jurisdiction, which can be furnished either if it involves some other federal question or if there is diversity of citizenship.

d) Courts will grant exceptions (set offs)(w/out independent jurisdictional status) if counterclaim would not require a separate trial, the claims could be liquidated at the same trial (Marks v. Spitz), and/or the dispute arises out of a contract (RPD: This is right from Wigglesworth)i) “When an insurer sues for a declaratory judgment of nonliability, counterclaims interposed by

the accident victims are permissive since they are based on the insured's tort liability and not the insurer's contractual liability.” Wright and Miller, § 1421. E.g., victim’s claims are permissive: In an action by an insurer for a declaration of coverage provided by a farfom and ranch liability policy, a counterclaim by the insured's injured son alleging that the insured was negligent in failing to warn the son of the alleged dangerous condition presented questions entirely separate from the main claim and was not a compulsory counterclaim. St. Paul Fire & Marine Ins. Co. v. Mannie 91 F.R.D. 219 (D.C.N.D. 1981). The court refused to hear the counterclaim.

5) What would an example be of a counterclaim arising out of the same transaction? Great Lakes Tire Corp. v. Herbert Cooper Co. Plaintiff’s initial complaint was about the theft of trade secrets. This was dismissed for lack of subject matter jurisdiction. After defendant asserted an anti-trust counterclaim in its Answer, plaintiff then repackaged its original claim in its counterclaim to the Ds counterclaim. Court ruled this was propera) Defendant’s counterclaim asserted that P’s claim violated anti-trust lawb) Therefore, a necessary relationship existed between plaintiff’s original complaint and the

defendant’s counterclaim (the former was the sine qua non of the latter)c) The facts regarding P’s counterclaim (voiced in the original complaint), then, ought to be

examined by the court as a compulsory counterclaimRule 14. Third Party Practice

6

Ryan Preston Dahl; 917 687 7146

1) D may bring in a Third party when the third party is or may be liable to the third party plaintiff (i.e., the defendant in the original action) for all or part of the plaintiff’s claim against the third party plaintiff. Rule 14(a)a) P may bring in a third party as a result of a counterclaim by the D. Rule 14(b)

2) Plaintiff cannot defeat complete diversity requirement for subject matter jurisdiction by bringing action against an out-of-state party who then files a third party complaint against an in-state party pursuant to Rule 14(a) and then suing the in-state third party defendant. Owen Equipment Co. v. Kroger, 437 U.S. 365 (1978)a) i.e., supplemental jurisdiction does not override the requirements of § 1332(a); see also 28 U.S.C.

§ 1367(b), infra.b) Court will not allow “a plaintiff [to] defeat the statutory requirement of complete diversity by the

simple expedient of suing only those defendants who were of diverse citizenship and waiting for them to implead nondiverse defendants.” Id. at 374.

Rule 18. Joinder of Claims and Remedies1) Can assert as many claims (note that Rule 18 does not involve joinder of parties) as you have against

the party regardless of whether you were joined as a third party or were an original party. Rule 18(a)a) Glannon: this is the broadest of the basic joinder rulesb) There is no common transaction or occurrence requirement in Rule 18(a); you can bring in totally

unrelated claims once you have a proper claim (Glannon, p. 236)Rule 19. Joinder of Persons Needed for Just Adjudication1) Allows the court to join a party “whose joinder will not deprive the court of jurisdiction over the

subject matter” (emphasis RPD) iffa) Complete relief could not be accorded in their absence; orb) The person claims a related interest and his absence could

i) Impair his ability to protect his rights; orii) Result in risk of inconsistent decisions (Rule 19(a))

c) Note: Rule 19(a) requires the court to join such parties; “the court shall . . .”2) Court can dismiss a claim if a necessary party without joinder of an “indispensable party”. Rule 19(b).

Factors to consider:a) Extent judgment rendered in the apparently indispensable party’s absence would be prejudicial to

that party or those already partiesb) Extent to which protective provisions in the judgment, by shaping of relief or other measures, the

prejudice could be mitigatedc) Whether judgement rendered in absence would be adequated) Whether P would have a remedy if case were dismissed for nonjoinder

Rule 20. Permissive joinder. 1) Plaintiffs may sue together if:

a) They assert claims arising out of the same transaction; andb) Claims against defendants involve a common question of law or fact. Rule 20(a). c) Note: Rule 20(a) does not require parties to be joined whenever the criteria is met

2) Court can order separate trials at its discretion. Rule 20(b). 3) Rule 20 is distinct from 18 and 13

a) 18 and 13 involve claimsi) Notice, too, Rule 13 compels certain claims, whereas Rule 20(a) does not require joinder

b) Rule 20 relates to joinderc) Note: parties joining as plaintiffs diversity suit under Rule 20 must satisfy the jurisdictional

requirements of § 1332 (i.e., no aggregation and you don’t get around Strawbridge)Rule 22. Interpleader1) Individual w/claim against plaintiff may be joined and required to interplead if the claims are such that

P could be exposed to extra liability otherwise. a) Interpleader “affords a party who fears being exposed to the vexation of defending multiple claims

to a limited fund or property that is under his control a procedure to settle the controversy and satisfy his obligation in a single proceeding.” Wright and Miller § 1704

7

Ryan Preston Dahl; 917 687 7146

2) The primary test for determining the propriety of interpleading the adverse claimants and discharging the stakeholder (the so-called "first stage" of interpleader) is whether the stakeholder legitimately fears multiple vexation directed against a single fund. Wright and Miller. Id.

3) Cf. 28 U.S.C. § 1335a) District courts have original jurisdiction over any value >$500 if

i) There are two adverse claimants of diverse citizenshipii) P has deposited money/property in question with the court to be adjudicated by the court

4) The “minimal diversity” of 28 U.S.C. § 1335 held to be constitutional in State Farm Fire & Cas. Co. v. Tashire, 386 U.S. 523 (1967)a) Strawbridge rule is limited to the “words of the act of Congress” and does not create/infer a

Constitutional requirement for complete diversityb) Thus, diversity rules may be relaxed and Federal jurisdiction extended by acts of Congressc) See also Treinies v. Sunshine Mining Co., 308 U.S. 66 (1939).

i) Court held that interpleader filed by Sunshine had diversity citizenship despite the fact that Sunshine shared citizenship with one of the parties.

ii) Court held that since, Sunshine was not asserting a claim to the funds the Constitutional requirement that the controversy existed between citizens of different states was satisfied.

iii) See also Boice v. Boice, 135 F.3d 919 (3d Cir. 1943), citing Treines, held that diversity citizenship was not satisfied under interpleader when the plaintiff had an active controversy with one of the defendants, who was from the same state.

iv) Filip: The strict rule that the interpleader could have no claim in the dispute (otherwise diversity might/could be destroyed) has been relaxed somewhat in recent years

5) Interpleader may properly limit litigation to a single forum where the plaintiff’s fund is the sole object of contention. However, an insurance company cannot properly use interpleader to limit the confines of the litigation ensuing out of a mass tort claim. State Farm, supra. a) State Farm had insured a bus driver and BusCo, all OR citizens, who were defendants in a mass

tort claim arising out of a bus accident. State Farm used 28 § 1335 to interplead it’s maximum coverage amount and require that all subsequent litigation be held in the OR district court

b) “[A]n insurance company whose maximum interest in the case cannot exceed $20,000 and who in fact asserts that it has no interest at all, should not be allowed to determine that dozens of tort plaintiffs must be compelled to press their claims – even those claims which are not against he insured . . . in a single forum of the insurance company’s choosing.”i) InsuranceCo. can’t make non-Oregon citizens litigate in ORii) InsuranceCo. can’t preclude others from litigating in certain courts at all

6) Two types of interpleader:a) Statutory interpleader

i) Broader (§ 1335)ii) Citizenship of stakeholder is irrelevantiii) Only $500 at issueiv) Nationwide service of process

b) Rule interpleaderi) Based in Rule 22; complete diversity is required b/t the stakeholder and the claimants (though

you have multiple claimants from the same state)c) Amount in controversy appliesd) Service of process: The ordinary Rule (Rule 4) applies to get jurisdiction

i) Can serve w/in the State/District court sitsii) May serve outside of state only if allowed by state law

Rule 24. Intervention1) Intervention of right when:

a) Statute confers the rightb) Applicant is so situated that he would not be able to protect his interest otherwise (i.e., unless there

is a proxy through which the applicant would otherwise be able to protect his right)c) E.g., enforcement action by the SEC and there are punitive victims of fraud under the SECs

investigationi) The applicant might claim that they do not want things put into receivership as this would

take too long, and apply to intervene to get their money first

8

Ryan Preston Dahl; 917 687 7146

ii) Filip: this type of claim tends to faild) Filip: this is really the mirror image of Rule 19; if the requirements of 19(a) are met the court must

allow the claim2) Permissive intervention allowed when

a) Statute confers conditional rightb) When applicant’s claim and the other claim have a common issue of law or fact

3) Absentees and the Federal process in generala) Under some circumstances the interest of the absentee may be so affected by the litigation that it

cannot be permitted to proceed without him, at least if it is possible to bring him in. This is compulsory joinder, governed by Rule 19.

b) In other instances those who have come forward as named parties will be permitted to represent the interest of others similarly situated. This is the class action, governed by Rule 23.

c) Finally there are situations in which the absentee may be allowed in on his application. This is intervention, governed by Rule 24. Wright and Miller § 1904.

Rule 42. Consolidation; Separate trials1) Court may consolidate trials involving common questions of law or fact. Rule 42(a). 2) Court may also order separate trials regarding counter, cross, or third party claims. Rule 42(b)Rule 82. 1) RPD: THIS IS IMPORTANT2) “These rules shall not be construed to extend or limit the jurisdiction of the United States district courts

or the venue of actions therein. An admiralty or maritime claim within the meaning of Rule 9(h) shall not be treated as a civil action for the purposes of Title 28, U.S.C., §§ 1391-93.”

3) Why is this important? Three requirements for Federal court:a) A rule that confers right to hear the issueb) Personal jurisdictionc) Subject matter jurisdiction

Subject Matter Jz`urisdiction: Supplemental claims and parties 1) Federal courts have supplemental jurisdiction over “all other clams that are so related to claims in the

action within such original jurisdiction that they form part of the same case” in cases where the Federal court has original jurisdiction. 28 U.S.C. § 1367

i) It is generally accepted that Gibbs is the test that courts ought to use to determine the limits of “case or controversy” for § 1367. Concern is that it forces the courts to interpret Art. III § 2 every time they get a case

b) § 1367(c) The district courts may decline to exercise supplemental jurisdiction over a claim under subsection (a) if--i) (1) the claim raises a novel or complex issue of State law,ii) (2) the claim substantially predominates over the claim or claims over which the district court

has original jurisdiction,iii) (3) the district court has dismissed all claims over which it has original jurisdiction, oriv) (4) in exceptional circumstances, there are other compelling reasons for declining jurisdiction.

c) 28 U.S.C. § 1367(b): the court does not have supplemental jurisdiction under § 1367(a) by plaintiff’s claims in a diversity suit against parties brought in under:i) Rule 14 (Defendant third party practiceii) Rule 19 (Necessary joinder of necessary parties)iii) Rule 20 (Permissive joinder of parties)iv) Rule 24 (Permissive intervention)v) See Owen Equipment, supra, is the paradigmatic case of where § 1367(b) would bar a plaintiff

claim against such a party(1) Thus, a claim by P against a party joined under Rule 19 would require an independent

basis of jurisdictionvi) However, § 1367(b) only bars a claim by a plaintiff against the third party – the third party

may assert claims against the D. (1) However, if the third party has been brought in as a plaintiff under Rule 19 or Rule 24 §

1367(b) bars supplemental jurisdiction over any claims asserted by that party if, again, the original suit is a diversity suit

9

Ryan Preston Dahl; 917 687 7146

(2) Note, however, plaintiff’s claims or the third party claims under Rule 19 or 24 could have independent jurisdiction then they may be asserted. Again, the real problem is people using § 1367(a) to back-door the jurisdictional requirements of § 1332 in a diversity suit

vii) Rule 14: d) Court can decline to exercise jurisdiction for various reasons. 28 U.S.C. § 1367(c)(1-4)

i) A court can decline to exercise jurisdiction only if it falls within one of the exceptions under 28 U.S.C. § 1367. Executive Software N. America, Inc. v. United States District Court for the Central Dist. Of Ca. 24 F.3d 1545 (9th Cir. 1994). (1) Case arose from an underlying sexual harassment/civil rights case. District court

dismissed the underlying state claims so that they might be pursued in State court after Defendant removed the entire thing to Federal court. (a) District court had argued that such dismissals were entirely discretionary(b) P sought a mandamus order to keep the action in Federal court

(2) Circuit court held that the exceptions codified in § 1367(c)(1-3) codified the discretion articulated in Gibbs. Section 1367(c)(4) ought only be used in “exceptional circumstances”

e) Statute of limitations on the state claim is suspended (“tolled”) while the Federal claim is pending plus 30 days after dismissal. 28 U.S.C. § 1367i) Toll: Of a time period, esp. a statutory one; to stop the running of; to abate <toll the limitations

period> ii) The Constitutionality of this abrogation of state law was upheld in Jinks v. Richland Cty.,

S.C., 538 U.S. 456 (2003) (Scalia, J.)f) When the federal basis for an action drops away, the district court retains discretionary as to

whether or not to assert jurisdiction under § 1367. Similar discretion exists when the amount in controversy falls below the statutory minimum. Shanaghan v. Cahill, 58 F.3d 106 (4th Cir. 1995)i) P had brought a diversity action suit against D to recover three separate debts of $40,000,

$23,696, and $14,700. ii) District court granted summary judgment for D on the $40,000 claim, and then dismissed the

others for want of jurisdiction. Upheld by the Circuit Court. iii) Endorsed in the Third Circuit by New Rock Asset Partners, L.P. v. Preferred Entity

Advancements, 101 F.3d 1492 (3d Cir. 1996)iv) However, this holding was questioned by the Second Circuit in Wolde-Meskel v. Vocational

Instruction Project Community Servs., Inc., 166 F.3d 59 (2d. Cir. 1999), holding that a court’s original jurisdiction over a matter was unimpaired by the reduction in claims that bring the total under the statutory amount. (1) Jurisdiction was established based on the time at which the suit was brought and would

not/could not chance (citing Red Cab to that effect)(2) Hence, district court would have no discretion to dismiss a claim properly brought under

§ 1332 if the total fell under the threshold amount – application a supplemental jurisdiction doctrine (as Shanaghan did) was therefore inappropriate since the original basis for jurisdiction was not § 1367

g) Question: Does § 1367 challenge the ruling of Zahn v. Int’l Paper Co. which required that each party had to meet the jurisdiction amount to be joined to a class action suit?i) Seventh Circuit addressed the issue in Stromberg Metal Works v. Press Mechanical, Inc., 77

F.3d 928 (7th Cir. 1996) (Easterbrook, J.)(1) 2 Ps of diverse citizenship sued for collection of unpaid debts. One P met the statutory

minimum and one did not(2) “Section 1367(a) has changed the basic rule by authorizing pendent-party jurisdiction

[over claims without an independent basis of jurisdiction], and that change effects . . . Zahn.”

(3) “The claims of [both plaintiffs] satisfy [§ 1367(a, c)], however. The two plaintiffs are affiliated corporations under common control. The claims arose out of the same construction project. According to the complaint, the defendants pursued a single course of action--fraudulently representing to Bechtel that the subcontractors had been paid, and thus obtaining money intended for the subcontractors without remitting it--that injured both plaintiffs. The same form of purchase order was used for both subcontracts, so

10

Ryan Preston Dahl; 917 687 7146

factual and legal issues are identical. This strikes us as exactly the sort of case in which pendent-party jurisdiction is appropriate”(a) Court also distinguished between when an Owen-type situation in which a third party

defendant would be brought in, and the present case in which it was essentially an identical plaintiff

ii) However, both the Third and Tenth Circuits have concluded that Zahn is not overruled, based on recourse to legislative history

2) Federal court may assert jurisdiction over a matter of state law when they arise from the same “nucleus of operative fact” if the federal claim has substance sufficient to confer subject matter jurisdiction on the court. United Mine Workers of America v. Gibbs, 383 U.S. 715 (1966)a) Case arose from an action by Gibbs against the union. Gibbs claimed the union violated a Federal

statute regulating union picketing and also a state common law claim for business interference.b) If, considered without regard to the nature of the claim (State or Federal) the claims would

ordinarily be expected in the same action then the Federal court can assert pendent jurisdiction over the state claim if the Federal jurisdiction is strong enough.

c) Court based its ruling heavily upon the liberalized pleading of the Rules, asserting the Federal court’s right to adjudicate the state claim:i) “Pendent jurisdiction . . . permits the conclusion that the entire action before the court

comprises but one Constitutional ‘case’.” (1) Consistent with a broad reading of Art. III § 2 “case or controversy”

ii) This supplemental jurisdiction is exercised at the discretion of the court rather than as a matter of plaintiff’s right(1) State claim should be dismissed if Federal claim is dismissed(2) Importance of the underlying Federal issue should weigh in a court’s consideration of

whether or not to exercise discretion(3) If, during litigation, it appears the Federal claim is “only an appendage” of the state

claim, the state claim may be dismissed(a) This led to the “substantiality doctrine”: there must be a substantial Federal issue at

workiii) This is a broad construction of pendent jurisdiction

d) Rule was subsequently codified in 28 U.S.C. § 1367 as part of the Judicial Improvements Act of 1990

3) Federal court may not assert pendent jurisdiction over a third party in a claim not arising under Federal law. Aldinger v. Howard, 427 U.S. 1 (1976)a) P, a citizen of WA had brought suit against Spokane County police officers under the Civil Rights

Act (42 U.S.C. § 1983). He then attempted to bring action against the County in an action under state law. (Construction of 42 U.S.C. § 1983 at the time did not hold that a municipality was subject to suit. This has subsequently changed).

b) Court rejected the argument that the Gibbs “common nucleus of operative fact” test provided Federal jurisdictioni) Factually different in that Gibbs involved two parties already involved in litigation, this would

involve a third partyii) Construction of the underlying Federal claim made it clear that such Federal claims were

disfavored (as opposed to the Federal issues at work even underlying the state claim in Gibbs)c) See also, Finley v. Unites States, 490 U.S. 545 (1989)

i) P had sued the U.S. for the FAA’s negligence under 28 U.S.C. § 1346(b). P then attempted to amend her complaint to join both the municipality and power company under state claims.

ii) Court held that federal jurisdiction over one party did not create jurisdiction additional parties (as opposed to claims) under state law(1) Court emphasized this distinction of “pendent jurisdiction” (Gibbs) and “pendent party”

jurisdiction (disfavored in Aldinger)iii) “Just as the statutory provision ‘between citizens of different states and no one else . . . so also

here we conclude that ‘against the United States [in the language of the Federal Tort Claims Act] means against the United States and no one else.”

11

Ryan Preston Dahl; 917 687 7146

4) Aldinger and Finley have been overruled by the last sentence of 28 U.S.C. § 1367(a): “Such supplemental jurisdiction shall include claims that involve the joinder or intervention of additional parties.” (emphasis RPD). a) “Congress responded [to Finley] by enacting a statute . . . that codifies pendent jurisdiction under

the name "supplemental jurisdiction," expressly including pendent party jurisdiction.” Brazinski v. Amoco Petroleum Additives Co. 6 F.3d 1176, 1181 (7th Cir. 1993) (Posner, J.)

b) Compare Guaranteed Systems, Inc. v. American National Can Co., 842 F. Supp. 855 (M.D.N.C. 1994)i) D removed action to Federal court on diversity grounds and asserted a counterclaimii) P’s motion to implead a non-diverse third party on a state claim was denied. Court held that

the supplemental claim fell squarely within Owen, but was pre-empted by § 1367. (1) “If it were not bound by the plain terms of the statute, the court would be swayed by the

interests of justice and efficiency to construe Plaintiff's claim as a claim by a defendant against a person made party under Rule 14 rather than a claim by a plaintiff, and thus to allow it to proceed under 28 U.S.C. § 1367(b).”

(2) Note: subsequent cases have emphasized how Guaranteed Systems only states that a plaintiff may not implead a third party defendant w/out diversity per § 1332. See, e.g., Blackwood, Inc. v. Ventresca, 2002 U.S. Dist. LEXIS 24745 (E.D.Pa. 2002)

5) Federal courts have ancillary jurisdiction to enforce their decrees, such as judgment. Kokkonen v. Guardian Life Ins. Co. of America, 511 U.S. 375 (1994)

Removal1) Possibility for removal created and defined by 28 U.S.C. § 1441

a) Defendant has right to remove action to district court covering the area if it involves an issue over which Federal courts have original jurisdiction

b) Any civil action over which the district courts have original jurisdiction for claims/rights arising under the Constitution “shall be” removable without regard to citizenship. Any other action is removable if none of the parties and served as defendants are citizens of the State in which the action is served

c) If claims arising under 28 U.S.C. § 1331 (Federal question) are joined with “separate and independent” non-removable claims, the entire action can be removed to Federal courti) The entire action can be moved!ii) “Whenever a separate and independent claim or cause of action within the jurisdiction

conferred by section 1331 of this title, is joined with one or more otherwise non-removable claims or causes of action, the entire case may be removed and the district court may determine all issues therein, or, in its discretion, may remand all matters in which State law predominates.” 28 U.S.C. § 1441(c).

d) Can remove an action brought in State court against a foreign state to Federal courte) Court

i) may remove the action to district court if (1) Action could have been brought under 28 U.S.C. § 1369 (multi-party jurisdiction); or(2) Defendant could have been brought in under 28 U.S.C. § 1369 even if the action in state

court could not have been brought in as an original matterii) If action is removed, damages determined on remand to state courtiii) Order to remand not effective until 60 days after order determining liability; choice of law and

appeal w/respect to liability may be taken in this 60-day period. Remand is not effective until after resolution of any appeal

iv) A decision concerning remand shall not be reviewable by appealv) [Revisit me]vi) Nothing in § 1441 restricts district court’s ability to dismiss an action on grounds of

inconvenient venuevii) District court to which an action is removed under § 1441 may also hear claims that might not

have been brought in State court2) The original plaintiff to an action in State court may not remove the case to Federal court even if he is

named as a defendant in a counterclaim within the original action. Shamrock Oil & Gas Corp. v. Sheets, 313 U.S. 100 (1941)

12

Ryan Preston Dahl; 917 687 7146

a) Sheets was a citizen of Texas who had brought an action against Shamrock, not a citizen of Texas, in Texas state court. Shamrock filed a counterclaim against Sheets. Sheets removed the case to Federal district court. Sheets won on both the original claim and the counterclaim in district court. Shamrock appealed on jurisdiction grounds.

b) Court held that 28 U.S.C. § 1441 did not allow a plaintiff to remove a case to Federal court, even as a party to a counterclaim.i) Referring to pre-reform cases, “The Court ruled that the plaintiff, having submitted himself to

the jurisdiction of the state court, was not entitled to avail himself of a right of removal conferred only on a defendant who has not submitted himself to the jurisdiction.” (emphasis RPD)

ii) “We can find no basis for saying that Congress, by omitting from [§ 1441] all reference to 'plaintiffs,' intended to save a right of removal to some plaintiffs and not to others.”

3) Once a case has been removed to Federal court, a plaintiff cannot destroy jurisdiction by amending his complaint to claim an amount below the statutory minimum. St. Paul Mercury Indem. Co. v. Red Cab Co., 303 U.S. 283 (1938)a) “Thus events occurring subsequent to removal which reduce the amount recoverable, whether

beyond the plaintiff's control or the result of his volition, do not oust the district court's jurisdiction once it has attached.” Id. at 293.

b) But see Capps v. New Jellico Coal Co., 87 F.Supp. 369 (E.D.Tenn. 1950): P cannot defeat removal by claiming, in his original complaint “not more than the [jurisdictional threshold amount]”i) However, this can be distinguished on the facts – i.e., if the amount prayed for is at odds with

the claimed damages, the court can overlook the claimii) “The value of the matter in controversy here is what the compensation statute allows. The

suggestion in the original petition that plaintiff's injury may prove to be not so bad as it seems, does not erase the allegation elsewhere that 'he will be disabled the remainder of his life as to ability to carry on a gainful occupation.' The petition would leave the impression that plaintiff's actual recovery will be less than [the statutory threshold], but at the same time it does not show a purpose to foreclose him from proving total permanent disability when the case is tried.”

PERSONAL JURISDICTION AND VENUETraditional bases for jurisdiction1) Constitutional basis

a) “Full Faith and Credit shall be given in each State to the public Acts, Records, and judicial Proceedings of every other State. And the Congress may be general Laws prescribe the Manner in which such Acts, Records and Proceeding shall be proved, and the Effect thereof.” Art. IV § 1

b) “The Citizens of each State shall be entitled to all Privileges and immunities of Citizens in the several states.” Art. IV § 2

c) “All persons born or naturalized in the United States, and subject to the jurisdiction thereof, are citizens of the United States and of the state wherein they reside. No state shall make or enforce any law which shall abridge the privileges or immunities of citizens of the United States; nor shall any state deprive any person of life, liberty, or property, without due process of law; nor deny to any person within its jurisdiction the equal protection of the laws.” 14th Amendment, § 1

2) Territoriality Theorya) Since each state had exclusive power over all persons w/in its orders, it could render binding

judgments in any suits brought against those persons. i) Referred to as in personam jurisdiction: “of or against a particular person”ii) Not allowed over people outside the state as it violated the sovereignty of another stateiii) Also, traditionally held that the state could render and enforce a valid in rem judgment against

property within the stateb) Most important development in the expansion of sovereignty was the development of quasi-in-

rem jurisdictioni) In quasi-in-rem jurisdiction property in a particular state can be used to establish jurisdiction

to adjudicated rights that might/might not relate to the property

13

Ryan Preston Dahl; 917 687 7146

(1) Different from traditional in rem jurisdiction in that the property was important only to the remedy and not the focus of the substantive claim at issue

(2) Could also be used when the personal rights at issue were totally unrelated to the propertyii) In either instance, quasi-in-rem jurisdictionwas asserted by attaching or seizing property at the

commencement of the action. (1) Utilizing the fiction that property was a manifestation of the D the court could adjudicate

a personal issue(2) A D who entered to defend submitted to the authority of the court(3) If D did not appear and defaulted, judgment was limited to the value of the property; no

full faith and credit was given beyond that amount. iii) Attachment of the property is key to quasi-in-rem jurisdiction as it established court’s

jurisdiction and ensured the enforceability of the judgment P might obtain. Pennoyer. 3) A State court must attach property held by an out-of-state defendant at the outset of the action in order

to assert personal jurisdiction over the defendant absent in-state personal service or voluntary appearance. Without such an attachment, there is no jurisdiction. Pennoyer v. Neff, 95 U.S. 714 (1877).a) Neff had lost his land as a result of a suit by one J.H. Mitchell, an attorney, for services rendered.

Neff, not a resident of the state of Oregon, was not served with process and did not appear in the action. Judgment was entered by default, with constructive notice having been given by virtue of a publication of summons in Oregon. Mitchell took title to the land by virtue of the judgment against Neff and sold it to Pennoyer through a quitclaim deed. Neff subsequently sought to recover his land from the current owner, Pennoyer. i) “in personam” judgment: a judgment against the rights and assets of the individual defendant

(as opposed to a particular asset, which would of course be in rem judgment)ii) Current case (Pennoyer v. Neff) is brought in Federal court under a theory of ejectment.

b) Court held that the Oregon court lacked jurisdiction over Neff because it failed to attach Neff’s land until after it had judged against him.i) “[The Oregon court’s] first connection with the case was caused by a levy of the execution. It

was not, therefore, disposed of pursuant to any adjudication, but only in enforcement of a personal judgment, having no relation t the property rendered against non-resident without service or process . . . or appearance.”

c) Based on two principle of jurisdiction:i) Jurisdiction over the “civil status and capacities of its inhabitants” is limited to the individual

states; butii) no state “can exercise direct jurisdiction and authority over persons or property without [i.e.,

outside] its territory”(1) Anything else would be an “encroachment upon the independence of the State in which

the [defendant is] domiciled or the property is situated . . . .”(2) Filip: Presence is necessary and sufficient to establish jurisdiction for the Pennoyer court;

based on the “full faith and credit” clause of Art. IV § 1; but, again, you only need to give “full faith and credit” to something that is valid

d) The State may, however, subject property owned by non-citizens within its limits to claims by its own citizens against that land. But if the non-citizen has no property, there is nothing to adjudicate.i) Constructive service by publication of process is achieved once the court has seized the

property in question(1) Based on the theory that “property is always in the possession of its owner . . . its seizure

will inform him [of the action].” ii) However, mere publication of process (as in Pennoyer) is insufficient when the suit is in

personame) The seizure of property is therefore an essential step through which the court asserts its

jurisdiction over the out of state partyi) “the jurisdiction of the court . . . is only incidental to its jurisdiction over the property. Its

jurisdiction in that respect cannot be made to depend upon facts to be ascertained after it has tried the cause and rendered the judgment.”

14

Ryan Preston Dahl; 917 687 7146

ii) Also, seizure is necessary as jurisdiction could be defeated subsequent to the filing of the action if the defendant were to sell or dispose of his property during the action

iii) “[T]he validity of every judgment depends upon the jurisdiction of the court before it is rendered, not upon what may occur subsequently.”

f) This limitation on the jurisdiction of a particular state court is based in the 14th Amendmenti) Opinion notes that prior cases had attempted to enforce judgments by one state court against

out of state parties under the “full faith and credit” language of Art. IV § 1ii) 14th Amendment, however, clearly states that such an extension of a particular state’s

jurisdiction would work a violation of due process: “proceedings in a court of justice to determine the personal rights and obligations of parties over whom that court has no jurisdiction do not constitute due process of law.”

iii) The defendant must be brought within the jurisdiction of the court either by:(1) “Service of process within the State”; or(2) Voluntary appearance

g) Service of process as regards a non-citizen is only affected where the property of the non-citizen is seized by the state and brought under the control of the court so long as the case does not involve a determination of the status of the defendant.i) Court refers to this as “in the nature of a proceeding in rem”ii) i.e., the quasi-in-rem jurisidictioniii) Court uses the example of marriage as an example of “personal status” that does not fall

within the realm of quasi-in-rem jurisdiction(1) Court also notes that its ruling does not prevent a state from requiring an out-of-state

individual to have an agent or person in state to take service of any actionh) Filip: If you make a limited appearance to contest the in personam jurisdiction of the court, and

you lose, you have still preserved this issue for appeal as it is a requirement of due processi) Note: Holmes reads Pennoyer to state that “There is no dispute that service by publication does

not warrant a personal judgment against a nonresident. Some language of Pennoyer v. Neff would justify the extension of the same principle to absent parties” McDonald v. Mabee, 243 U.S. 90, 92 (1917) (Holmes, J.)

4) A person in transit in an airplane over a particular jurisdiction is deemed to satisfy the requirements of notice when he is served in transit. Grace v. MacArthur, 170 F.Supp. 442 (E.D.Ark. 1959)a) RPD: Remember, Pennoyer requires in-state service, supra. b) Note: The court’s big concern was whether or not state sovereignty extended upwardsc) RPD: Compare to Burnham v. Superior Court, infra. This case seems to come out the same way

under Burnahm and probably has a better justification (if you adopt Scalia’s view. Not so much under Brennan)

5) Court has jurisdiction to convict an out-of-country citizen for contempt of court resulting from failure to comply with a subpoena. Blackmer v. United States, 284 U.S. 421 (1932)a) U.S. statute makes it a criminal offense for an out of country U.S. resident to fail to obey a

subpoena without a showing of good causeb) As a citizen of the U.S., the defendant’s presumed knowledge of the law was adequate to supply

notice to the defendant and the jurisdiction of the court for in personam judgment i) “The jurisdiction of the United States over its absent citizen, so far as the binding effect of its

legislation is concerned, is a jurisdiction in personam, as he is personally bound to take notice of the laws that are applicable to him and to obey them.”

c) Similarly, a state court’s jurisdiction over its citizens still domiciliaries of that state does not cease simply because that citizen is absent. This gives a state court jurisdiction to render in personam judgement after appropriate service. Milliken v. Meyer, 311 U.S. 457 (1940)i) Meyer, a Wyoming resident, had been sued by Milliken. Service was effected on Meyer in

Colorado under a WY statute allowing such service. Court rendered in personam judgement against Meyer.

ii) Supreme Court upheld the judgment on the basis of Blackmer, making no distinction between “citizen” and “domicile”: “the state which accords him privileges and affords protection to him and his property by virtue of his domicile may also exact reciprocal duties.” (1) Hence, the WY judgment was entitled to full faith and credit

15

Ryan Preston Dahl; 917 687 7146

iii) Filip: This case perhaps conflates “residence” and “domicile” a little too loosely. But the important part is that if you go to the place where someone is a citizen you can sue them there even if the defendant has moved away(1) It’s a reciprocity concept between the states.

6) Adam v. Saenger, 303 U.S. 59 (1938)a) P was assignee of a cross-claim against the Beaumont Export Co. The original cross-claimant

brought his action in CA state court and won on a default (the question of service to Co.’s lawyer was disputed).

b) P attempted to enforce his the CA court’s judgment in Texas. Texas court dismissed on the basis that the CA court lacked jurisdiction since there was no proper service under Texas law and therefore a nullity (emphasis RPD).

c) Court held that the Texas court misapplied the law (by using Texas rather than CA law) and failed to give full faith and credit as required by Art. IV § 1i) “But the question presented by the pleadings is the status of a cross-action under the

California statutes, not under those of Texas. We think its status is adequately disclosed by the California statutes and decisions pleaded by petitioner, and is that for which he contends”

d) Nor was the CA statute that allowed final judgment to be entered in personam to an out-of-state party based on notice served to an attorney a violation of the 14th Amendmenti) “There is nothing in the Fourteenth Amendment to prevent a state from adopting a procedure

by which a judgment in personam may be rendered in a cross-action against a plaintiff in its courts, upon service of process or of appropriate pleading upon his attorney of record. The plaintiff having, by his voluntary act in demanding justice from the defendant, submitted himself to the jurisdiction of the court, there is nothing arbitrary or unreasonable in treating him as being there for all purposes for which justice to the defendant requires his presence. It is the price which the state may exact as the condition of opening its courts to the plaintiff.”

Modern Personal Jurisdiction1) A personal judgment entered against a nonresident who has not been served with process within the

state of notice nor appeared voluntarily is a nullity. Nor does the mere transaction of business within a state imply a consent to be bound by the law of that state. However, a state may determine that certain activities require or impliedly create an agent by which service may be performed on a non-resident. Hess v. Pawloski, 274 U.S. 352 (1927)a) Pawloski, a MA resident, was injured by Hess in a car crash in MAb) MA law held that any nonresident operating a car consented that the state registrar would serve as

agent for any “lawful processes . . . growing out of any accident or collision.” Hence, any actual process was served on the registrar.

c) Court held that this requirement did not work a violation of the 14th Amendmenti) State has power to regulate its highways; this extends to residents and non-residents. ii) State may therefore exercise this sort of jurisdiction based on this sort of implied consent

doctrine(1) In part, might be a reflection of the technological advances (and limitations) of the time;

the forms required by Kane v. New Jersey, 242 U.S. 160 (1916) (Brandeis, J.) just aren’t realistic/possible

iii) RPD: Pennoyer seems to have foreseen this (but not quite to this degree)d) See also Henry L. Doherty & Co. v. Goodman, 294 U.S. 623 (1935): Court used Hess to uphold an

IW law applying a similar concept to assert jurisdiction over a nonresident selling securities in IW2) Evolution of jurisdiction over corporations:

a) Consent theory: Corporation could only operated with the consent of the State. Hence the requirement that it designate an Agent to be served

b) Presence theory: “A foreign corporation is amenable to process . . . if it is doing business within the State in such a manner and to such extent as to warrant the inference that is present there.” Phil. & Reading Ry. Co. v. McKibbin, 243 U.S. 264 (1917) (Brandeis, J.)

3) A court may assert personal jurisdiction over a nonresident defendant only so long as there exist minimum contacts between the defendant and the forum. International Shoe v. Washington, 326 U.S. 310 (1945)a) Int’l Shoe had allegedly failed to pay its taxes incurred as a result of having employees in WA.

Shoe had no agents in WA; “notice of assessment for the years in question was personally served

16

Ryan Preston Dahl; 917 687 7146

upon a sales solicitor employed by appellant in the State of Washington, and a copy of the notice was mailed by registered mail to appellant at its address in St. Louis, Missouri”i) Shoe had made a special appearance to claim lack of jurisdictionii) Very doubtful there could be personal jurisdiction under Pennoyer; couldn’t have quasi-in-

rem jurisdiction as there was no real property (the state of WA did not want shoes/chattels)b) Two prongs first establish presence per International Shoe:

i) Traditional notions of fair play and substantial justice(1) Convenience is viewed from the p.o.v. of the defendant

ii) “Due process” requires and is ensured by the nature and the quality of activity in relation to the fair and orderly administration of the laws(1) A shift from “is the defendant there” (Pennoyer) to “is it fair” (International Shoe)(2) Utter dearth of discussion of sovereignty

c) Presence (as opposed to the literal presence required by Pennoyer) based on:i) Level of activities in the stateii) Relation of the lawsuit to the activities of the companies in the state

(1) Primary v. secondary activity distinction; does the suit arise from the primary conduct of the corporation?

iii) “Presence” as required by due process/jurisdictional requirements is satisfied when the operations there are “continuous and systematic” and give rise to action

iv) “Presence” is not satisfied by “single and isolated” activitiesv) “Presence” is also satisfied when the continuous presence is so substantial as to allow suit

though the action involves dealings unrelated to the corporation’s actions in the statevi) Court also admits “presence” is satisfied by Hess-type exception in which consent is

impliedly gotFilip’s Quadrant:

Degree of relatedness Personal jurisdiction? Personal jurisdiction?Highly related to Ds activities Questionable (4) Personal JurisdictionUnrelated to D’s activities No personal jurisdiction Questionable

Isolated, sporadic contact Continuous and systematicLevel of contact

vii) Note: The court also weighs this against the state interest as well(1) This becomes a clear case of personal jurisdiction for the court (highly related to

activities and based on continuous and systematic connection to the state)d) RPD: Notice the departure from Pennoyer’s claim that “[t]he authority of every tribunal is

necessarily restricted by the territorial limits of the State in which it is established.”e) RPD: Implied analysis:

i) Was the contract irregular or casual?ii) Did it result in a large volume of business?iii) Did the cause of action arise out of these activities?iv) Is it reasonable in the ends of substantial justice to allow the suit?

4) Review: International Shoe uses contact with the forum in question in two ways:a) General jurisdiction : D may have sufficient contact with the forum to warrant asserting

jurisdiction over it for all mattersi) Occurs when contacts b/t a D and state are so continuous and systematic as to submit D to the

jurisdiction of the court on any claimb) Specific jurisdiction : D may have sufficient contact with the forum to warrant asserting

jurisdiction for matters related to its activity within the forum without have sufficient contact to warrant general jurisdictioni) This is Quadrant 4 (above)

c) Pennoyer categories still exist post Shoei) Quasi-in-rem (suits based on property)ii) In rem (suits about property)

17

Ryan Preston Dahl; 917 687 7146

iii) Transient jurisdiction; process can be served even if you’re just passing through the state. The fact that you were served gives jurisdiction over you assuming you were not induced to enter the jurisdiction through fraud

d) Filip: Pennoyer – in the modern world – had created enormous jurisdiction problems for corporate activity. For corporations to exist, all sides involved in corporate activities (consumers, partners, etc.) must feel that they are not forced to enter into an unfair situationi) Hence, there must by a system in place to deal with corporate and personal affairs; you can’t

make someone go half way around the country to litigate a dispute. That type of system just doesn’t work

ii) The “Corporate registration statutes” requiring a company to submit to jurisdiction if it does business in that state are most susceptible to the most dubious parties (e.g., Snake Oil companies) who need to be brought into court(1) These were also fairly conclusory sorts of tests, e.g., “doing business within a state in

such a manner and to such an extent as to warrant the inference that it is present there.”iii) Still, presence is the key, but there would still be gaps. Various other tests (employees present

in state, etc.) were insufficient to make this work (e.g., “that’s not an employee, it’s an independent contractor”)

Specific jurisdiction1) Long-Arm Laws

a) Hess and International Shoe encouraged states to develop laws that could expand jurisdictionb) “Long-arm” statutes predicate jurisdiction over nonresidents upon the commission of a specified

act or acts within the stateiv) E.g., in IL, an individual or Corp. is subject to IL jurisdiction if he transacts any business

within the state, commits a tort within the state, uses/owns/possesses any property in IL, or contracts to insure someone in IL

v) IL expanded this to include family law (esp. divorce) latervi) Modern long-arm statute vary in the degree to which they cover non-residents

e) However , a Long-Arm statute must be consistent with the due process clause of the Constitutionc) Two types of longarm statutes:

i) “Enumerated” statutes: The statute clearly specifies which acts support jurisdiction by the forum State(1) The court then determines whether the act falls under the statute and(2) Whether jurisdiction is consistent with Due Process

ii) “Unenumerated statutes”(1) E.g., “IL authorizes jurisdictions up through the full extent authorized by the Due Process

Clause”(2) Hence, it’s just a Constitutional analysis

2) Premiums mailed to an insurance company by a customer in California are sufficient to satisfy due process requirements to establish personal jurisdiction over non-resident insurance company. McGee v. Int’l Life Ins. Co., 355 U.S. 220 (1957)a) Lowell Franklin, a CA resident, had purchased a life insurance policy from D’s predecessor in

interest, an AZ-based life insurance company. Franklin made payments from California. D conducted no other business in CA. D was based in TX. When P died as a result of an apparent suicide, his beneficiary attempted to collect, but D refused. i) P brought suit in CA court through a long arm statute and won. ii) P attempted to enforce judgment in TX.iii) TX held the CA Long-Arm statute to be Unconstitutional

(1) RPD: See the specific provision in IL law for insurers, supra(2) cf. Adam v. Saenger, 303 U.S. 59 (1938)

b) Court held that the insurance payments were sufficient to establish jurisdiction and that the CA law did not violate the 14th Amendmenti) “The contract was delivered in California, the premiums were mailed from there and the

insured was a resident of that State when he died. It cannot be denied that California has a manifest interest in providing effective means of redress for its residents when their insurers refuse to pay claims. These residents would be at a severe disadvantage if they were forced to follow the insurance company to a distant State in order to hold it legally accountable. When

18

Ryan Preston Dahl; 917 687 7146

claims were small or moderate individual claimants frequently could not afford the cost of bringing an action in a foreign forum--thus in effect making the company judgment proof. Often the crucial witnesses--as here on the company's defense of suicide--will be found in the insured's locality.” Emphasis RPD(1) Pay attn. to how the highlighted part weighs into Brennan’s dissent in World-Wide

Volkswagonc) Pay att’n to the importance of convenience here. The interest of the plaintiff dominates the court’s

analysis. Filip: This is the high water mark of personal jurisdiction. d) Filip: Insurance is generally considered to be solely a matter of state concern; the Federal

government does not like to get involved3) Unilateral activity of those who claim relationship with a nonresident defendant is insufficient to

satisfy even the minimal contact requirement necessary for in personam jurisdiction over a necessary party. Hanson v. Denckla, 357 U.S. 235 (1958) (Warren, C.J.)a) Case arose over a dispute regarding the appointment of a trust. Parties tried to assert jurisdiction

in a Florida court over the trustee, a Delaware corporation. The DL trustee had no assets, no office, no solicitation of business, and no business in Florida.

b) Court held that the DL defendant lacked sufficient contact with Florida under which a Florida court might assert personal jurisdiction. Without the defendant “purposefully avail[ing]” himself of the State forum there can be no jurisdiction i) “It is essential in each case that there be some act by which the defendant purposefully avails

itself of the privilege of conducting activities within the forum State, thus invoking the benefits and protections of its laws.”

ii) However, Black, J., dissenting contended that the assignment by the beneficiary (which created the issue) which was made in FL (and related to a will made in FL) should have been sufficient to confer jurisdiction to the state of FL (and its court) court which outweighed any other interest at issue.

iii) World-Wide Volkswagon, infra, cites Hanson in support of the proposition that “Even if the defendant would suffer minimal or no inconvenience from being forced to litigate before the tribunals of another State; even if the forum State has a strong interest in applying its law to the controversy; even if the forum State is the most convenient location for litigation, the Due Process Clause, acting as an instrument of interstate federalism, may sometimes act to divest the State of its power to render a valid judgment.”

iv) Note how Hanson sort of involves a choice of law question; whether or not the trust would be adjudicated per DL or FL law. Hence, the Court’s concern with the interests of the states (though this analysis proceeds on a different dimension than the jurisdictional question)

c) See also, All-State Ins. Co. v. Hague, 449 U.S. 302 (1981): for State’s substantive law to be selected in a constitutionally permissible manner, that State must have a significant contact or significant aggregation of contracts creating state interests, such that choice of law is neither arbitrary nor unfair.i) Plurality opinionii) Plurality upheld MN’s decision to apply its own law in an action by a former WI resident who

had moved to MN just after filing a claim under an insurance policy made in WI owned by WI resident who died in a crash.

iii) Casebook: Hague allows extraordinary latitude in developing choice of law rules. Due process restrictions on state jurisdiction are greater than choice of law.

d) Pay att’n to the importance of sovereignty in this case4) Foreseeability alone does not enable a court to assert in personam jurisdiction over a nonresident

defendant who has no other connections to the state and has in no way availed itself of the laws or privileges of conducting business in that state. World-Wide Volkswagon Corp. v. Woodson, 444 U.S. 286 (1980). Can also phrase it as: Consumer’s unilateral act of bringing D’s product into forum state is insufficient basis on which to support jurisdiction. (this is how holding was construted in Asahi). a) VW (the actual car dealership) and its distributor (which was limited to the Tri-State area) were

made parties to a product defect action in Oklahoma as a result of a car accident. The car was sold in New York. Neither VW nor its distributor conducted any business in Oklahoma. Plaintiffs were citizens of NY. OK supreme court ruled that the “foreseeability” that VW’s products would

19

Ryan Preston Dahl; 917 687 7146

be driven through OK was sufficient to warrant personal jurisdiction (the manufacturer and the importer did not contest jurisdiction). Dealership and distributor claimed this worked a violation of the Fourteenth Amendment

b) “Minimum contacts” serves two functions: i) Protects the D against the burden of litigating in a distant/inconvenient forumii) Ensures that states do not encroach on the sovereignty of other states

c) World-Wide Volkswagon cites Hanson, supra, in support of the proposition that “Even if the defendant would suffer minimal or no inconvenience from being forced to litigate before the tribunals of another State; even if the forum State has a strong interest in applying its law to the controversy; even if the forum State is the most convenient location for litigation, the Due Process Clause, acting as an instrument of interstate federalism, may sometimes act to divest the State of its power to render a valid judgment.”

d) Court held that there was insufficient contact b/t VW and OK to warrant personal jurisdiction:i) “[VW] carry on no activity whatsoever in Oklahoma.”

(1) This weighs in, significantly, against a McGee-type analysis of whether or not OK had a compelling interest in the case: there would be no meaningful evidence/interest in OK

(2) After all, the care aren’t designed there, manufactured there, or assembled thereii) “’Forseeability’ alone has never been a sufficient benchmark for personal jurisdiction under

the Due Process Clause” citing Hanson in support(1) In this case, a distinction of constitutional magnitude will not be made simply because the

object in question has wheels on it. iii) Court also distinguished the case form one in which a Corporation “avails itself of the

privilege of conducting activities within the forum State.”iv) Court also rejected the argument that since VW service centers existed in OK there was

jurisdiction:(1) “In our view, whatever marginal revenues petitioners may receive by virtue of the fact

that their products are capable of use in Oklahoma is far too attenuated a contact to justify that State’s exercise of in personam jurisdiction over them.”

e) But see Brennan, J., dissentingi) Court placed too little weight on the State interest “and fail to explore whether there would be

any actual inconvenience to the defendant” as required by McGee. ii) Any burden “must be of constitutional dimension. Due process limits on jurisdiction do not

protect a defendant from all inconvenience of travel.”(1) RPD: This misses the fucking point.

iii) Also puts greater weight on the “foreseeability” (though even for Brennan this is not dispositive): “the sale of an automobile does purposefully inject the vehicle into the stream of interstate commerce so that it can travel to distant States.”

iv) Brennan prefers a test under which “minimum contacts must exist among the parties, the contested transaction, and the forum State’s interest in the dispute. The contacts between any two of these should not be determinative”(1) Brennan implies that he might be willing to abandon the International Shoe framework

(the technological changes post-shoe have been at least as significant as the ones post-Pennoyer)

f) See also, Ins. Corp. of Ireland v. Compagnie des Bauxites de Guinee, 456 U.S. 694 (1982) characterizing World-Wide’s restriction on state sovereignty as a proper function of the individual rights asserted in the Fourteenth Amendmenti) “[The Fourteenth Amendment] is the only source of the personal jurisdiction requirement and

the clause itself makes no mention of federalism concerns. Furthermore, if the federalism concept operated as an independent restriction on the sovereign power of the court, it would not be possible to waive the personal jurisdiction requirement: Individual actions cannot change the powers of sovereignty, although the individual can subject himself to powers from which he may otherwise be protected.”

g) Personal injury suit against Hong Kong manufacturer and purchasing agent for flammable clothes bought at Wal-Mart could be properly dismissed for lack of personal jurisdiction. Nelson by Carson v. Park Indus., Inc., 717 F.2d 1120 (7th Cir. 1983)i) P was injured when her shirt ignited, and she sued the HK manufacturer and purchasing agent

20

Ryan Preston Dahl; 917 687 7146

ii) Court interpreted World Wide Volkswagon to hold that a “secondary” manufacturer (such as the ones in Volkswagon or in Carson) or distributor’s foreseeable market is narrower than that of a primary manufacturer or distributor

iii) It is questionable as to whether or not this case would come out the same way post-Asahi5) A forum may exercise personal jurisdiction over a nonresident who purposefully directs his activities

towards forum residents. Burger King Corp. v. Rudzewicz, 471 U.S. 462 (1985)a) D was a MI-based Burger King franchisee. BK was a FL based corporation. Franchise contract

(which had a 20-year duration) required that all disputes would be resolved in Federal Court in Florida. All payments and correspondence were directed to Florida. However, all regular contact was with BK’s MI district HQ. BK subsequently sued in FL for breach of contract when D fell behind in his contractually-required payments. D challenged the venue/jurisdiction of FL district court. FL long arm statute allowed for FL jurisdiction over breach of contract

b) Court held that the FL courts jurisdiction was proper since the defendant had created the substantial connection with the forum statei) Defendant’s conduct, as opposed to foreseeability, was the key part of the analysisii) Note: Physical presence is not required to create jurisdiction

c) Once D’s conduct had created these ties, it should then be evaluated against:(1) State’s interest in adjudicating the dispute(2) Burden of litigating in that particular forum(3) Balancing of these issues could permit a court to uphold jurisdiction “upon a lesser

showing of minimum contacts than would otherwise be required.”(4) Legal questions could be resolved through choice-of-law issues(5) Note: Black, J. (author of International Shoe) would probably agree with the outcome,

but not the sort of fuzzy test applied by the court; Int’l Shoe was, after all, concerned with keeping courts from making the Due Process clause whatever courts wanted

ii) While a contract alone could not create this requisite level of contact, the contract considered with its intent behind the formation could satisfy the minimum contact requirement (this is known as the “contract-plus” test)(1) “[P]rior negotiations and contemplated future consequences, along with the terms of the

contract and the parties actual course of dealing – that must be evaluated in determining whether the defendant purposefully established minimum contacts within the forum.”

(2) Also, choice of law and choice of venue type provisions(3) Court noted that contract formation defenses (fraud, unconscionablility) could defeat this,

but no such defenses were available here(4) Filip: Less clear how the Burger King test plays out in the one-purchase consumer goods

type situation(5) RPD: Court seemed that the fact that that FL jurisdiction was stated in the contract

mattered considerablyiii) Also balanced with plaintiff’s interest in getting efficient relief and the interest of the forum

State/interstate judicial system(1) D must show a grave difficulty in litigating in the particular forum such that the burden

place on it puts it at a significant disadvantage (high threshold to change forum)(2) MI’s interest in adjudicating did not reach the level of Constitutional magnitude, nor did

the defendant’s inconvenience in the disputed) Dicta: “A defendant who has purposefully derived commercial benefit from his affiliations in a

forum may not defeat jurisdiction there simply because of his adversary’s greater net wealth.”i) Court was rejecting D’s claim that balance of harms in litigating in FL favored the wealthier

partye) However, this is a two-way street:

i) The D in BK could have sued BK in a MI court and gotten a MI jury/courtii) RPD: Why not attach a MI party to the suit to destroy diversity?

(1) RPD: But see cases above that prevent P from attaching client just to quash jurisdiction6) Purposeful actions directed by a defendant toward the forum State are required to establish minimum

connections (careful, only 4 justices supported this one). Asahi Metal Industry Co. v. Superior Court, 480 U.S. 102 (1987)

21

Ryan Preston Dahl; 917 687 7146

a) RELEASING YOUR PRODUCT INTO THE STREATM OF COMMERCE AND BEING AWARE OF WHERE IT MIGHT GO IS INSUFFICIENT TO SATISFY PERSONAL JURISDICTION

b) Asahi, a Japanese corporation, had supplied Cheng Shin, a Taiwan-based tire manufacturer with component parts. The tire became the subject of a products liability suit which Cheng lost. Cheng sought indemnity from Asahi. Asahi challenged the jurisdiction of the California court.

c) CA longarm statute was an “unenumerated” statute → it reached as far as Due Process would allow so statutory analysis defaults into a Constitutional analysis

d) Court held that Asahi’s generalized awareness that it’s products might end up in California was insufficient to establish “minimal contacts” despite having placed the products in the “stream of commerce”i) “Additional conduct of the defendant may indicate an intent or purpose to serve the market in

the forum state, for example, designing the product for the market in the forum State, advertising in the forum State, establishing channels for providing regular advice to customers in the forum State . . .”(1) RPD: What would the effect, say, of a 24-7 call center be?

ii) But “a defendant’s awareness that the stream of commerce may or will sweep the product into the forum State does not convert the mere act of placing the product into the stream into an act purposefully directed toward the forum state.”(1) There must be voluntary availment of the putative forum (2) Simple knowledge is not the same as control over the product as regards the product

iii) Even if you have control, you still need to satisfy the “fair play and substantial justice” requirement

iv) RPD: It is unclear whether this is actually the holding (as it is a plurality opinion)(1) Brennan and 3 other justices did not like the narrow stream of commerce argument

offered by O’Connor in the majority(2) “As long as a participant in this process is aware that the final product is being marketed

in the forum State, the possibility of a lawsuit there cannot come as a surprise.”(a) The stream of commerce is wide and swift for Brennan.(b) But this almost implies that you just shouldn’t ship parts anywhere(c) Note: This is different from the unilateral nature of things in World-Wide

Volkswagenv) Stevens, J. suggests that shipment of >100,000 units would constitute purposeful availment;

though b/c the case does not turn on this it really isn’t a great test (e.g., difference b/t 100,000 cars and 100,000 ball beargs)

e) Even apart from the “stream of commerce” issue, jurisdiction would violate due process. This is the determinative factor of Asahi. i) Burden on D is substantial → Asahi is Japanii) No compelling P interest → it’s just a claim for indemnificationiii) No compelling state interest → P is not a CA resident; it’s an indemnification suit b/t foreign

companies f) Generalized knowledge that a product maybe sold in the U.S. does not meet the requirements for

additional contacts. Parry v. Ernst Home Center Corp., 779 P.2d 659 (Utah, 1989)i) P was injured using a borrowed maul. Attempted to sue two Japanese manufacturer/suppliersii) Court held that despite the fact that Ds knew they could be sold in the U.S., their lack of any

agents, property, solicitations, representatives, brochures, or further service in Utah did not meet the Asahi of “further contacts”

iii) Note: Casebook does not seem to like this opinion (pp. 128-29)g) There is some disagreement about the applicability of Asahi’s personal availment test:

i) Rule that mere awareness that products will be carried to a particular market does not constitute purposeful availment supported in Lesnick v. Hollingsworth, 35 F.2d 939 (4th Cir. 1994); Boit v. Gar-Tec Products, Inc., 967 F.2d 671 (1st Cir. 1992)

ii) Cases following Brennan’s “stream of commerce test”: Dehmlow v. Austin Fireworks, 963 F.2d 941 (7th Cir. 1992)

22

Ryan Preston Dahl; 917 687 7146

iii) But see Irving v. Owens-Corning Fiberglass Corp., 864 F.2d 383 (5th Cir. 1989), rejecting both tests:(1) “[defendant] should have anticipated being haled into a Texas court given its contacts

with that forum.”(2) Casebook: This really isn’t a departure from Brennan’s test

7) How much or how little business does a company need to transact to establish minimum connections?a) A single contract is sufficient to establish personal jurisdiction over a non-resident defendant when

there has been substantial mail and telephone interaction between D and resident P. Alchemie International, Inc. v. Metal World, Inc., 523 F. Supp. 1039 (D.N.J. 1981)i) D, through numerous telephone and mail contacts, had solicited business from P, though it

had no offices nor conducted any other business in NJii) Court held that the Pennoyer requirements of “presence and power” were unnecessary in the

modern world and therefore “[regarded] defendant’s calls and mail communications to plaintiff as significant contacts”

b) But see,e.g., PCP-Recell, Inc. v. La Corona Foods, Inc., 912 F.2d 241 (8th Cir. 1990)i) P, a DL corporation based in MO attempted to use a MO longarm statute to assert jurisdiction

over D, an AZ-based supplier(1) “The defendant's contacts with the State of Missouri consisted of numerous telephone

and telefax orders to plaintiff's St. Louis office to purchase $1.7 million worth of goods, and defendant's payment by mail to a St. Louis lock box.”

ii) Court conducted a two-part inquiry:(1) Does personal jurisdiction exist under the long-arm statute?(2) Does personal jurisdiction violate due process?

iii) Court held that the contacts between the two were not substantial and that personal jurisdiction was not appropriate (it did not reach the due process issue)(1) RPD: Court dubiously claimed that its analysis satisfied Burger King.

8) Contacts and mass tort cases. a) Weinstein, J. in In re DES Cases, 789 F.Supp. 552 (E.D.N.Y.) tried to reconcile the Supreme

Court precedents in the mass tort contextb) Based on 2 principles:

i) Would a substantial number of citizens in forum state be affected by the resolution of the case?

ii) Prima facie case confers presumption constitutional jurisdiction absent showing of substantial hardship

c) Required evidence of hardship:i) D’s assetsii) Extent of D’s interstate commerceiii) D is represented by an indemnitor or going it aloneiv) Balance of hardships based on forums

General jurisdiction1) Specific jurisdiction focuses on the cause of action; general jurisdiction focuses on whether

“continuous and systematic contacts” exist. Dickinson Marine v. Palapatia, Inc., 179 F.3d 331, 339 (5th Cir. 1999)

2) General jurisdiction allows you to sue a D for any conduct, even for conduct unrelated to activities in that jurisdictiona) 6 ways to get it:

i) Lawsuit in D’s domicileii) Corporate defendant’s domicile (place of incorporation)iii) Consent (within limits, you can consent to be sued in a particular jurisdiction)iv) [Did not cover other 3 in class]

3) Knudsun, Keeton, Calder, Helicopteros, and Burger King – International Shoe’s Most Recent Progeny, 39 U. Miami L. Rev. 809 (1985)a) Burger King:

i) Key to the court’s decision was that all decision making power rested in Miami(1) Disputes had to be resolved with Miami(2) MI office was only a link b/t HQ and the franchisee

23

Ryan Preston Dahl; 917 687 7146

ii) Stevens’ dissent (with its emphasis on the disparity between contracting parties and the hardship suffered by D in litigating in FL) fails – like Helicopteros – “completely ignores the realities of the modern commercial world”(1) Physical presence is no longer necessary to conduct business(2) Major corporate decisions are still made at HQ despite existence of regional offices; the

relationship was between a FL franchisor and MI franchisee and the regional office was irrelevant

b) Helicopterosi) Court characterized the issue as one of general as opposed to specific jurisdictionii) Article notes that “Helicol's business dealings with Texas involved buying, selling,

negotiating, and training of personnel over a six- year period on a regular monthly basis, totaling some nine million dollars.”

iii) Brennan’s dissent correctly took issue with: (1) defendant had insufficient contacts to be subject to general jurisdiction; (2) the Rosenberg could be valid authority in 1984

(a) Rosenberg was decided under a pre-International Shoe standard (3) the issue before the Court was limited to a question of general jurisdiction only, rather

than involving both specific and general jurisdiction.iv) Helicopteros’ use of Perkins was wrong: “the Court seems to have confused the factual

situation in Perkins with doctrine”(1) Perkins relied on the International Shoe rule of “[T]here have been instances in which the

continuous corporate operations within a state were thought so substantial and of such a nature as to justify suit against it on causes of action arising from dealings entirely distinct from those activities.”

(2) Helicopteros, in effect, misapplies International Shoe as it was applied in Perkins:(a) Jurisdiction permitted if it does not offend “fair play”(b) Court should have determined if the dealings were so continuous and substantial as

to permit jurisdiction (the same analysis that the Perkins court appropriately made) not whether the cause of action arose out of Helico’s presence in TX(i) The millions in purchases should have met this standard

4) Despite the fact that a foreign corporation does not conduct those activities which might require it to designate an agent to be served, the Fourteenth Amendment does not preclude a State forum from asserting personal jurisdiction over a corporation which engages in substantial corporate activities in that State in an action for matters unrelated to those corporate activities. Perkins v. Benguet Consol. Min. Co., 342 U.S. 437 (1952)a) P, a non-resident of OH, sued D, a Philippines mining corporation for failure to pay due dividends

or issue stock. P sued in OH.i) During WWII and beyond, D’s President had conducted substantial corporate activities

(“consisting of directors' meetings, business correspondence, banking, stock transfers, payment of salaries, purchasing of machinery, etc”) in OH from his residence.

ii) These activities, however, did not subject the company to its longarm statuteb) Court held that these activities were substantial enough to warrant jurisdiction and did not work a

violation of the Fourteenth Amendment, even though the action was for acts unrelated to those activities under which in personam jurisdiction was established

c) Compare, Fisher Governor Co. v. Superior Court, 347 P.2d 1 (1959) (Traynor, J.)i) Wrongful death action. Decedent was killed in an explosion in Idaho. P served D, an IW

corporation by delivering papers to its CA manufacturing agent.ii) Court held that “although a foreign corporation may have sufficient contacts with a state to

justify an assumption of jurisdiction to enforce causes of action unrelated to its activities in that state . . . more contacts are required for the assumption of such extensive jurisdiction than sales and sales promotion.”

d) with Frummer v. Hilton Hotels Int’l Inc., 227 N.E.2d 851 (N.Y. 1967)i) NY tourist was injured when he fell in the shower of a London Hilton, a UK corporation. P

brought action in NY

24

Ryan Preston Dahl; 917 687 7146

ii) Court held that the interlocking ownership structure of D and Hilton Reservation Service (nominally a separate corporation) allowed the London Hilton to do business in NY as if it were London Hilton itself. Hence, there was jurisdiction

e) Ratliff v. Cooper Laboratories, Inc., 444 F.2d 745 (4th Cir. 1971): SC could not assert personal jurisdiction over Ds, 2 noncitizen drug makersi) “Activities of the defendant corporations in South Carolina, although possibly sufficient to

constitute 'presence' are nonetheless minimal. Neither maintains an office in South Carolina, and neither warehouses goods there. With the exception of the samples and materials used by Sterling's detail men, neither has any real or personal property in the state. Nor does either maintain a bank account in the state or advertise in directories there (although advertisements make there way here)”

ii) Court was also pissed off that P, a IN resident, “only interest in South Carolina is in its relatively long statute of limitations (six years), and plaintiffs' only contact with South Carolina is the bringing of these lawsuits for the sole purpose of availing themselves of that statute-- the limitation periods having run in all other states having any connection with the claims presented-- Florida, Indiana, New York, Connecticut, and Delaware.”

5) Purchases, training of personnel connected to those purchases, and accepting checks drawn on banks in the Forum State are insufficient grounds under the Due Process Clause on which to assert general in personam jurisdiction over a foreign corporation for a cause of action unrelated to those activities. Helicopteros Nacionales de Colombia, S.A. v. Hall, 466 U.S. 408 (1984)a) Helicopteros, a helicopter/transporter based in Peru, had contracted with the Peru-based alter ego

of a Texas corporation to provide its services to an operation in Peru. Contract was written in Spanish, formed in Peru, and agreed to be governed by Peruvian law. Helicopteros purchases choppers in TX and also trained its personnel there. Helicopteros did its banking in NYC.

b) This is not a contract dispute case so the clause restricting it to Peru doesn’t really applyc) TX long arm statute extended as far as 14th Amendment would allow but Court held requirements

of due process were not satisfied: “mere purchases, even if occurring at regular intervals, are not enough to warrant a State’s assertion of in personam jurisdiction over a nonresident corporation in a course of action not related to those purchase transactions.”i) Court applied the continuous and general test of Perkins

d) But see Brennan, J., dissenting. The contacts were of a nature to grant specific jurisdiction over this particular cause of action.i) Contacts with Texas were sufficiently related to the cause of actionii) Helicopteros had purposefully availed itself of the benefits and obligation of Texas through

“numerous and frequent” transactions (training, contracts, purchases with TX companies, etc.)(1) “[A] nonresident corporation [should be] amenable to suit in nay forum that is

significantly affected by the corporation’s commercial activities”iii) Opinion also misunderstands the distinction between contacts that are “related to” the cause of

action and those which “give rise to” the cause of action(1) “arise out of” is a direct test(2) “related to” is broader and should be sufficient to grant specific jurisdiction

(a) The opinion seems to imply that a court may not even assert specific jurisdiction under this broader test.

(b) Rather, the opinion requires that even specific jurisdiction requires that the cause of action “arise out of” the contacts – i.e., an element of the cause of action found in the nature of the contact with the state (e.g., the pilot’s training was negligent)

(c) This hypertechnicality is anathema to Brennan; he is trying to assert as broad a possible ground for specific jurisdiction

(d) However, Knudsun believes Brennan’s emphasis on this distinction is misguided (despite an otherwise good dissent) since the opinion actually uses the two terms interchangeably. Supra at 833-34.

6) Applicability of Helicopterosa) See de Reyes v. Marine Management & Consulting, Ltd., 586 So.2d 103 (La. 1991);

i) Principal defendant (a HK based corporation) had:(1) Maintained corporate office in forum state for 5 years, staffed with management level

employees

25

Ryan Preston Dahl; 917 687 7146

(2) Employees in forum state performed substantial servicesii) Court used International Shoe and specific jurisdiction cases to determine an assertion of

general jurisdictionb) Cresswell v. Walt Disney Prods., 677 F. Supp. 284 (M.D.Pa. 1987) Disney’s solicitation of PA

citizens to visit, broadcasting of the Walt Disney (a DL corp based in CA) channel in PA, recruiting of PA citizens for employment was “substantial and continuous” contact on which to properly base general jurisdiction for a personal injury claim arising from an accident at Walt Disney World Florida.

7) General jurisdiction may be obsolete. See, e.g., Nichols v. G.D. Searle & Co., 991 F.2d 1195 (4th Cir. 1993)a) Case arose from a suit by a group of more than 30 individual plaintiffs against a manufacturer of

IUD devices. Searle only connection with the forum state was that it employed some 13 people who were citizens of that state. None of the plaintiffs had been a resident of forum state.

b) Court has jurisdiction over a non-resident defendant if:i) An applicable state long-arm statute confers jurisdiction and ii) The assertion of that jurisdiction is consistent with constitutional due process.iii) “When, as in the case at bar, a suit does not arise out of the defendant's activities in the forum

state, the court must exercise "general jurisdiction" and the requisite "minimum contacts" between the defendant and the forum state are "fairly extensive."” Citing Ratliff

c) Court cited Ratliff, McGee, and others to note that while the domain of specific jurisdiction had expanded, “broad constructions of general jurisdiction should be generally disfavored.”

i) Court dismissed the claim for want of personal jurisdiction. Jurisdiction Based on Power Over Property1) Court has jurisdiction over personal legal obligations and property, such as unpaid debts, even if those

obligations or property did not arise in that jurisdiction if the court can assert personal jurisdiction over the individual in question. Harris v. Balk, 198 U.S. 215 (1905). a) Harris is essentially a quasi in rem jurisdiction per Shaffer.

i) RPD: It seems that the “in rem” part is that the legal obligation is – in a sense – a piece of property that Harris carried with him to Maryland

b) Case arose from an unpaid debt case.i) Harris, a citizen of NC, owed Balkii) Balk, a citizen of MD, owed Epstein, a citizen of Maryland.iii) When Harris was visiting MD, Epstein served him and garnished the debt Harris owed Balk

in MDiv) Harris attempted to defend his debt to Balk in NC under the Full Faith and Credit Clause.

c) Court held that the Maryland court had jurisdiction over the NC debt.i) Power over the person of the garnishee confers jurisdiction on the courts where the writ

ensuesii) The obligation of the debtor [Harris] was a personal obligation which could be enforced

anywhereiii) Therefore, the NC debt could be enforced/garnished in MD after personal service

(1) Court noted that the case would have turned out very differently had Harris not been properly served by Epstein per MD law

d) RPD: Pay attn’ to the element of garnishment in this case. Compare to Fuentes and its progeny regarding the Due Process requirements of garnishment

2) All assertions of state court jurisdiction (in personam, in rem, quasi-in-rem) must be evaluated according to the standards set forth in International Shoe. Shaffer v. Heitner, 433 U.S. 186 (1977)a) [Shaffer] held that a Delaware court could not assert jurisdiction over the defendants' shares of

stock in a Delaware corporation when the defendants did not have minimum contacts with the state and the cause of action was not related to any acts that had occurred in the state. Wright & Miller § 1072.

b) Shaffer places all suits against absent nonresidents on the same constitutional footing, regardless of whether a separate Latin label is attached to one particular basis of contract. (Shaffer per Burnham v. Superior Court)

26

Ryan Preston Dahl; 917 687 7146

c) Heitner, owning one share of stock, sued 21 executives of a DL corporation, all non-residents of DL. DL Court asserterted jurisdiction through the sequestration of stock listed on the books of the DL company, as quasi in rem jurisdiction.

d) Supreme Court ruled that the Pennoyer-type rules governing jurisdiction based on power over property needed to be modified by International Shoe, in addition to in personam jurisdictioni) The mere presence of property seemed to fly in the face of due process and Int’l Shoe-type

“fair play and substantial justice”(1) “If a direct assertion of personal jurisdiction over the defendant would violate the

Constitution, it would seem that an indirect assertion of that jurisdiction should be equally impermissible.”

ii) There would be no significant change in the ability of state courts to hear cases based on in rem jurisdiction(1) Property disputes being the prototype(2) The state’s interest in adjudicating/ensuring the fair marketability of property were

aligned as required by Int’l Shoeiii) Quasi in rem jurisdiction would change significantly

(1) Existence of property supports inference of substantial relation to forum state, but it is not conclusive

(2) Other requirements of due process must be satisfied.(3) The court is particularly concerned with property alone being used to assert jurisdiction

over matters completely unrelated to the property in question (as opposed to, say, a land dispute)

e) See Stevens, J. concurring.i) Purchaser of stock should not be open to suit in any state of the union. Stevens was

particularly upset with the DL statute which required stock in DL corps to be, legally speaking, in DL. The stock could then be sequestered for the purposes of establishing jurisdiction in DL.

ii) However, he did not like the idea of invalidating in rem jurisdiction where real estate is involved

f) Brennan, J. concurring and dissenting.i) Quasi in rem jurisdiction “is no longer Constitutionally viable”

(1) RPD: That quasi-in-rem jurisdiction is “no longer viable” seems to be an overstatementii) But court improperly ruled on minimum contacts issue

(1) Treated the minimum contracts issue as if it were a long-arm statute(2) The factual record was insufficient

iii) There were in fact minimum contacts to assert jurisdiction(1) State has substantial interest in allowing for restitution by corporations(2) Regulatory interest is a clear state interest(3) DL has a clear interest in overseeing an entity that is purely a creation of DL law(4) Choice-of-law problems created are substantial (5) Appellants are clearly “invoking the benefits and protections” of DL law

g) After Shaffer, DL amended its laws to create a longarm statute applicable to any nonresident director of a DL corporation, which was upheld by the DL supreme court

3) But see Rhoades v. Wright, 622 P.2d 343 (Utah 1980), cert. denied 454 U.S. 897 (1981)a) Court upheld attachment for jurisdictional purposes of D’s property (a Colorado resident) in Utah

who was sued for the wrongful death of a Utah resident killed in Utahb) Court distinguished Shaffer on the basis that real property and not intangible property was

involved and tha the Colorado defendant actively used the land to satisfy the due process requirements for quasi in rem jurisdiction

c) But see also Feder v. Turkish Airlines, 441 F. Supp. 1273 (S.D.N.Y. 1977)i) Court upheld quasi-in-rem jurisdiction in a post Shaffer caseii) Attached D’s bank account – used for the purchase of aircraft parts and components – which

was D’s only contact with forum state. Action was a wrongful death suit that arose from an accident that occurred in Turkey.

d) Attachment of an insurance policy to effect quasi in rem jurisdiction over a non-resident defendant is unconstitutional. Rush v. Savchuk, 444 U.S. 320 (1980)

27

Ryan Preston Dahl; 917 687 7146

i) Savchuck, a MN resident, brought suit in MN against Rush, an IN resident. Rush had no ties to MN sufficient to confer personal jurisdiction on the MN court.(1) Savchuck then attached State Farm to the lawsuit. State Farm was Rush’s insurer and

was contractually bound to indemnify Rush. Savchuck tried to garnish the policy. State Farm did business in MN on a continuous and systematic basis.(a) Argument was based on an interpretation of Harris v. Belk to claim the insurance

policy was a debt by State Farm owed to Rush, thus a form of property sufficient to permit quasi in rem jurisdiction over an absent D

(b) Thus, since the “debt follows the debtor” per Harris, the debt (which would thus be Rush’s property) would be found wherever State Farm was subject to personal jurisdiction and Rush could therefore properly be sued in MN under a theory of quasi in rem jurisdiction

(2) Court held that this exercise of jurisdiction was improper(a) The contractual obligation of State Farm, alone, was insufficient to confer

jurisdiction in the case(b) “The mere presence of property in a State does not establish a sufficient relationship

between the owner of the property and the State to support the exercise of jurisdiction over an unrelated cause of action”

(c) Despite the fact that the debt (the insurance policy) arose from the cause of action, the other requirements of International Shoe were not satisfied

ii) Court distinguished the contacts between the insured and the forum from the contacts between the Insurer and the forum

iii) Having done so, the Court held that sufficient contacts between D and the forum did not exist and that the Due Process Clause forbade jurisdiction

e) This killed a line of cases from Seider v. Roth, 216 N.E.2d 312 (N.Y. 1966): A contractual obligation that a constitutes a debt is subject to attachment for purposes of establishing jurisdiction. i) Plaintiff’s attached the obligation of an automobile insurance company to indemnify an out of

state policy holder, and jurisdiction was upheldii) Contractual obligation was a debtiii) Even after Shaffer, the Second Circuit noted that “the fall of Harris v. Balk does not

necessarily topple Seider.” O’Connor v. Lee-Hy Paving Corp., 579 F.2d 194 (2d Cir. 1978)(1) Court distinguished Shaffer and Harris as cases in which property that furnished the basis

for quasi-in-rem jurisdiction was totally unrelated to plaintiff’s claim, but that in O’Connor the attached property was an insurance policy that was purchased to protect against the type of liability that was the subject of the lawsuit

(2) RPD: To bad this all died in Rush v. Savchuk.f) But see Amoco Overseas Oil Co. v. Compagnie Nationale Algerienne de Navigation, 605 F.2d 648

(2d Cir. 1978)i) Court upheld jurisdiction based on attachment of a foreign corporations’ bank account in New

York in an Admiralty suitii) Distinguised Shaffer based on the admiralty context

(1) Constitutional power of Admiralty courts has its own Constitutional basis(2) Admiralty suits involve different policy concerns(3) Tradition suggests, too, that actors should/do reasonably expect to be sued where their

bank acc’ts are heldJurisdiction Based on Presence1) Assertion of personal jurisdiction over a physically present non-resident defendant from actions not

arising out of his contacts to the forum state does not require the International Shoe minimum contacts requirements and does not violate Due Process. Burnham v. Superior Court, 495 U.S. 604 (1990) (Scalia, J.)a) Case related to a divorce proceeding. Burnham, a NJ resident, was in CA visiting his children

when he was servedb) Court framed the question: Does due process require an International Shoe-type connection to the

litigation and the defendant’s connection to the forum state where defendant is physically present in the State at the time process is served upon him?

28

Ryan Preston Dahl; 917 687 7146

c) Court notes that petitioner’s claim that he can be “subjected to jurisdiction only as to matters that arise out of or relate to his contacts with the forum . . . . rests on a thorough misunderstanding” of the lawi) States traditionally had personal jurisdiction over anyone within their borders for any reason

(1) Scalia interprets “traditional notions of fair play and substantial justice” from a hard originalist standpoint

ii) Physical presence in the state satisfies the requirements of due processiii) Shaffer distinguished on the basis that it involved an absent defendant for whom the

“contacts” test had to serve as a proxy for actual presence (1) Shaffer places all suits against absent nonresidents on the same constitutional footing,

regardless of whether a separate Latin label is attached to one particular basis of contract.(2) Shaffer does not require that physically present defendants must be treated identically to

absent onesd) Dislikes Brennan’s test as being a break w/historical precedent and the result of the subjective

interpretations of judgesi) Also, a defendant who visits a state does so in the reasonable expectation that he might be

open to suit there. e) White, J. concurs on simply on the basis that personal service in the state in question is sufficient

to confer jurisdictionf) Brennan, J. concurs in judgment

i) The historical analysis adopted by Scalia is “foreclosed” by International Shoe; Shaffer also clearly states that historical notions of jurisdiction must conform to current beliefs regarding Due Process(1) Nor should Shaffer’s analysis be limited to quasi in rem jurisdiction(2) “Notwithstanding the nimble gymnastics of Justice Scalia’s opinion today, it is not

faithful to our decision in Shaffer.”ii) Defendant’s visit to California was sufficient to “enjoy the benefits” of the state sufficient to

confer jurisdictiong) RPD: This is another plurality opinion, so be careful how you use it.

2) Due Process does not prohibit a state from assessing personal jurisdiction over a mail-order company with neither outlets nor stores in the state if there is sufficient contract with in-state customers through the mail. Quill Corp. v. North Dakota, 504 U.S. 298 (1992)a) “To the extent that [prior] decisions have indicated that the Due Process Clause requires physical

presence in a State for the imposition of a duty to collect a use tax, we overrule those holding as superseded by developments in the law of due process.”

b) But the tax in question was held unconstitutional on the grounds that it unduly burdened interstate commerce

Jurisdiction Based on Consent1) Rule 12(h): Waiver or preservation of defenses

a) Defense of lack of jurisdiction over person, improper venue, insufficiency of process, or insufficiency of service is waivedi) If omitted from a motion under Rule 12(g); orii) It is neither made by motion or included in a responsive pleading under Rule 15(a)iii) Wright & Miller § 1391

(1) “Any time a defendant makes a pre-answer Rule 12 motion, he or she must include, on penalty of waiver, the defenses set forth in subdivisions (2) through (5) of Rule 12(b). If one or more of these defenses are omitted from the initial motion but were "then available" to the movant, they are permanently lost”

(2) But if a party did not make a pre-answer motion or a defense was not available to them at the time of the motion, then nothing has been waived. Id.

(3) If you go trial on the merits without objecting to personal jurisdiction, your ability to object to such jurisdiction is waived.

(4) Federal courts will consider a Rule 12(b) motion by a party in default as untimely and therefore as having been waived.

29

Ryan Preston Dahl; 917 687 7146

b) Defense of failure to state a claim upon which relief might be granted, failure to join a Rule 19 indispensible party and an objection of failure to state a legal defense to a claim may be made in any pleading permitted under Rule 7(a), by motion, by pleading, or at trial

c) Lack of subject matter jurisdiction may be made at any time. Rule 12(h)(3)i) Wright & Miller § 1393

(1) It is impossible to cure a defect in subject matter jurisdiction through consent of parties, time, or estoppel (in contrast to consent being sufficient to confer personal jurisdiction)

(2) You can even bring the defense in as a means of escaping final judgment(3) Court may bring the objection sua sponte.

d) Wright & Milleri) Rule 12(h) must be read with Rule 12(g) – i.e., to reduce number of preliminary motions

2) Challenging personal jurisdiction compels the defendant to submit to court orders related to the review of personal jurisdiction. Ins. Corp. of Ireland v. Compagnie des Bauxites de Guinee, 456 U.S. 694 (1982)a) D was a bauxite producer incorporated in DL but doing business entirely in Guinea. D attempted

to claim on a business interruption against a U.S. insurer and a consortium of foreign InsuranceCo’s in a PA district court. The foreign co’s made a special appearance to contest personal jurisdiction

b) D requested Discovery to establish it’s claim of jurisdiction, with which the InsuranceCo’s refused to comply. District Court sanctioned the Insurance Co

c) Supreme Court upheld the sanctionsi) Submitting to the jurisdiction of the court to adjudicate jurisdiction required the

InsuranceCo’s to comply with orders related to a finding regarding that jurisdictionii) Failure to comply with the Discovery order was tantamount to admitting the defense was

without merit3) Generally

a) In most states, a foreign corp. that registers as a condition of doing business in a state is regarded as having consented to suit in the courts of that state even as to actions unconnected with the corporations actions in that forum.

b) However, the extent of this is bound by due process. E.g., Ratliff v. Cooper Laboratories, Inc. c) A corporation that is not registered to do business in the forum state may still be subject to

jurisdiction if the corporation has represented that it is so registered and the corporation is estopped from claiming the jurisdictional defense. Dale-Steer-Inn v. Steer Inn Realty Corp., 274 N.Y.S.2d 379 (Sup. Ct. 1966)

4) Parties will often include in contracts clauses that require one or both to submit to personal jurisdiction in a particular venuea) M/S Bremen v. Zapata Off-Shore Co., 407 U.S. 1 (1972)

i) Respondent/Plaintiff-below Zapata had contracted with Bremen to tow oil drilling equipment. The contract contained a clause requiring actions to be tried in the U.K. After the tow was damaged and towed to Florida, Zapata commenced an action in the District Court.

ii) Supreme Court threw out the U.S. action on the basis of public policy:(1) “We cannot have trade and commerce in world markets and international waters

exclusively on our terms, governed by our laws and resolved in our courts.”(2) Court noted, however, that the forum clause could be invalidated if it was shown to be

“unreasonable under the circumstances”(3) Just having a contractual selection clause is not sufficient, per Burger King, you need to

also need to show that it’s just. iii) Cf. Stewart Org. v. Ricoh Corp., 487 U.S. 22 (1988)

(1) Action arose out of am contract dispute. Contract contained a clause requiring that any action tried in a court in New York/Manhattan. An action was ultimately brought in the Northern District for Alabama. Ricoh motioned to move the proceedings to the Southern District of New York under 28 U.S.C. § 1404(a)

(2) AL state law was hostile to such removal clauses. However, the Supreme Court held that the 28 U.S.C. § 1404(a) had greater weight than AL law in such an instance, and the motion to remove was therefore governed by analysis required by § 1404(a)

30

Ryan Preston Dahl; 917 687 7146

(3) However, the removal clause itself was not dispositive to the analysis required to see whether a motion under § 1404(a) was proper. Other factors include:(a) Balance of hardships(b) State interest(c) Systemic integrity(d) All under the statutory heading of “the interest of justice”

(4) Note: Most people refer to these as “1404(a) issues” from a practical p.o.v.b) Court upheld a forum selection clause between a corporation and an individual consumer in

Carnival Cruise Lines, Inc. v. Shute, 499 U.S. 585 (1991)i) P, a WA-resident, purchased a travel package from Carnival which required any litigation

arising from the action to be tried in Florida “to the exclusion of the Courts of any state or country.” P was injured on the boat in a slip-and-fall and attempted to bring the action in the Federal District Court for Washington

ii) Supreme Court did not undergo any “minimumcontacts” analysis and instead invalidated the WA-court’s jurisdiction on the basis of the forum provision in the contract(1) Cruise line was selling tickets to people all over the country(2) Consumers would benefit from the lower prices allowed by the forum shifting clause

(a) However, could argue that this in fact deters meritorious suits and therefore increases risks to passengers

iii) Limits on the enforceability of such provisions is “fundamental fairness”; though this is apparently not a difficult standard to get around since the Ps had no contact with FL and probably didn’t even see the clause and yet the clause was still upheld(1) Filip: There is substantial leeway for an actor to resist personal jurisdiction in a particular

forum through contractNotice1) Filip:

a) Notice is ½ of the analysis under the Due Process Clause w/respect to Personal Jurisdiction (the other half is “power” in the jurisdictional sense)

b) Sufficient notice must be provided to the other side that they are a party to the lawsuit, typically per Fed. R. Civ. P. 4.

c) Mullane, since it was articulated, is the standard of noticed) Notice analysis is a very common-sensical process and fact-specific (i.e., were the efforts to

provide notice reasonable under the circumstances?)i) A factor in Mullane, instance, was the distinction between the interests of beneficiaries and

contingent remaindermen; but at the end of the day their interests start looking rather similar2) Where the interested parties are known, notice as required by due process cannot be merely

perfunctory but must be reasonably conducted in such as way as to apprise the parties of the pendency of the action, the rights and obligations to be adjudicated, and allows the parties a reasonable amount of time with which to respond. This is an independent standard from the International Shoe requirements Mullane v. Central Hanover Bank & Trust Co., 339 U.S. 306 (1950) (Jackson, J.)a) Filip: This is jurisdiction by necessity caseb) Case related to a NY law which allowed for the pooling of small, individual trusts into a larger

trust which was to be managed by a single trustee. i) D, upon creating such a pool, sent out letters to the individual beneficiaries informing them of

the statute and the judicial pooling of the common trust fund accounts. The NY law essentially permitted a declaratory judgment on a triannual basis insulating the trustee from liability. (1) Note: Two trustees were appointed: one for the beneficial interests and one for the

contingent interests (since they wouldn’t necessarily line up)ii) Appellant appeared specially indicating that the statutory provisions which required

publication of notice in a NY newspaper and a letter to be mailed to all beneficiaries or potential beneficiaries

iii) A lower court ruling dismissed this objection, and foreclosed any future objections related to the common trust, including improper management or improper accounting during the period in question

31

Ryan Preston Dahl; 917 687 7146

c) NY State interest in this was substantiali) Interest of a state to regulate its trust is highii) Property is in the state; this is pure old-fashioned in rem jurisdiction

d) “An elementary and fundamental requirement of due process in any proceeding which is to be accorded finality is notice reasonably calculated, under all the circumstances, to apprise the interested parties of the pendency of the action and afford them an opportunity to present their objections. . . . The notice must be of such nature as reasonably to convey the required information. . . . But if with due regard for the practicalities and peculiarities of the case these conditions are reasonably met the constitutional requirements are satisfied.”

e) Process which is a mere gesture is not due processi) “It would be idle to pretend that publication alone as prescribed here, is a reliable means of

acquainting interested parties of the fact that their rights are before the courts.” Publication might be appropriate in the case of abandonment, missing persons.

ii) Personal service is the most iron-clad form of noticeiii) Jackson does not believe that the in rem-type distinctions matter here: the procedural

distinctions of in rem/in personam arose from pre-14th Amendement type systems which are irrelevant to the modern court:(1) “Distinctions between actions in rem and those in personam are ancient and originally

expressed in procedural terms what seems really to have been a distinction in the substantive law of property under a system quite unlike our own”

(2) Such distinctions are irrelevant to the uniform rule of Due Processiv) But Jackson is concerned w/finding a good balance of Notice with the very real benefits

provided by Trusts and the necessity of finding/having jurisdiction somewhere to make things work(1) Filip: Mullane is a good case to cite if you’re looking to get a sort of “jurisdiction by

necessity” → RPD: i.e., they had to try the case somewhere, so it might as well be NYf) Attachment of a chattel or real estate, together with publication, may provide adequate notice

(though Jackson seems unconcerned with the distinction of in personam/in rem/quasi in rem jurisdiction)i) But see Walker v. City of Hutchison, 352 U.S. 112 (1956): Notice in state condemnation

proceedings did not meet due process requirements when the only notice to the property owners was by publication in a local newspaper

ii) But see also Schroeder v. City of New York, 371 U.S. 208 (1962): Notice in state condemnation proceedings did not meet due process requirements when the only notice was publication coupled with signs posted on trees

iii) But see also Mennonite Board of Missions v. Adams, 462 U.S. 791 (1983): notice by publication and posting did not provide a mortgagee of real property with adequate notice of a proceeding to sell the mortgaged property for nonpayment of taxes.(1) “When the mortgagee is identified in a mortgage that is publicly recorded, constructive

notice by publication must be supplemented by notice mailed to the mortgagee’s last known available address, or by personal service. But unless the mortgagee is not reasonably identifiable, constructive notice alone does not satisfy the mandate of Mullane.”

g) Statutory notice is sufficient when it is clear that the persons in question are missing or unknowni) “Whatever standards of diligence, in view of the character and proceedings and the nature of

the interests here involved we think them unnecessary”ii) “We have no doubt that such impracticable and extended searches are not required in the

name of due process” especially given that such trusts were created for their economic efficiencies

h) However, where the beneficiaries names, locations, addresses are known, then the requirements of due process are not relaxed and they are entitled to a “serious accounting” on a regular basisi) Thus, “statutory notice to known beneficiaries is inadequate, not because in fact it fails to

reach everyone, but because under the circumstances it is not reasonably calculated to reach those who could be easily informed by other means at hand.”

32

Ryan Preston Dahl; 917 687 7146

ii) If one or 2 people don’t get the mailing, says Filip, Jackson has set up a system that still accomplishes the requirements of Due Process without undermining the efficacy of the trust by, say, requiring personal service to every single individual

i) Basic rule is that due process must be conducted in a matter reasonably calculated to succeed. 3) Cf., McDonald v. Mabee, 243 U.S. 90 (1917)

a) Case related to an action to enforce a promissory note made in Texas. The debtor had left Texas to set up domicile elsewhere. Service was attempted through publication in a paper once a week for four weeks. Mabee never appeared

b) Supreme Court held that service by publication could not warrant a judgment against a non-resident; “an advertisement in a local newspaper is not sufficient notice to bind a person who has left a state not intending to return.”

c) RPD: Case might come out differently post-Mullane. d) See also, Wutcher v. Pizzutti, 276 U.S. 13 (1928): Court invalidated a non-resident motorist statute

like the one at work in Hess on the basis that it failed to required the Secretary of State to communicate notice to the non-resident

4) What form of notice is sufficient?a) Tulsa Professional Collection Servs. Inc. v. Pope, 485 U.S. 478 (1988)

i) OK probate law barred any creditors’ claims against an estate unless presented to executor w/in two months of the publication of the publication of notice of the commencement proceedings.

ii) Court held that if a creditor’s identity was “reasonably ascertainable” by an executor, Due Process required that the creditor be given notice by mail or some other means so as to reasonably ensure actual notice

b) Greene v. Lindsey, 456 U.S. 444 (1982)i) KY statute provided, in forcible entry and detainer actions, for service of process by posting a

summons on the door of a tenant’s apartment. Plaintiffs challenged on the basis that the Mullane requirements were not satisfied, that they failed to see the notices, and that they did not learn of eviction proceedings until after the writs of possession were executed following default judgments

ii) Brennan, J. held that the notice given by such posting was insufficient to meet the requirements of Due Process (RPD: This appears to be a much higher standard than Mullane)(1) O’Connor, J. dissented(2) Noted it was absurd to put such a premium on service by mail as opposed to posting

notice on a doorwayc) Dobkin v. Chapman, 289 N.Y.S.2d 161 (1968)

i) NY court upheld court ordered notice when the defendants could not be foundii) Notice involved ordinary mail to Ds’ last known address and publication in a newspaperiii) Court emphasized that this was the best that could be expected under the circumstances and

any fault would lie on the Ds who failed to provide an address as required by lawd) Notice by mail of a proceeding to foreclose a lien for delinquent taxes on real property is

ordinarily sufficient to provide notice. However, such notice is insufficient if the person to be served is known to be insane, committed to a hospital, and without the protection of a guardian. Covey v. Town of Somers, 351 U.S. 141 (1956)

e) Actual notice is required when the defendant is incarcerated in Federal prison and the United States in the plaintiff. Weng v. United States, 137 F.3d 709 (2d Cir. 1998)i) But see, United States v. Clark, 84 F.3d 378 (10th Cir. 1996), notice mailed to the detention

facility in which the D was being held was sufficient notice(1) But see also Donovan v. United States, 172 F.3d 53 (7th Cir. 1999), requirement of actual

notice is too burdensome on the Government when the party is in State and not Federal Prison

ii) But see the intermediate position in United States v. One Toshiba Color Television, 213 F.3d 147 (3d Cir. 2000), government need only provide either ractual notice or a notice giving procedure reasonably calculated to provide notice

5) Content of the noticea) Aguchak v. Montgomery Ward Co., 520 P.2d 1352 (Alaska 1974)

33

Ryan Preston Dahl; 917 687 7146

i) D sold a snowmobile and freezer to P, which they took to where they lived. Ps allegedly did not pay, and Montgomery Ward sent a summons to which P did not respond and a default judgment.

ii) Summons failed to state that they could appear by a written pleading nor did it inform them that hey had a right to request a change of venue; Ps would have had to fly at a cost of $186 (on a case worth all of $988) with at least one night stopover

b) Alaska held tha the summons in small claims had to have at least this information and reversedc) See Wright and Miller §§ 1088, 1131 for a good form for how a summons ought to look

Mechanics of Giving Notice1) Depends on the jurisdiction2) Congress rejected an amendment to the Rules that would allow service via return receipt of certified

mail in 19823) Congress adopted a form in 1983:

a) First class mail with an acknowledgement to be filled out by the recipientb) If the acknowledgement not sent in, P had to find another way of making servicec) D had to reimburse costs of service if he could not show cause for the difficulty

4) Modified under Fed. R. Civ. P. 4(d)a) Action commences when P sends a form entitled “Notice of Lawsuit and Request of Waiver and

Service of Summons” by mail or “reliable” meansb) Domestic Ds have 30 days from date on which waiver was sent to return; otherwise charged with

the costs of servicec) Ds receives 60 from waiver was sent to answer if the waiver is returned in a timely fashiond) If P is confronting a statute of limitations deadline, statute continues to run until defendant is

servede) Rule 4(e) allows that state procedures may be used to effect service

5) Rule 4(f) covers service in foreign countries; designed to maximize the likelihood that judgments will be enforced abroada) Based on the Hague Service Conventionb) Under which contracting states see that a certification of service that has been effected is returned

to the court of originc) Volkswagenwerk Aktiengesellschaft v. Schlunk, 486 U.S. 694 (1988)

i) Hague convention only applies if the service is actually made abroad, otherwise the normal Rule 4 procedures apply

ii) And the Rule 4 change from “service by agreement” to “waiver of service” is inapplicable when the D takes advantage of waiver since Hague is only invoked by actual service(1) Note: Foreign Ds, too, are given incentives under Rule 4 to waive service

Venue1) Venue is entirely a creation of statute. However, it is waivable unlike subject-matter jurisdiction.

a) If you remove from state to Federal court, it goes into the Federal court covering that particular state geography

b) If you want to move to another Federal court from there, then it is a § 1404(a) issuec) Or if you want to move into a foreign country it becomes a forum non conveniens issue

2) 28 U.S.C. § 1391. Venue generally (Note: the majority rule is the “resident” is equivalent to “domicile” for the purposes of venue. Wright & Miller § 3805)a) § 1391(a): where jurisdiction is founded only by diversity citizenship you can only bring an action

ini) Judicial district where any D resides, if all the Ds reside in the same state; orii) District in which a substantial part of the events/omissions occurred or a substantial part of the

property in question exists; oriii) District in which any D is subject to personal jurisdiction at the time the action is commenced

b) § 1392(b): Action where jurisdiction is not founded solely on diversity may be brought only ini) District where any D resides if Ds all reside in same stateii) District where a substantial part of events or omissions arise or a substantial part of the

property in question is locatediii) A judicial district in which any defendant may be found, if there is not district in which the

action may otherwise be brought

34

Ryan Preston Dahl; 917 687 7146

c) § 1391(c): D that is a corporation deemed to reside in any district in which it is subject to personal jurisdiction at the time the action was commenced. i) If state has more than 1 district and in which D is a corporation and subject to personal

jurisdiction in that state, the corporation is deemed to reside in any district in that state “within which its contact would be sufficient to subject it to personal jurisdiction if that district were a separate state”

ii) i.e., you need sufficient contracts to allow for personal jurisdictiond) § 1391(d): alien may be sued in any district

i) This has nothing to do w/personal jurisdiction. i.e., the court in which the alien is sued must still have personal jurisdiction over the defendant in question. Wright & Miller § 3810.

e) § 1391(e): Where D is an officer/employee of the U.S., action may be brought wherei) D resides; or ii) District where a substantial part of events or omissions arise or a substantial part of the

property in question is located; oriii) If no property is involved, then where the P resides

f) § 1391(f): Civil action against a foreign state may be brought in i) District where a substantial part of events or omissions arise or a substantial part of the

property in question is located; orii) Any district in which vessel/cargo of a foreign state is situatediii) District where agency or instrumtentality is licensed to do business or is doing business if the

action is brought against that agency or instrumentality iv) In the D.C. district court if the action is brought against a foreign state or political subdivision

g) § 1391(g) Action under § 1369 may be brought in any district where D resides or in which a substantial part of the accident giving rise to the action took place

3) 28 U.S.C. § 1404 a) 28 U.S.C. § 1404(a): A district court may transfer a civil action to any other district or division “in

the interest of justice”i) RPD: the equities involved in such a determination are incredibly broad. Cf. Stewart Org. v.

Ricoh Corp., 487 U.S. 22 (1988), suprab) § 1404(b): Upon motion, consent or stipulation of all parties, any action, suit or proceeding of a

civil nature or any motion or hearing thereof, may be transferred, in the discretion of the court, from the division in which pending to any other division in the same district. Transfer of proceedings in rem brought by or on behalf of the United States may be transferred under this section without the consent of the United States where all other parties request transfer.

c) § 1404(c): A district court may order any civil action to be tried at any place within the division in which it is pending.

4) 28 U.S.C. § 1407. Multidistrict litigation.a) The judicial panel on multidistrict litigation may determine to transfer actions to any district for

“coordinated or consolidated pretrial proceedings.” Panel has the power to separate any claim, cross-claims, etc. and remand such claims b/f it deals w/the remainder of the action

b) A judge may be designated to oversee the litigation in the transferee district by the Chief Justice of the United States

c) Proceedings to transfer an action under § 1407 may be initiated byi) Judicial panel on multidistrict litigation, orii) Motion filed with the panel by a party in any action in which such a coordination might be

appropriated) Panel composed of seven circuit and district judges designated from “time to time” by the Chief

Justice. Simple majority necessary to movee) No proceedings for review are permitted except by extraordinary writs authorized under 28 U.S.C.

§ 1651 (RPD: I think mandamus is the exemplar here)f) Panel may prescribe rules not inconsistent with the Rulesg) § 1407 does not apply to anti-trust litigation in which the U.S. is a partyh) § 1407 does not apply to actions brought by U.S. under 15 U.S.C. § 15c (Actions by State

attorneys general)

35

Ryan Preston Dahl; 917 687 7146

i) 28 U.S.C. § 1407 involves pre-trial preparation only. After the preparation the cases must be remanded to their original courts for trial. Lexecon, Inc. v. Milberg Weiss Bershad Hunes & Lerach, 523 U.S. 26 (1998).

5) 28 U.S.C. § 1441. Actions removable generally.a) § 1441(a), unless expressly provided, any civil action in a state court of which the Federal district

court has original jurisdiction may be removed by the defendant or defendants to the district court. Citizenship of Ds sued under fictitious names is disregarded for a removal under § 1441(a)

b) Any civil action over which the district courts have original jurisdiction founded on a claim arising under the Constitution, treaties, or Federal law are removable without regard for citizenship or residencei) Any other action shall be removable only if none of the parties in interest properly joined and

served as Ds is a citizen of the State in which the action is broughtc) If separate or independent claims under § 1331 (federal question) are joined with non-removable

claims, the entire case may be removed to federal court at the discretion of the court, or may remanded the state matters or all matters to state court at its discretion

d) Any civil action brought in a state court against a foreign state may be removed by the foreign state to the district court where the action is pending

6) Venue generallya) Venue is the place of trial in an action within a state; venue is a creature of statuteb) Exceptions are generally made for special genii of cases such as divorce, probate, trust law, etc.c) Other factors matter in the statutory creation of venue, e.g.,

i) Where the action arose;ii) Where some fact happened, iii) D’s residence or where D is doing business, has office, agent, officer, etc.iv) P’s residence, where P is doing businessv) Where D may be summoned, served, etc.

7) A state venue provision that permits a resident corporation only to be sued in its home county but permits plaintiffs to bring suit against non-resident corporations in any county does not work a violation of the Due Process Clause. Burlington Northern R..R. Co. v. Ford, 504 U.S. 648 (1992). a) Montana permitted a MT-incorporated corporation to be sued only in the county in which the

corporation had its principle place of business. Any other corporation could be sues in any county in Montana.

b) Court held that a State would in fact be in its discretion to allow plaintiffs to sue a corporation anywhere in Montana, and such a provision that balanced the D’s interests somewhat did not work a violation of the 14th Amendmenti) “Montana could thus have decided that a nonresident defendant’s interest in convenience is

too slight to outweigh the plaintiff’s interest in suing in the forum of his choice.”8) 28 U.S.C. § 1404(a) allows transfer only to those districts in which the plaintiff could have originally

brought the action; D’s move to transfer cannot override a plaintiff’s objection to the move. Hoffman v. Blaski, 363 U.S. 335 (1960). a) Ds in a patent infringement case tried to remove a case to the Ps home district.b) Court rejected the move; “the power . . . under § 1404(a) to transfer an action to another district is

made to depend not upon the wish or waiver of the defendant but, rather, upon whether the transferee district was one in which the action ‘might have been brought’ by the plaintiff.i) Court distinguished this case from one in which the D agreed with the P to be sued in a

particular jurisdictionii) “It would constitute gross discrimination to permit transfer to be made with the defendant’

consent and over the plaintiff’s objection to a district to which the plaintiff could not similarly obtain transfer over the defendant’s objection.”

c) RPD: P could not have brought the action in his home district per § 1392. d) Frankfurter, dissenting, thought that the fact that the D moved to transfer should have been

sufficient since the language of § 1404(a) was rooted in the interest of justice and not proceduree) Filip: The Hoffman rule is generally criticized

i) It doesn’t make sense to have a case “boxed in” to a particular venue if the Defendant is willing to have it litigated somewhere else

36

Ryan Preston Dahl; 917 687 7146

9) Reasonable probability of service in the transferee district is sufficient grounds at the time at which the action was commenced is sufficient to warrant a transfer. Dill v. Scuka, 198 F. Supp. 808 (D.C. Pa. 1961)a) Involved a tort claim by P, a KS citizen, against D, a doctor from KS who was doing some grad

work in Philadelphia. b) PA district court permitted transfer to KS on the basis that

i) The partial paralysis of P clearly made it a matter of convenience under § 1404(a)ii) Distinguished from Hoffman v. Blaski, supra, where there was no possibility of making

service on defendant. (1) D had many contacts with the transferee district over a period of fifteen years, except for

five years of military service during World War II. (2) D had no permanent intention to remain in Pennsylvania. His wife's family lived in the

transferee district. (3) “There was a reasonable probability on this record that he could have been served in the

transferee district within a reasonable time after institution of this suit on April 11, 1956.” (emphasis added)

c) A transfer may also be upheld if the action could have been brought as a permissive counterclaim in an action already underway between the parties. A.J. Indus., Inc. v. United State District Court, 503 F.2d 384 (9th Cir. 1974)

10) When you have a diversity case, the law applicable in the transferor state follows the transfer, including the choice of law rules. Van Dusen v. Barrack, 376 U.S. 612 (1964)a) Filip: A different rule applies in Federal question litigationb) However, Van Dusen is sometimes held not to apply to Federal claims.

i) See, e.g., In re Korean Airlines Disaster, 829 F.2d 1171 (D.C. Cir. 1987): the law of transferor forum on federal questions merits close consideration but does not have precedential effect in a transferee forum situated in anther circuit.

11) Plaintiff moving to transfer venuea) See Ferens v. John Deere Co., 494 U.S. 516 (1990)

i) P, a PA resident, brought a tort claim against D in MS where the statute of limitations had not yet run. P then moved to transfer the case to PA where he had contract and warranty claims against D pending (which were not time-barred), where the MS law should still apply to the tort claim

ii) Supreme Court upheld the move and the applicability of MS law on the basis of Van Duseniii) Filip: This is a rather crazy application of Van Dusen

12) Mechanics of the movea) Burden is on the movant in § 1404(a) to show that the alternative venue is better/more efficient.

This is also true of forum non conveniensb) The balance of hardship must clearly favor the transferc) Factors are defined by Gilbert

13) A district court, finding it lacks personal jurisdiction, may properly transfer the action to the relevant district court under 28 U.S.C. § 1406 in order to prevent the injustice of a P who has “erroneously guessed” at the proper forum from being barred. Goldlawr, Inc. v. Heiman, 369 U.S. 463 (1962)a) P had brought suit in E.D. Pa., which found itself without personal jurisdiction and transferred to

S.D.N.Y., which then dismissed on the basis that E.D. Pa. had no jurisdiction to effect the transfer.b) Court upheld the transfer: “The language of § 1406(a) is amply broad enough to authorize the

transfer of cases, however wrong the plaintiff may have been.”c) However, Harlan dissented (“the notion that a District Court may deal with an in personam action

in such a way as possible to affect a defendant’s substantive rights without first acquiring jurisdiction over him is not a familiar one in federal jurisprudence.”) and Goldlawr might not be good law

Forum non conveniens1) Definition

a) The doctrine that an appropriate forum -- even though competent under the law -- may divest itself of jurisdiction if, for the convenience of the litigants and the witnesses, it appears that the action should proceed in another forum in which the action might also have been properly brought in the first place.

37

Ryan Preston Dahl; 917 687 7146

b) “Forum non conveniens allows a court to exercise its discretion to avoid the oppression or vexation that might result from automatically honoring plaintiff's forum choice. However, dismissal on the basis of forum non conveniens also requires that there be an alternative forum in which the suit can be prosecuted.”

2) The court may, at its discretion, invoke forum non conveniens even when jurisdiction is authorized by the letter of a general statute. Gulf Oil Corp. v. Gilbert, 330 U.S. 501 (1947)a) Factors to be considered are:

i) Access to sources of proofii) Availability of compulsory process for attendance of unwillingiii) Cost of obtaining attendance of willing witnessesiv) Possibilty of view of premisiesv) All other practical problems that make a case ease, expeditious and inexpensivevi) Factors of public interest, “there is a local interest in having localized controversies decided at

home.”3) The prospect of change of substantive law is neither conclusive nor significant factor in an analysis

under a motion of forum non conveniens. Piper Aircraft Co. v. Reyno, 454 U.S. 235 (1981) (Marshall, J.)a) This is the seminal forum non conveniens caseb) Case related to a plane crash in Scotland. American executor of the decedents – who were all

Scottish – attempted to bring action in the U.S. against the manufacturer b/c U.S. laws were more favorable to such tort claims. The case was removed to Federal court

c) Whereas the Circuit Court had denied dismissal on the basis that since Scottish law would prejudice the P’s claim (it did not recognize strict products liability), Supreme Court upheld the dismissal on the basis of forum non conveniensi) Initial state of a forum non conveniens inquiry is existence of another forum, which is

satisfied if D is amenable to process in the alternative jurisdictionii) “If conclusive or substantial weight were given to the possibility of a change in law, the forum

non conveniens doctrine would become virtually useless.”iii) The doctrine is designed to avoid conducting comparative law exercisesiv) “We do not hold that the possibility of an unfavorable change in law should never be relevant

consideration in a forum non conveniens inquiry . . . if the remedy provided by the alternative forum is so clearly inadequate or unsatisfactory that it is no remedy at all, the unfavorable change in law may be given substantial weight.”(1) Filip: This is really your best way to survive a forum non conveniens(2) e.g., you’re in a wacky foreign court that is just going to hammer you

d) All the other factors in forum non conveniens analysis required by Gilbert clearly favored dismissal since Scotland’s interest was obviously more compelling

e) Marshall was clearly concerned that the whole world would start litigating in U.S. courts4) Forum non conveniens does not relieve the court of its obligation to decide whether it had jurisdiction

to hear the lawsuit. De Cedeno v. Arosa Mercantile, S.A., 398 N.Y.S. 250 (Sup. Ct. 1977)5) Inconvenience to D may be less than that required to invoke Due Process protections may be sufficient

to justify granting a motion to dismiss under forum non conveniens. a) However, a more convenient forum must first exist. (See Reyno, supra). b) Courts will generally base their willingness to grant motions to dismiss for forum forum non

conveniens on such stipulations by D as:i) Waiver of objection to personal jurisdictionii) Waiver of statute of limitations defense, etc.

c) Thus, a court may lack the ability to transfer a suit to another state ,but it may use its ability to shape the forum non conveniens dismissal to allow such a result regardless. See, e.g., In re Union Carbide Corp. Gas Plant Disaster, 809 F.2d 195 (2d Cir. 1987)

d) However, forum non conveniens motions are granted infrequently at best6) While existence of an alternate forum is “most important” to a forum non conveniens analysis, it is not

dispositive and a case may be properly dismissed under forum non conveniens even without such an alternative. Islamic Rep. of Iran v. Pahlavi, 467 N.E.2d 245 (N.Y. 1984)a) Iran sued the Shah in NY courts for allegedly misappropriated funds. b) Court threw out the case on the basis of forum non conveniens.

38

Ryan Preston Dahl; 917 687 7146

i) No forum state interest though burdens would be tremendousii) Court backhanded the Gilbert-language that required an alternate forum

7) Note: in addition to forum non conveniens decrees, a party may seek an injunction to restrain parties from proceeding an another court, but they are often found to be unenforceable

8) Difference b/t 1404(a) and forum non conveniens:a) If a forum non conveniens motion is granted the case is dismissed

i) Hence, courts will often grant contingent on the agreement of parties not to, say, press a statute of limitations defense in the other countries

b) Under 1404(a) the case is just transferredc) Gilbert factors are applied in both forum non conveniens and in § 1404(a)d) You’re better off trying to get a case moved on 1404(a) / forum non conveniens ground then you

are contesting personal jurisdiction from a practical standpoint

Governing Law and Erie issues Before Erie1) Generally, revolves around the question of what substantive law should be applied when you have

litigants from diverse states. a) Originally, the district courts were obliged to use the laws of the several states” unless there was a

Constitutional or Federal law question. 28 U.S.C. § 1652.i) This was interpreted in Swift v. Tyson, 41 U.S. 1 (1842) to hold that this required the courts to

use state law only if it was in a state statute, in the absence of the statute, the common law interpretation employed by the federal court applied

ii) “It has never been supposed by us that [28 U.S.C. § 1652] didapply, or was designed to apply, to questions of a more general nature, not at all dependent upon local statute or local usages of a fixed and permanent operation”

b) The rule of Swift owes a lot to the fact it appeared in the “heyday” of the common law”2) Black & White Taxicab and Transfer Co. v. Brown & Yellow Taxicab and Transfer Co., 276 U.S. 518

(1928)a) P, a KY corporation, incorporated in TN for the sole purpose of establishing diversity of

citizenship and evade Kentucky common law which they believed was not in sync w/more general practice. P subsequently sued D to enjoin them from infringing upon P’s exclusive transfer contract with a KY railroad company. D claimed that diversity jurisdiction did not exist since the TN incorporation was a mere sham.i) KY courts held such exclusive (anti-competitive) rights contract as invalid whereas Federal

courts had upheld themii) Note: Brown was plaintiff below, Black was defendant below

b) Court upheld the jurisdiction: “The motives which induced the creation of respondent to become successor to its Kentucky grantor . . . have no influence on the validity of the transactions which are the subject of the suit.”

c) Holmes, J., dissentingi) The Federal court’s ability to hold forth on common law derived from the fallacy that “the

parties are entitled to an independent judgment on matters of general law.”ii) Yet this presupposes a common body of “Law” which is simply nonexistent

(1) “The common law so far as it is enforced in a State, whether called common law or not, is not he common law generally but the law of that State existing by the authority of that State without regard to what it may have been in England or anywhere else.”

(2) Therefore, “I do not perceive how it would be possible for a Court of the United States to refuse to follow that the State Court decided in that domain.”

iii) Holmes acknowledges that Swift misunderstands the Framer’s intention, but that isn’t even the real point; “it is a question of authority by which certain particular acts . . . are governed.”

Erie1) Two basic choice of law questions:

a) Horizontal choice of law: What State’s laws will apply in a given suit?i) E.g., a lawsuit in a TX regarding construction of a machine in ID.

b) Vertical [revisit me]2) 28 U.S.C. § 1652: State laws as rules of decision

39

Ryan Preston Dahl; 917 687 7146

a) “The laws of the several states, except where the Constitution or treaties of the Unites States or Acts of Congress otherwise require or provide, shall be regarded as rules of decision in civil actions in the courts of the United States incases where they apply.”i) Filip: this is a bit vague. It can be read to say “federal laws do not apply except where they

apply.”ii) Swift was the seminal case interpreting the Rules of Decision for the first century

(1) Swift involved what is known as a bona fide holder in due course of a negotiable interest; and did forgiveness of an existing debt constitute valuable consideration?

(2) In Swift, did the Federal Court have to apply the fairly well-articulated NY rule? Or, instead, can the court apply general common law?

(3) Federal courts need not apply state court decisions and substantive rules in diversity cases. (a) Lurking in the background in Swift is the jurisprudential notion that courts do/did not

make law; they discovered it in a senseiii) General Common Law, through Swift, is not binding on states b/c of the Supremacy Clause of

the Constitution; the argument is that the rule is in fact better (i.e., a persuasive argument and not a Constitutional argument)

b) RPD: So state law governs civil actions unless you have a Federal Question3) U.S. Const. art. III

a) Section 1i) Creates the Supreme Court and “such inferior Courts as Congress my . . . ordain and

establish.”ii) Judges shall hold office and compensation cannot be reduced during their time in office

b) Section 2: i) creates Federal jurisdiction in

(1) Constitutional and Federal law(2) Cases involving ambassadors, ministers, and public counsels(3) Admiralty/maritime cases(4) Cases in which U.S. is a party(5) Cases b/t two or mores states(6) Between a state and citizen of a different state(7) Citizens of different states (diversity)(8) Between states/its citizens and foreign subjects or nations

ii) Trial of all crimes – except Impeachment – is by jury4) U.S. Const. amend. X.

a) “The powers not delegated to the United States by the Constitution, nor prohibited by it to the States, are reserved to the States respectively, or to the people.”

5) State law governs an action except in questions of Federal law or Constitutional law. Erie R. Co. v. Tompkins, 304 U.S. 64 (1938) (Brandeis, J.)a) Case was a diversity suit based on a tort claim in PA. PA held a RR owed a lower standard of care

to a person walking by a RR (only to prevent wanton disregard) then other states (ordinary care). P wanted a Fedeal court to apply the ‘general law’ as implied by Swift.

b) Court held that Swift was inherently flawed i) Prevented uniformity in the application of law in a particular state (since Federal courts would

use one standard and state courts would use another)ii) Was open to abuse, since parties could switch their domicile just to get into Federal court and

get a “better” rule(1) RPD: Why can’t 28 U.S.C. § 1359 take care of this problem?

c) Swift was also unconstitutionali) Neither Congress nor the judiciary has any power to make laws for the States except for

matters specifically authorized by the Constitution.(1) RPD: Cf. U.S. Const. amend. X.

ii) Holmes (dissenting in Kuhn):(1) Swift is predicated on the notion that there is some “transcendental body” of law beyond

the particular law of the State – i.e., an objective Law – that the Federal court may choose to apply

40

Ryan Preston Dahl; 917 687 7146

(2) This is incorrect: law is derived from “some definite authority” by which Holmes means political authority

d) Reed, J., concurring: it is enough to say that Swift is flawed and should be reversed. You don’t need to go so far as to call in Unconstitutional. (RPD: What’s the difference?)i) No one doubts for the Federal Rules to make procedural rules

6) Summary of Erie’s rationalea) Swift misunderstands the nature of the Rules of Decision Act; based on a fundamental

misunderstanding of lawb) Policy considerationsc) Constitutional concerns; however the allusion to equal protection concerns is more of a “fig leaf”

to get to where the court wants to go7) Results of Erie

a) Courts in diversity cases must apply the substantive law of the statei) Filip: Easy to say, but this is the fundamental rule

b) Procedural rules are another matterc) Hence the analytic dichotomy b/t substantive rules and procedural rules

i) Erie itself declares that the Federal gov’t has no power to declare substantive rules of common law over the States

ii) Hence, the procedural rules of the forum will almost always controlEvolution of Erie1) In suits solely predicated upon diversity claims, State law applies in all matters, procedural and

substantive, if it would significantly alter the result of a litigation for a federal court to disregard a law of the State. Guaranty Trust Co. v. York, 326 U.S. 99 (1945) (Frankfurter, J.)a) Guaranty Trust was a trustee for certain creditors of the Van Swearingen Corp. Guaranty also

loaned money to Van Swearingen affiliates.i) Guaranty worked out a recapitalization when Van Swearingen started having financial

troubles. ii) York, who purchased some of Van’s debt from a creditor who refused to participate in the

recap, sued Guaranty for breach of fiduciary duty.iii) Question was whether a Federal court was required to apply a state statute of limitations in a

diversity suit brought in equity. b) Court held that the rule of Erie applied whether the relevant issue was a matter of “substance” or

procedure”i) “A federal court adjudicating a State-created right solely because of the diversity of

citizenship of the parties is for that purpose, in effect, only another court of the State, it cannot afford recovery if the right to recover is made unavailable by the state nor can it substantially affect the enforcement of the right as given by the State.”

ii) The question to ask, then, is not whether the rule is one of procedure or substance, but “does [the rule or law] significantly affect the result of a litigation for a federal court to disregard a law of a State that would be controlling in an action upon the same claim by the same parties in a State court?”(1) RPD: The goal is to get similarly situated diversity cases to come out the same way

regardless of whether they are brought in State or Federal courtiii) In permitting diversity suits, “Congress afforded out-of-State litigants another tribunal, not

another body of law.”c) However, the Federal court is allowed to use remedies not available in a State court since these are

not a matter of creating or denying rights which are a function of State lawi) “State law cannot define the remedies which a federal court must give simply because a

federal court in diversity jurisdiction is available as an alternative tribunal to the State’s courts.”

ii) Federal courts may use whatever equitable remedies are consonant with the practices of Equity so long as the Federal courts were enforcing rights created by the Sttates “and not arising under any inherent or statutory federal law.”

d) Glannon: York should be viewed as an expansion of the Constitutional doctrine articulated in Erie to a desire for uniform outcomes in the state and federal systems

41

Ryan Preston Dahl; 917 687 7146

e) “York . . . spelled death to the hope for a completely uniform federal procedure. When its doctrine is logically applied, each important step in a diversity action must be examined in the light of two systems of law – first, under the Federal Rules, and then under the law of the state in which the federal court sits.” Merrigan, Erie to York to Ragan, 3 Vand. L. Rev. 711, 717 (1950)

2) Federal Rules of Civil Procedure v. state procedurea) State law and not the Federal Rules govern the tolling of statutes of limitations in diversity suits.

Ragan v. Merchants Transfer & Warehouse, 337 U.S. 530 (1949)i) Case arose out of a tort claim in KS. P filed an action within the 2-year statute of limitations

defined by KS law. However, notice was not served until after the 2-year statute had tolled.(1) Fed. R. Civ. P. 3 holds that an action commences with its filing(2) KS, however, required that the statute did not toll until notice was served

ii) Court held that Fed. R. Civ. P. 3 did not apply to statutes of limitations, and the State law was applicable

b) In diversity suit for a shareholder derivative suit governed by NJ law, P was required to post bond as required by NJ law even though Fed. R. Civ. P. 23.1 does not require such a bond. Cohen v. Beneficial Indus. Loan Corp., 337 U.S. 541 (1949)i) Court held that since the action created substantive liabilities for expenses, it did not matter

whether the NJ statute was classified as “substantive” or “procedural”c) A corporation that cannot enter the state court in which the resident defendant is based cannot

maintain a diversity suit in the federal courts of that state. Woods v. Interstate Realty Co., 337 U.S. 535 (1949)i) A TN corporation was not qualified to do business in MS and was therefore precluded from

entering MS courts.ii) Court held that since the TN corporation could not enter a MS court, it could not enter a

federal court in MS to bring a diversity suit3) The possibility of a different result cannot compel the use of state law – not bound up with rights and

obligations – if such an application it would substantially alter the nature of the Federal judicial system. Byrd v. Blue Ridge Rural Elec. Coop., Inc., 356 U.S. 525 (1958)a) Diversity case b/t a NC resident employed by a SC company. P was working for a contractor

employed by D. P was injured. Under SC law, if a contractor is employed in a job also performed by the EmployerCo, he is obliged to go into the state Worker’s Comp system as the exclusive remedy for his injuries. i) P was injured and sued.ii) D defended on the basis of the worker’s comp statute as an affirmative defense. iii) Under SC caselaw, such immunity from prosecution was a matter of law to be determined by

the judge and not the juryb) Court held that the substantial 7th Amendment requirement of putting such a factual issues (such

as immunity) to a jury rather than a judge warranted putting aside state law in this case in addition to the general “housekeeping” administration/procedural concerns of the federal systemi) “The policy of uniform enforcement of state-created rights and obligations . . . cannot in every

case exact compliance with a State rule – not bound up with rights and obligations – which disrupts the federal system of allocating functions between judge and jury.”(1) Note: Brennan says it is the “influence – if not the command – of the Seventh

Amendment” which warrants this result(2) See also Simler v. Conner, 372 U.S. 221 (1963): the jury-trial right is to be determined by

federal law and not state law in diversity cases in order to maintain the uniformity required by the Seventh Amendment.

(3) RPD: Note how this rule restrict the Federal court’s ability to not use state law only if that law is not “bound up with rights and obligations – but how do you draw the line?

ii) “state statutes and constitutional provisions [can] not disrupt or alter the essential character or function of a federal court.”

iii) Hence, York might be sufficient, but a York analysis must be balanced with:(1) Desire for Federal procedural uniformity(2) Seventh Amendment requirements, broadly understood

iv) Court noted in dicta that it was not certain that a different result would in fact occur

42

Ryan Preston Dahl; 917 687 7146

c) Casebook/Filip: Byrd is advocating a balancing test b/t state and federal concerns in determining Erie questionsi) See, e.g., Allstate Ins. Co. v. Charneski, 286 F.2d 238 (7th Cir. 1960)

(1) Case arose from a InsuranceCo seeking declaratory judgment in a WI court as allowed by WI law. The WI Supreme Court dismissed the action on the basis that such a judgment would conflict with WI-state policy

(2) Seventh Circuit affirmed the WI ruling:(a) WI interest in the case was clearly significant (as the WI Supreme court noted)(b) Federal interest was slight; the Federal Declaratory Judgment acts did not implicate a

Constitutional right as was at play in Byrd(i) “The case of action arising from the accident, the issue of coverage of the policy

. . . are intimately connected with Wisconsin law and have no connection with the federal government except that the alter provides a fair and orderly forum in which to try the diversity case.”

ii) Bernhardt v. Polygraphic Co. of Am., Inc., 350 U.S. 198 (1956)(1) Court upheld the application of VT law which allowed a party to invalidate a arbitration

clause at any time prior to the award in a diversity suit in federal court(2) “If the federal court allows arbitration where the state court would disallow it, the

outcome of litigation might depend on the courthouse where suit is brought.”(a) Court was particularly concerned with the loss of Seventh Amendment rights that

might/could ensue in arbitration(b) RPD: Unclear how Bernhardt would come out after Byrd

iii) Filip: Byrd gets more mileage in law school than in practice. Hanna is really the leading case insofar as Erie questions are concerned

4) Do you use a federal standard or a state standard to determine the existence or nonexistence of in personam jurisdiction over a foreign corporation?a) Majority rule is state standard. See, e.g., Arrowsmith v. United Press Int’l, 320 F.2d 219 (2d Cir.

1963)b) See also Wright & Miller § 4510

5) Erie does not require an “outcome determination analysis” but an examination into the avoidance of inequitable decisions and forum shopping. In the absence of either, a federal rule controls a state even if the two are in “direct collision” and they possess the same scope. Hanna v. Plumer, 380 U.S. 460 (1965)a) Diversity tort claim in MA. Respondent was an executor, who claimed that service under the

Federal rules did not conform to the service as required by Mass. state law which required personal service on an executor within 1-year of the executor assuming that position. Applying Mass. state law, respondent would immediately win (apparently b/c the statute of limitations on the suit would have run after the Supreme Court ruling)i) D had attempted to argue that Ragan controlled

b) Court held that Rule 4 and the MA statute were in direct collision, and that Rule 4 therefore prevailed

c) Hanna-test to asses conflicts b/t State law and the Rules (per Burlington N’thn v. Woods)i) Is the rule, fairly construed, sufficiently broad to create a “direct collision”?ii) Does it then leave no room for operation of the state law?iii) If Yes, then the Rule applies

d) Court held that the Erie requirement of the application of state law is not merely based on an “outcome determination” (i.e., would the outcome change if you used different law), but in regards to Erie’s principle concerns regarding:i) Forum shopping; and ii) “Avoidance of inequitable administration of the laws”

e) There is no forum shopping / substantive rights concerns here:i) P was not faced with a choice of laws (that might/might not bar recovery) at the time of

initiating the suit, only the manner of serving process(1) RPD: apparently the statute only tolled after the federal suit was ongoing and not before

as in Ragan?

43

Ryan Preston Dahl; 917 687 7146

ii) Moreover, service as prescribed by the Federal Rules does not alter state created rights in any substantial way such as to raise Erie concerns

f) When a situation is clearly covered by one of the Federal Rules, the court must apply the Federal rulei) Erie does not require the displacement of a Federal Rule with state law if they are not

consistent. Rather, if the Federal Rule does not cover the point in dispute then the state law governs. (1) RPD: This distinguishes Ragan nicely; Ragan held that the nature of Rule 4 did not

involve the statutes of limitations, so the state law was the applicable lawii) “It is true that [Erie] says, roughly that federal courts are to apply state ‘substantive’ law and

federal ‘procedural’ law, but from that it need not follow that the tests are identical.”iii) Procedural law is not a substantive right of the states, it is a necessary power authorized by

Congress to govern the efficiency and uniformity of the federal courts(1) “Erie and its offspring cast no doubt on the long-recognized power of Congress to

prescribe housekeeping rules for federal courts even though some of those rules will inevitably differ from comparable state rules.”

g) Thus, the Federal Rule wins in Hanna. h) Harlan, J., concurring

i) Opinion is right to modify the “outcome determinative” holding of Guaranty Trustii) But limiting Erie to forum shopping concerns is incorrect; court should have focused on the

procedural v. substantive distinction(1) In a substantive conflict b/t a Rule and state law, the state law should prevail(2) The opinion’s “arguably procedural, ergo constitutional” test moves too fast and far”

iii) Harlan thinks that Ragan was simply “wrong.”(1) Marshall, J., notes Harlan’s opinion in Walker v. Aramco Steel Corp.

iv) Cohen should be viewed in the sense that the state law reflected a legitimate policy concern about frivolous suits that was not reflected in the Federal law, and therefore the state law prevailed

v) Concurs in judgment b/c the effect would not be substantial regardless of the rule applied6) Rules Enabling Act v. Rules of Decision Act

a) Under the Rules Enabling Act, no Federal Rule may “abridge, enlarge, or modify any substantive right”

b) RPD: Hanna seems to imply that this cuts the other way as well – i.e., a Rule of Decision does not impede a procedural rule.

c) Cf., Sibbach v. Wilson & Co., 312 U.S. 1 (1941): Court affirmed district court’s order of P to undergo physical examination pursuant to Rule 35 despite an IL law prohibiting compulsory physical examinationsi) Rule 35 did not enlarge any substantive rightsii) The distinction b/t a procedural and substantive rule forms the basis of the applicability.

Procedure defined as “the judicial process for enforcing rights and duties recognized by substantive law and for justly administering remedy and redress for disregard of them.”

d) Filip: The problems come, then, b/c the Rules are not statutorily enacted in the same manner in which statutes are. How then does Erie apply?

7) When a Federal Rule does not encompass the same scope as a state law, the Erie/Ragan analysis applies. Walker v. Aramco Steel Corp., 446 U.S. 740 (1980)a) Case arose from a tort claim. OK state law held that a statute of limitations did not toll until the

process was served w/in 60 days of filing in court prior to the expiration of the statute. P failed to serve in time. P claimed that Rule 3 superseded the state law in this diversity suit under the Hanna.

b) Court held that Rule 3 and the OK statute were not in fact in “direct collision” and therefore the OK statute prevailedi) Rule 3 only applied to the measuring of time insofar as the Federal Rules are concerned and is

totally unrelated to Statutes of Limitations problemsii) Hence, Hanna distinguished Ragan rather than overruling it

(1) i.e., the Rule in Hanna was held to be in “direct collision” as opposed to the case at issue in Walker

44

Ryan Preston Dahl; 917 687 7146

(2) But see fn. 9: “This is not to suggest that the Federal Rules . . . are to be narrowly construed in order to avoid a “direct collision” with state law. The Federal Rules should be given their plain meaning”

iii) There was no Erie concern of an “inequitable administration” of the law which might otherwise require using the Federal Rule(1) “There is simply no reason why, in the absence of a controlling Federal rule, an action

based on state law which is would be barred in the state courts by the state statute of limitations should proceed through litigation to judgment in federal court solely because of the fortuity that there is diversity of citizenship.”

c) Casebook: Walker does not address the issue of whether or not Rule 3 effects the Federal statutes of limitations

8) Burlington Northern R. Co. v. Woods, 480 U.S. 1 (1987)a) P had obtained a money judgment at a jury trial. The verdict was affirmed on appeal w/out

modification; Court of Appeals assessed the 10% penalty prescribed by applicable state law for all unsuccessful appeals of money judgments. Burlington appealed on the basis of Rule 38 of FRAP, which limited such penalties to frivolous appeals.i) FRAP Rule 38: “If a court of appeals determines that an appeal is frivolous, it may, after a

separately filed motion or notice from the court and reasonable opportunity to respond, award just damages and single or double costs to the appellee.”

b) First step is to determine whether or not the laws and the rules are in “direct collision” – i.e., do they clearly speak to the same issue b/f the court such that the application of one would leave no room for the operation of the other

c) Court held Rule’s discretionary nature “unmistakably” conflicts with the mandatory nature of the state provisioni) Rule 38 is case by case (rather than mandatory)ii) Rule 38 allows discretion as to the amount of damages (rather than mandatory 10%)iii) RPD: This application seems to fall squarely w/in the Erie concern w/ “unjust” application of

lawd) Federal rules which incidentally affect litigants' substantive rights do not violate section of Rules

Enabling Act, providing that federal rules must not abridge, enlarge, or modify any substantive right, if reasonably necessary to maintain integrity of that system of rules.

9) If a state law contradicts a federal statute, the court must determine whether (1) the federal statute sufficiently covers the issue at hand and (2) whether application of the statute is consistent with the Constitutional power given under Art. III, § 2. Cf. Stewart Org. v. Ricoh Corp., 487 U.S. 22 (1988). If the statute is arguably procedural, it takes precedence over the state law. (RPD: This is Hanna.)a) Action arose out of a contract dispute. Contract contained a clause requiring that any action tried

in a court in New York/Manhattan. An action was ultimately brought in the Northern District for Alabama. Ricoh motioned to move the proceedings to the Southern District of New York under 28 U.S.C. § 1404(a)

b) If a Federal statute covers the point in contention, the district court must:i) Determine whether the statute covers the point in contentionii) Determine whether or not the statute is a valid exercise of Congressional authority under the

Constitutioniii) Whether Congress intended the statute to apply to the matter before the court

c) AL state law was hostile to such removal clauses. However, the Supreme Court held that the 28 U.S.C. § 1404(a) had greater weight than AL law in such an instance, and the motion to remove was therefore governed by analysis required by § 1404(a) i) However, the removal clause itself was not dispositive (i.e., you might refuse to transfer a

case even w/a selection clause) to the analysis required to see whether a motion under § 1404(a) was proper. Other factors include:(1) Balance of hardships(2) State interest(3) Systemic integrity(4) All under the statutory heading of “the interest of justice”(5) Note: Most people refer to these as “1404(a) issues” from a practical p.o.v.

45

Ryan Preston Dahl; 917 687 7146

ii) Forum selection clauses thus don’t mean nothing as AL likes, they don’t mean everything, but it’s something that’s a balanced test → thus 1404(a) trumps the state approach

iii) Then, you ask whether or not the law is “arguably procedural” and if yes, then the Rule trumps

10) It is the proper responsibility of a district court to review jury verdict according to the relevant state standard, while the Federal appellate court is constitutionally obliged to review under an abuse of discretion standard. Gasperini v. Center for Humanities, Inc., 518 U.S. 415 (1996) Glannon: State standard for reviewing size of jury verdict is “substantive.” a) Dicta: It is settled law that if the point in question is consonant with the Rules Enabling Act and

the Constitution, the Federal Rule supersedes the State rule. b) NY statute held that a jury verdict could be vacated by the either NY trial and appellate court if it

“deviates materially” from similar awards. i.e., the “deviates materially” qualification was the standard for appellate review under NY law → meant to replace the “shocks the conscience” test. Also called for de novo appellate review of the jury verdict at the appellate level. i) District court denied motion for a new trial applying the federal standardii) Circuit court vacated a judgment of the district court applying NY law.iii) Circuit Court found that the district court’s reliance solely on expert testimony and failure to

consider factors used in similar New York cases required the reversal under the NY statutec) Could the Federal court give effect to the “substantive thrust” of the NY statute without upsetting

the federal procedural scheme as required by the Seventh Amendment? d) Court held that Federal court would be obliged to apply the NY statute; the NY rule was “arguably

procedural” but also substantive. i) Using a pure abuse of discretion view standard and not the “deviates materially” standard of

the statute would significantly alter the outcome per Yorkii) Application of the state law would not significantly disrupt the federal procedural processes

per Byrd insofar as the “deviates materially” standard is applied by the district courtiii) The review of jury verdicts is consistent with the requirements of the VII Amendment at the

Federal Circuit court level if the review is conducted from a Constitutionally required standard – i.e., abuse of discretion(1) RPD: The problem the Court had with the Circuit court’s use of the relevant standard was

that this standard was essentially a finding of fact. (2) Thus, the Circuit court improperly assumed the role Constitutionally conferred to the

district court (as the finder of fact) under the VII Amendment(a) Distinction from Byrd is that Byrd was initially a trial/jury issue (would there be a

jury) as opposed to appellate review of a jury verdict(3) Circuit court review of a jury verdict is only appropriate under a “abuse of discretion”

standard → this is only reviewing a monetary amount and not a finding of fact (in the opinion’s thinking)

e) Stevens, J., dissentingi) Would affirm the Circuit Courtii) Byrd does not require VII Amendment analysis to determine the standard of review of a

district court’s application of state lawiii) Hostile to the extreme historicism of Scalia; Stevens in fact believes the history comes out

quite differentlyf) Scalia, J. dissenting

i) VII amendment was intended to prevent Federal appellate review of jury verdicts; appellate review was historically restricted to an error of law

ii) A review of verdict size necessarily involves a reexamination of the facts found by a jury; hence the state standard is constitutionally impermissible(1) The NY law is not a rule of law analogous to a cap on damages but a rule of review

which determines how much scrutiny is applied to the jury verdict(2) “A tighter standard for reviewing jury determinations can no more plausibly be called a

“substantive” disposition than can a tighter appellate standard for reviewing trial court determinations.”

iii) The opinion will lead to forum shopping(1) NY appellate review is de novo

46

Ryan Preston Dahl; 917 687 7146

(2) Federal appellate review is abuse of discretioniv) The state standard is simply inconsistent with the Constitutional requirements regarding the

ability of a judge to set aside a verdict(1) Court is particularly concerned with the degree of discretion held by the trial judge(2) Statute allows for verdict to be set aside if the verdict “in any degree” differed from the

proper measure of compensation(3) Rule 59 is thus essentially in direct collision with the state law and the Rule should take

precedence11) Method for Erie

a) If Constitution speaks to an issue: the Constitution will be applied in lieu of state lawb) If there is a Federal substantive statute (as opposed to common law) that speaks to the issue (e.g.,

duty of care for interstate shippers); apply the Federal statute (as opposed to common law)i) This is Supremacy Clause issue

c) If you have a Federal procedural statute (e.g., § 1404(a)) if it is sufficiently broad to cover the issue before the court (Stewart) and if it legitimately enacted (“arguably procedural”)

d) If you have a Federal Rule of Civil Procedure/Evidence, apply it if it satisfies the Rules Enabling Act and does not enlarge a substantive right.

e) If it’s a Federal judge-made rule, apply the i) Twin-aims test of Hannaii) Byrd, Gasperiniiii) York, though York more of a “slogan” than an analytical tool

Interstate Choice of Law1) Federal courts must apply the conflicts of laws rules of the states in which they sit in diversity cases.

Klaxon Co. v. Stentor Elec. Mfg. Co., 313 U.S. 487 (1941)a) Rule is based on a desire for uniformity in the application of the substantive law within a state,

despite any effects it might have on the Federal systemb) Filip: In Federal question cases you have, in effect, Federal common law rules to deal with choice

of law principles and you can therefore have different choice of law rules with respect to Federal question and state claims in one actioni) The Federal choice of law question would regard how the court treats precedents on the

Federal question that arise from different circuits (not the Federal law per se)c) E.g., any garden-variety police misconduct civil suit (e.g., false arrest)

i) P would bring a claim under § 1983/4th Amendment (Federal question)ii) This will bring with it a state law battery claim (state law cause of action under supplemental

jurisdiction)iii) Or, alternatively, a trademark claim by an IL company against an IW defendant which would

give rise to Lanham Act claims and state claimsd) E.g, a tort claim arising in PA is filed in NY under diversity jurisdiction. The NY district court

must apply NY law regarding choice of law issues. 2) State can apply its substantive law in a case so long as the state has significant contacts or a significant

aggregation of contacts with the parties and the transaction. Allstate Inc. Co. v. Hague, 449 U.S. 302 (1981).a) This offers a great deal of discretion to the courtsb) Filip: Allstate analysis proceeds on a plaintiff-by-plaintiff basis, not on an aggregate basis

3) A change in venue under 28 U.S.C. § 1404(a) does not change the substantive law that governs the case. Van Dusen v. Barrack, 376 U.S. 612 (1964)a) Primary concern underlying this policy is that of forum shopping w/in the Federal System

Prior Adjudication Generally1) Former adjudication has four underlying features

a) Party only has one chance to litigate a claim; if you litigate only a portion you risk losing the ability to litigate the rest

47

Ryan Preston Dahl; 917 687 7146

b) Party gets only one chance to litigate a factual or legal issue; once decided you can’t ask to have it re-litigated

c) A party is entitled to one “full and fair” chance to litigate b/f being precludedd) Preclusion may be waived unless it is claimed at an early stage of litigation

2) Res judicata speaks to two distinct issuesa) Claim preclusions (true res judicata)

i) A judgment, once rendered, is the full measure of relief accorded b/t parties on the same “claim” or “cause of action”

ii) If P wins, his claim “merges” into the judgmentiii) If D wins, plaintiff’s claim is extinguished and judgment acts as a “bar”iv) Claim preclusion extends to all issues relevant to the same claim whether or not raised at trial

b) Issue preclusioni) Issue preclusion bars the re-litigation of issues actually adjudicated and essential to judgment

in a prior litigationii) The contested issue must have been litigated and necessary t the judgment earlier renderediii) E.g., dismissal of a suit for want of federal subject matter jurisdiction does not bar action on

the same claim, but it normally does preclude relitigation on the issue of subject matter jurisdiction in a second federal suit on the same claim

3) Res judicata and sequence of actions and judgment (Wright & Miller § 4404)a) If two actions are pursued simultaneously, the first judgment entered has res judicata effect on the

remaining actioni) Note: Failure to protest/object may waive the defense that a single cause of action has been

inadmissibly been split into two partsb) A Court may choose to consolidate multiple actions, or allow them to be pursued independently

and simultaneously4) Filip: Res judicata and collateral estoppel are related but different. However, both speak to the effect

of prior litigation on the rights/obligations/duties of parties in the futurea) Res judicata → claim preclusion

i) Seeks to prevent people from litigating the same disputeii) Prohibition on “claim splitting ”

(1) E.g., tort suit for damage to right panel of car, followed by another suit for damage to left panel

b) Collateral estoppel → issue preclusion5) Res judicata

a) rationalei) Incentivizes parties to get things right the first timeii) Avoids inconsistent judgmentsiii) Minimizes the times you will try/re-try issuesiv) Allowing limitless litigation would simply reward the deepest pockets

b) Two elementsi) A judgment in a dispute is a full measure of relief on that claim/cause of action

(1) P wins: claim merges into judgment(2) D wins: claim is barred from proceeding further

ii) The claim cannot be retriedc) This extends to all issues related to that dispute

i) You forget to make an argument it’s done ii) E.g., you wanted to pursue the action as a fraud case but you forgot, too bad

d) Rule 13 feeds into these concerns6) Issue preclusion/collateral estoppel

a) Suits addressed to particular claims can prevent suits relevant to other claims; so the bar it related to particular issues that wherei) Actually adjudicatedii) Essential to the judgment in a prior litigation between the parties

Claim preclusion1) Generally

48

Ryan Preston Dahl; 917 687 7146

a) When a second suit is brought, the judgment in a prior suit will be considered conclusive both on the parties to judgment and those in privity w/themi) E.g., judgment is entered holding that a certain amount is due on a promissory noteii) Even if other valid defenses existed but were not offered, the judgment is conclusive insofar

as future proceedings are concerns as though those defenses never existedb) Per Cromwell v. Cty. Of Sac, 94 U.S. (4 Otto) 351 (1876) three factors must exist for claim

preclusion to operate:i) Judgment must be final, valid, and on the meritsii) Parties in a subsequent action must be identical to those in the first

(1) This distinguishes claim preclusion from issue preclusioniii) Claim in the second suit must involve matters properly considered included in the first action

c) A judgment does not preclude everything that might have been disputed between the parties, but only matters within a certain sphere. Three different labels are used to describe the precluded area: "claim," "demand," and "cause of action." Wright & Miller § 4406i) If the plaintiff loses, the entire claim is barred by the judgment, even as to evidence, theories,

arguments, and remedies that were not advanced in the first litigation. The process of defining the claim or cause of action is thus aimed at defining the matters that both might and should have been advanced in the first litigation

ii) The second lawsuit involves a new claim or cause of action, the parties may raise assertions or defenses that were omitted from the first lawsuit even though they were equally relevant to the first cause of action

d) What constitutes a claim/cause of action? Wright & Miller § 4407i) Broad definitions tend to arise:

(1) Would a judgment in a second action impair or destroy the rights established in the first?(2) Is the same evidence necessary to maintain the second action as was in the first?(3) Are the same essential questions of facts and issues present in the second as in the first?

ii) Restatement (Second) of Judgments § 24 the claim extinguished by a first judgment(1) Includes all rights of the plaintiff to remedies against the defendant with respect to all or

any part of the transaction, or series of connected transactions, out of which the action arose.

(2) What factual grouping constitutes a "transaction," and what groupings constitute a "series," are to be determined pragmatically, giving weight to such considerations as whether the facts are related in time, space, origin, or motivation, whether they form a convenient trial unit, and whether their treatment as a unit conforms to the parties' expectations or business understanding or usage

iii) Wright & Miller: The Restatement has been adopted in so many federal jurisdictions so as to become the rule, though it isn’t clear that the Supreme Court has explicitly done so

iv) One major function of claim preclusion is to force a plaintiff to explore all the facts, develop all the theories, and demand all the remedies in the first suit

e) Claim preclusion in actioni) Contract cases often can be resolved by a simple rule that the first suit must claim every

breach that has then occurred. Courts have increasingly come to apply this rule to situations in which they find that a single basic contractual commitment has been embodied in multiple instruments

ii) In Tort, A single injury gives a single cause of action, even if the underlying conduct is complex and the injury is abstract.(1) Note: Tort claims that involve more regular continuing practices or relationships present

more complicated problems iii) Multifaceted relationships provide other settings in which plaintiffs may be free to pursue

substantially independent disputes in separate actions.(1) The line between the consequences of completed past conduct and the impact of

continuing or renewed conduct is itself blurred when the consequences of completed conduct are aggravated by separate or continuing conduct. Aggravation of an original injury by separate or continuing conduct may support a new claim

49

Ryan Preston Dahl; 917 687 7146

(2) But a second challenge to continuing conduct may be precluded despite the general rule if the object of the first proceeding was to establish the legality of the continuing conduct into the future

f) Defendant preclusioni) Three main types:

(1) D seeks to advance a claim against original P not advanced in first action(2) D seeks to advance a claim against original P not advanced in first action but were not

barred by issue preclusion(3) In second action by original P, D seeks to raise defenses that were equally available in

first action but were not advanced thereii) Generally, most claims by D against P will be governed by Rule 13(a) (compulsory

counterclaims) and not claim preclusion, though permissive counterclaims are an obvious exception

iii) At a minimum, a former defendant must be precluded from enjoining enforcement of a judgment on the basis of a defense that was available but not raised in the first action.

2) A single tort gives rise to a single cause of action for the purpose of claim preclusion. Rush v. Cty. Of Maple Heights, 147 N.E.2d 599 (Ohio 1958)a) P was injured in a motorcycle action. P commenced two suits against the city, one in municipal

court and one in county court. i) Municipal court awarded damages of $100ii) County court found that the prior municipal finding of negligence was res judicata, and

entered a verdict for P of $12,000b) Court held that plaintiff was improperly allowed to split her cause of action between the two

courtsi) Court rejected the minority rule that a tort could give rise to two causes of action for property

damage and for personal injury(1) This rule is really more concerned with the ability of insurance companies as subrogees

to recover in an action for insured property and allow the plaintiff to recover for uninsured injuries

(2) The rule is not intended to simply allow for multiple claimsii) Majority rule is simply one tort, one action

(1) “A separation of causes of action is almost universally recognized where an insurer has acquired by an assignment or by subrogation the right to recover for money it had advanced to pay for property damage.”

3) A party may waive preclusion by failing to raise it as an affirmative defense in the second suita) RPD: Fed. R. Civ. P. 8(c).

4) The res judicata consequences are not "altered by the fact that the judgment may have been wrong or rested on a legal principle subsequently overruled in another case." Res judicata/claim preclusion will be strictly enforced. Federated Dep’t Stores, Inc. v. Moitie, 452 U.S. 394 (1981). If you don’t appeal, you’re going to be barred. a) Moitie and Brown brought were two of seven individuals to bring anti-trust suits. Their claims

were dismissed from Federal Court. They brought the claim in state court, which was removed to federal court and dismissed on res judicata grounds. i) The remaining 5 plaintiff’s had appealed to the Circuit Court, which reversed and remanded

in light of an intervening Supreme Court opinionii) Moitie and Brown appealed the res judicata dismissal, since it rested on the original district

court case that had been overruled by the intervening supreme court opinionb) However, the Supreme Court upheld the lower court’s res judicata dismissal

i) Interest in maintaining the doctrine outweighed the interest of the individuals in this caseii) “[res judicata] is a rule of fundamental and substantial justice . . . which should be cordially

regarded and enforced by the courts.”d) The plaintiffs "seek to be the windfall beneficiaries of an appellate reversal procured by other

independent partiese) But see McCarty v. First Georgia Ins. Co., 713 F.2d 609 (10th Cir. 1983): when prior judgment

was obtained by the use of fraud it will not be binding.

50

Ryan Preston Dahl; 917 687 7146

i) P had brought a claim against his InsuranceCo for failing to pay out on a policy. District Court granted summary judgment for InsuranceCo

ii) Circuit court held that res judicata did not bar the subsequent claim – though it was clearly the same transaction – since insureds discovered insurer's wrongful concealment of true facts surrounding issuance of policy only after conclusion of the contract action

8) Difference b/t res judicata and stare decisisa) Res judicata applies only to the parties of the suit; Stare decisis binds all subsequent litigantsb) Stare decisis is about legal principles; res judicata is about facts

9) Analysis: The Second case is precluded if the following 5 things are truea) Same parties as in the first case or people who are in privity with those partiesb) Same cause of action as existed in the first case, assessed under the “transaction” or occurrence

test (also Fed. R. Civ. P. 13)c) Judgment in first case has to have been Final, valid, and on the merits

i) E.g., lack of personal jurisdiction does not support a defense of res judicataii) With respect to Finality, finality under most state’s legal regimes, finality exists at the end of

the trial court’s decisioniii) With respect to the “merits” you’re talking about a summary judgment, trial, failure to state a

claim without leave to replead (i.e., a dismissal with prejudice)(1) Does not include lack of subject matter jurisdiction, venue, personal jurisdiction, or a

12(b)(6) motion with leave to re-pleadd) if the first three factors have been satisfied, then all the legal claims raised in the first case or that

could have been raised in the first case are precluded from re-litigationi) Thus, if you had brought the initial action in a Tax Court which would only hear tax cases,

then there is no preclusion b/c there was no opportunity to bring up the caseii) Also, if there were latent injuries for which you could not have known, then there is no bar

e) The res judicata bar is only raised if the defendant raises it10) Claim preclusion and appeals

a) Federal Courts grant preclusion pending appeal of an underlying judgment, but state rule varyb) Fed. R. Civ. P. 65(b)(5): On motion, the court may relieve a party from final judgment if . . . “the

judgment has been satisfied, released, or discharged, or a prior judgment upon which it is based has been reversed or otherwise vacated, or it is no longer equitable that the judgment should have prospective application.i) Hence, the preclusive effect of the judgment is likewise abolishedii) So a remand for retrial would then eliminate the preclusive effect of the prior judgment

c) What if the legal underpinning of the judgment is overruled?d) Many cases establish the rule that once the time for appeal has run, a final judgment of a trial court

or an intermediate appellate court is res judicata without regard to the fact that appeal might have been taken to a higher court. Wright & Miller § 4433i) This rule applies equally to failure to take a cross-appeal. ii) It makes no difference that an appeal was attempted but was thwarted by failure to satisfy

procedural requirements. iii) Nor is it likely to make a difference that some parties successfully appeal while others do not

appealiv) The bare act of taking an appeal is no more effective to defeat preclusion than a failure to

appeal.11) The existence of a defense of res judicata as a result of a federal decision does not allow for removal

from state court to Federal Court under 28 U.S.C. § 1441(b). Rivet v. Regions Bank, 522 U.S. 470 (1998)a) Removal under § 1441(b) is determined by the well-pleaded complaint rule (federal jurisdiction

exists only when a federal question is presented on the face of the plaintiff's properly pleaded complaint)i) Note: a plaintiff may not defeat removal by omitting to plead necessary federal questionsii) Thus, if a court concludes that a plaintiff has "artfully pleaded" claims in this fashion, it may

uphold removal even though no federal question appears on the face of the plaintiff's complaint. The artful pleading doctrine allows removal where federal law completely preempts a plaintiff's state-law claim

51

Ryan Preston Dahl; 917 687 7146

b) A defense is not part of a plaintiff's properly pleaded statement of his or her claim; “a defendant cannot remove on the basis of a federal defense.”

c) The defense should have been made in state court: “claim preclusion by reason of a prior federal judgment is a defensive plea that provides no basis for removal under §1441(b). Such a defense is properly made in the state proceedings, and the state courts’ disposition of it is subject to this Court’s ultimate review”

Issue preclusion1) Cases are not the same, but there are similar issues

a) E.g., landlord sues a tenant for failure to pay rent in 2004. Tenant does not want to pay rent in 2005i) No res judicata bar b/c the damages accruing in the future did not arise in 2004ii) But the issues will be the same

(1) E.g., if tenant attempted to invalidate the lease, issue preclusion will come into play(2) Thus, the D will be precluded from taking a second run at the defenses

b) Differences from res judicatai) Res judicata bars entire causes of actions; collateral estoppel is more preciseii) collateral estoppel only resolves part of the lawsuit (as opposed to the all-or-nothing element

of res judicata)iii) Res judicata requires privity; collateral estoppel has substantially relaxed that mutuality

requirementiv) Collateral estoppel does not deal with issues that could have been raised but weren’t

(1) It is all about issues that necessarily were decided and were essential to the prior judgment

(2) That is, collateral estoppel only function if an issue was raised and decided, as opposed to res judicata which applies to claims which were not in fact raised

v) Res judicata is a defensive doctrine only; you can’t offensively use res judicata. Collateral estoppel can be used offensively(1) This is a tremendous issue for serial defendants (RPD: e.g., insurance companies)

2) Requirements to trigger collateral estoppela) Issue of fact or law (typically fact)b) Issue must have been raised and necessarily decided in the first actionc) Issue must have been decided conclusively in the first action – i.e., final judgment on that issue on

the meritsd) Issue must have been necessary to the judgment of the first actione) Issue then becomes conclusive in the second point if:

i) The collateral estoppel issue is raisedii) If the second case involves the same parties (though this is not necessarily the case in most

jurisdictions)iii) And this is true even if the case involves a different claim

3) Collateral estoppel only functions to estop those matters actually decided in a previous action between the parties. Cromwell v. Cty. Of Sac, 94 U.S. (4 Otto) 351 (1876) (Field, J.)a) County of Sac had contracted to build a courthouse. Bonds were issued by the county judge to the

contractor. The contractor in fact gave the judge a bond as a “gratuity.” The courthouse was never constructed. i) In an earlier case, Smith, a coupon holder, brought action against the county but since he was

not a BFP his claim was held to be void as against the countyii) County relied on this to claim that Cromwell was estopped on the basis that Cromwell was in

fact the owner of the bonds in Smith’s action(1) RPD: I presume that this would then put Cromwell in privity with Smith such that res

judicata would bar the claim. iii) However, Cromwell was bringing suit for on four particular bonds not adjudicated in the first

actionb) Court held that the previous ruling did not estop Cromwell’s claim

i) The action on the four bonds constituted a different claim

52

Ryan Preston Dahl; 917 687 7146

(1) “The fact that a party may not have shown that he gave value for one bond or coupon is not even presumptive, much less conclusive evidence that he may not have given value for another and different bond or coupon.”

(2) RPD: Thus, each bond becomes a discrete cause of action/transaction, presumably if different circumstances surround their acquisition

ii) Therefore, the only issue relevant from the prior case was that if Cromwell could not demonstrate he was a BFP he would then be barred from recovery, the earlier case did not make any determination regarding his acquisition of the four bonds in question

c) Filip: If the issue is not raised and is not tried on the merits then there is no collateral estoppeli) P was not barred by res judicata because the bonds at issue had not matured when the

previous action had been brought (if it could not have been brought, no res judicata)d) See also Gwynn v. Wilhelm, 360 P.2d 312 (Or. 1961): default judgment against patient for failure

to pay does not estop a claim of malpractice against the physiciani) “If the exercise of due care and skill by defendant in caring for plaintiff was essential to the

judgment and was actually litigated and determined in the justice's court, plaintiff would be estopped by the judgment.” (Emphasis RPD).

ii) Since it was a default judgment, the physician’s level of care was not adjudicated and therefore not binding upon a subsequent action between the parties

iii) Court noted that this rule applies whether or not a compulsory counterclaim rule was in place: “[even absent a compulsory counterclaim rule] a party cannot recover in a separate action on a cause of action which he failed to plead in a prior action by way of setoff or counterclaim but which was necessarily adjudicated by the former judgment”

iv) Restatement (Second) of Judgments § 27 adopts the actually litigated rulee) But see Blair v. Bartlett, 75 N.Y. 150 (1878), for the minority rule: default judgment against

patient estops subsequent action for malpracticei) Essentially, a default judgment constituted a matter tried b/f the courtii) New York is the only state with this rule, per Gwynn

4) An issue raised in a prior action must have been clearly decided in that action for a subsequent party to be estopped from raising it. Russell v. Place, 94 U.S. (4 Otto) 606 (1876) (Field, J.)a) Patent infringement case. Ps had a patented process involving a two-stage process for the tanning

of leather. Ps claimed infringement on the patenti) District court record did not indicate how the infringement took place, nor which element of

the process was in fact infringed (it was a general verdict)ii) Ds subsequently claimed want of novelty in the design; P claimed D was estopped from

pleading that issue due to the finding of noveltyb) Court held that since the issue of novelty was not clearly decided by the lower court, D was not

estopped from making such a plea since “the validity of the patent was not necessarily involved”i) “It must appear, either upon the face of the record or be shown by extrinsic evidence, that the

prices question was raised and determined in the former suit.”ii) The paucity of the record did not indicate the basis on which the lower court found

infringement or that the court addressed the issue of novelty at alliii) This problem could have been avoided had there been a Special Verdict and not a General

Verdict5) Filip: in the wake of a general verdict there is a presumption against collateral estoppel; so be careful

about whether or not you want a special verdict (which might in fact be harder for you to get)6) What is a necessary fact for the purposes of collateral estoppel? “Those facts, upon whose combined

occurrence the law raises the duty, or the right, in question.” 141 F.2d 927 (2d Cir. 1944) (Hand, J.) – i.e., the fact must be “within the risk” of the original action

7) RPD: Is there really room for issue preclusion given the role of Rule 13?8) Example

a) P sues D to recover interest on a promissory note payable to P. Principle has not come due yet. D raises two defenses:i) D was fraudulently induced to make the note; and, alternatively,ii) P gave D a release under seal of payment obligations

b) P wins and gets interest.

53

Ryan Preston Dahl; 917 687 7146

c) Second case, P wants to sue D for the principle when the principle becomes due. Can D make the fraudulent inducement claim again?i) No. There is no way the plaintiff could have won without overcoming that defense (i.e., D

could not have won on his second defense but lost on his first)9) Example

a) P sues D to recover interest on a promissory note payable to P. Principle has not come due yet. Principle has not come due yet. D raises two defenses:i) D was fraudulently induced to make the note; and, alternatively,ii) P gave D a release under seal of payment obligations

b) Court finds that P gave a release as to the interest obligations, but that there was no fraudulent inducement.

c) Second case, P sues D for the principle. Can D assert a fraud defense in the second suit?i) Yes. The finding or lack of finding of fraud does not necessarily determine the outcome of the

first litigation. The only thing which necessarily determines the outcome of the prior action is the finding of the release.

ii) The outcome was necessarily determined by the finding that a release existed10) Example

a) P sues D to recover interest on a promissory note payable to P. Principle has not come due yet. Principle has not come due yet. D raises two defenses:i) D was fraudulently induced to make the note; and, alternatively,ii) P gave D a release under seal of payment obligations

b) Court finds that D was induced by fraud and that P gave a releasec) P sues for a second installment of interest. Is the determination that P is not liable for interest

conclusive?i) RPD: Claim preclusion?ii) No. Both issues can be re-litigated b/c it’s unclear which one was necessary to the previous

case. (This is the majority rule)(1) Filip: This almost makes it worse for the party who wins on multiple grounds than the

party who wins on just one ground; though this is a rather unsophisticated view of collateral estoppel

11) Examplea) P sues D to recover interest on a promissory note payable to P. Principle has not come due yet.

Principle has not come due yet. D raises two defenses:i) D was fraudulently induced to make the note; and, alternatively,ii) P gave D a release under seal of payment obligations

b) Court finds, on independent bases, as to the first interest payment that i) D was fraudulently induced to make the note; and, alternatively,ii) P gave D a release under seal of payment obligations

c) A court might hold that the issue of interest payments was sufficiently foreclosed as to estop D from litigating his liabilityi) Filip: this is a much more complicated case that likely will not appear on the Exam

Privity and Non-mutuality1) Examples (non-mutual collateral estoppel)

a) Example 1:i) Passenger sues the manufacturer of the S.S. Titanic in negligence. Titanic wins. Can Titanic

then assert this against a second passenger in a different suit?ii) No. Passenger two was not a participant in the first lawsuit

b) Example 2:i) P is injured by a Servant driving a truck. P wins suit against employee for negligence, and

company was not a party to the first suit. Can P assert preclusion against employer under respondeat superior?

ii)2) Offensive non-mutual collateral estoppel: you want to use it to attack a party3) Defensive non-mutual collateral estoppel: you want to fend off a party on that basis

54

Ryan Preston Dahl; 917 687 7146

a) Most jurisdictions do not require mutuality in a defensive setting; this is the majority rule and it is applied pretty broadly

b) Bernhard is the typical case of defensive non-mutual collateral estoppel4) Though a party against whom collateral estoppel is pleaded must have been a party to the original

action, the party pleading collateral estoppel (i.e., using collateral estoppel offensively) need not have been a party to the previous action. Bernhard v. Bank of America Nat. Trust & Sav. Ass’n (Traynor, J.)a) Case arose from a dispute regarding the transfer of assets from a decedent’s bank account prior to

her death in favor of her executor. Following the death of the decedent, the executor submitted an account making no mention of the transfer. He resigned his position and the next executor (Bernhart) brought suit against the previous executor. i) Probate court ruled the transfer was a valid giftii) Bernahart subsequently brought suit against the bank; claiming bank could not plead res

judicata since there was no mutuality of estoppelb) Court held that mutuality was not required for a party to plead estoppel, only that the party against

whom res judicata is pleaded would be bound by the previous rulingi) “The criteria for determining who may assert a plea of res judicata differ fundamentally from

the criteria for determining against whom a plea of res judicata may be asserted. . . . There is no compelling reason . . . for requiring that the party asserting the plea of res judicata must have been a party, or in privity with a party to the earlier litigation.”

c) See also Blonder-Tongue Labs. Inc. v. Univ. of Ill. Found., 402 U.S. 313 (1971) abrogating the requirement of mutuality in the federal systemi) Case was regarding a patent infringementii) Narrow reading of Blonder-Tongue might be to hold that it only applies in the context of

patent law, but lower federal courts have applied it to all types of substantive claimsiii) See Zdanok v. Glidden Co., 327 F.2d 944 (2d Cir. 1964) (Friendly, J.): Adopted to Bernhart

Ps may invoke collateral estoppel against D based on a previous action from an identical set of facts; though the opinion noted that this principle exposed the D to substantial liabilities from opportunistic, unrelated Ps.

d) Example:i) P sues N. P wins. Court finds that N was driving negligently. ii) N now sues D. Is N now subject to collateral estoppel? Yup (though some states might not

adopt this majority view)5) Trial courts have broad discretion to permit offensive collateral estoppel by a plaintiff who was not a D

in a prior suit against a defendant. Parklane Hosiery Co. v. Shore, 439 U.S. 322 (1979)a) Can a party not a litigant to the previous action use collateral estoppel offensively to prevent a

defendant from relitigating issues resolved by that prior action?b) Hornbook: Non-mutual collateral estoppel requires that the party who is being estopped must have

litigated and lost the issue in the prior suiti) The party invoking the estoppel may be newii) The party against whom the estoppel is invoked may not be new

c) D brought a shareholder’s derivative suit against P alleging fraud. SEC jumped in on the same charge. SEC won equitable relief. D sought to estop P from asserting the same set of facts that were decided by the SEC court. P claimed this violated their VII Amendment rights

d) Court held thati) “Trial courts [have] broad discretion to determine when [offensive collateral estoppel] should

be applied.”(1) General rule is that where a P could easily joined in the earlier action/application of

offensive estoppel might be unfair to a defendant trial judge should not allowii) P was not entitled to such a defense here

(1) They had a “full and fair” opportunity to make their case against the SEC(2) There are no procedural opportunities not available to the P that would have otherwise

been available(3) D is statutorily barred under 15 U.S.C. § 78u(g) from joining a SEC suit without the

consent of the SECe) But see Rehnquist, J., dissenting: P was denied their VII amendment right to trial by jury

55

Ryan Preston Dahl; 917 687 7146

i) RPD: Remember suits in equity are not tried to a juryii) Federal policy is to favor jury trialsiii) Different outcome is possible under a jury (RPD: That’s precisely why you don’t want to let

that sort of thing occur)6) A party not properly joined under Rule 19 to a prior action is not subject to collateral estoppel based on

that prior judgment. Martin v. Wilks, 490 U.S. 755 (1989)a) Black Firefighters and the City of Birmingham developed an affirmative action program, and

obtained a consent judgment from a court. White firefighters sued. City argued that the consent decrees precluded any allegations as to the propriety of the programi) Consent judgment: A settlement that becomes a court judgment when the judge sanctions it. •

In effect, an agreed judgment is merely a contract acknowledged in open court and ordered to be recorded, but it binds the parties as fully as other judgments. -- Also termed consent judgment; stipulated judgment; judgment by consent

b) Court held that since the White Firefighters were not joined – since they were interested parties – they could not be bound by the prior judgmenti) “Joinder as a party, rather than knowledge of a lawsuit and an opportunity to intervene, is the

method by which potential parties are subjected to the jurisdiction of the court and bound by a judgment or decree.”

ii) “A voluntary settlement in the form of a consent decree between one group . . . and their employer cannot possibly “settle,” voluntarily or otherwise, the conflicting claims of another group of employees who do not join in the agreement. This is true even if the second group of employees is a party to the litigation.”

iii) RPD: Rehnquist is basically saying that the onus is on the parties in the suit to join the third party as required by Rule 19, as opposed to a system of mandatory intervention per Rule 24 which apparently becomes voluntary

7) Examplesa) Case 1: A sues B. A loses. Case 2: C sues A and wants to use facts used in the first case to estop

A..i) C may estop A

b) Case 2: A sues B and A wins. Case 2: C sues B and wants to use facts that where decided against Bi) C may estop B so long as you meet the Parklane factors.ii) The equities in this scenario are slightly difference from Example 1, so the court will be more

hesitant to permit offensive non-mutual collateral estoppel

Class ActionsGenerally1) Filip:

a) Two perspectives on Class Actions: i) it’s a joinder device, like a super Rule 20 (permissive joinder of parties)ii) It’s a representative device, allowing the representative party to speak for others regardless of

whether they did litigate the issueb) Two viewpoints as the positive/negative nature of the Class action

i) Plaintiff’s bar likes them. Rationale:(1) They provide equal treatment to similarly situated Ps and Ds, and provide equal outcomes

for everyone.(2) Efficiency for courts and consolidation of claims (e.g., aggregating 100,000 cases of $50

harms)ii) Defendant’s bar and corporations tend not to like them

(1) Agency issues: conflict b/t the interests of the putative principles (plaintiff’s attorney has no real client)

(2) D can’t afford to go to trial based on the risks (3) One shot resolution of the problem isn’t the best way to litigate it, perhaps its better to

have a series of litigations to get a consensus as to where things come out2) Initiation of Class Actions

a) Permission of potential members not required prior to initiation of the action

56

Ryan Preston Dahl; 917 687 7146

i) E.g., it’s likely that the first class action suit for VIOXX was filed within 1 hour of the press release

b) Members have an opportunity to opt out in many jurisdictions; individuals are not considered members until they are notified and have an opportunity to “opt in”

3) Certification: Burden is the Plaintiff to prove all seven requirements to receive certificationa) Filip: Five-part analysis in certification

i) Certification can be adapted/molded notwithstanding the parties’ input (e.g., subclasses can be put in if the court thinks it necessary)(1) It’s subject to modification (provisional approval which can be altered later on)

ii) Frames the boundaries of the case as it moves forwardiii) Approves a particular description of the class

(1) Must be concrete enough to define the classiv) Defines the issue to be considered by the actionv) Specifically names who the class representatives will be and designates class counsel

(1) Cf., Hansberryb) There must be a definable class

i) Class must be determinate and have boundaries that can be reasonably ascertained w/out having to litigate each situation to determine whether or not they are in the class

ii) E.g., “class defined as all people who received negligent medical treatment” does not get you there

iii) E.g., “class of people who failed to get a rebate check from Sprint” is a definable classc) Class representative must be a member of the class (this is usually a question of standing)d) Rule 23(a)(1): Class must be so numerous that joinder is “impracticable”

i) >45 usually gets you thereii) <25 no certificationiii) <25<45; court will consider practical factors and the ability of joinder to satisfy individual

claims (joinder is usually feasible if individual claims are large)e) Rule 23(a)(2): Action raises a question of law or fact common to the class

i) Filip: this usually isn’t a big issue as you can find some sort of commonality; the real issue is whether or not the issue predominates

f) Rule 23(a)(3): Representative party must be typical of those of the classi) E.g., a defense/claim unique to the representative is not sufficient

g) Rule 23(a)(4): Representative party will fairly and adequately protect class interestsi) Court must ascertain representative party has an adequate stake in litigationii) Adequacy of class’ lawyeriii) Affirmative responsibility to determine internal antagonism w/in the class

h) Rule 23(b) must be satisfied as well (Filip: in practice, 23(b) collapses down into two classes)i) Rule 23(b)(1)

(1) Would individual actions prejudice the non-class party? (e.g. ability of election board to manage its operations if suits were brought individually)

(2) Would individual actions prejudice ability of class members to get recovery (e.g., multiple claimants to a limited fund, such as an insurance award)

(3) Filip: this is to prevent destructively inconsistent judgments, typically that concern standards of conduct(a) E.g., administration of government programs, limited funds cases

ii) Rule 23(b)(2)(1) For actions seeking injunctive relief, such relief would be generally appropriate (though it

doesn’t have to be appropriate for every member)(2) Most class actions are Rule 23(b)(2) class actions(3) Filip: 23(b)(1) & 23(b)(2) are pretty similar; (b)(2) is when a party has acted or refused to

act in a way that affects the entire class(a) E.g., where the CHA has a particular practice and the desired outcome is a settlement

for an injunctioniii) Rule 23(b)(3) “damage” class actions

(1) Question of law/fact must “predominate” any questions affecting individual members(2) The class action must be manageable from an efficiency standpoint

57

Ryan Preston Dahl; 917 687 7146

(3) Class action is superior to other adjudication methods (a) Essentially, the efficiency and economy must outweigh the interest in individual

adjudication(4) Factors to be considered in evaluating a Rule 23(b)(3) suit:

(a) Interest of members in individually their own cases(b) Nature of any litigation commenced by or against class members(c) (Un)desirability of concentrating litigation (d) Difficulties in class management

(5) Opt-in and Opt out provisions are only required in 23(b)(3) class actions4) Certification will then determine

a) Terms on which the litigation will / will not proceedb) Defines substantive issues the suit will considerc) The class representantive will be appointed

5) Effect of certificationa) Tactical importance in settlement talksb) Certification is not appealable as a final decision under 28 U.S.C. § 1291, but can be certified

under 28 U.S.C. § 1292 or under a writ of mandamus6) Notice

a) Discretion as regards notice in Rule 23(b)(1) and Rule 23(b)(2)b) Notice is required in every Rule 23(b)(3)c) Costs for notice are born by the class. Oppenheimer Fund, Inc. v. Sanders, 437 U.S. 340 (1978)

disapproved of prior practice which used Discovery rules to force the defendant to provide notice7) Administering relief/proving claims

a) Problem is matching relief to individual harms suffered by class members (which obviously can be diverse)

b) Methods used to solve this problem arei) Bifurcated trials (liability, damages)ii) Court’s discretioniii) Sampling

8) Settlement. Rule 23(e) provides claims cannot be settled/dismissed/compromised w/out court approval9) Role of Attorneys

a) Attorneys have tremendous control over the process. Generally court will appointi) Liaison to the court; andii) Attorney to hand the practice elements; andiii) Some sort of management committee (if the suit is big enough)

b) Fees pose a tremendous issuei) Lodestar method is used most oftenii) There is considerable debate over whether performance enhancements may be used to

increase the feesiii) Note: Atty’s fees are shifted in Federal Civil Rights litigation under 42 § 1988; this can create

special problems in class action litigation10) A representative must adequately represent the interests of the members of the class. A representative

that represents the interest of a class only insofar as its members may either oppose or desire a particular obligation – as opposed to where the “sole and common interest of the class . . . is either to assert a common right to challenge an asserted obligation” does not adequately represent the class. Hansberry v. Lee, 311 U.S. 32 (1940)a) Filip: You can have a collateral attack on a class judgment. Someone who is at least arguably

bound by a prior class judgment has an opportunity to prove that his interests were not adequately represented in the first case

b) Case arose from racist zoning ordinances in Chicago. P attempted to push a sale through which D opposed, pleading res judicata by virtue of an earlier class action brought by white homeowners in the same area. i) This prior case litigated the original zoning covenant on behalf of all the residents of the areaii) P had claimed that 95% of landholders required to uphold the ordinance was not met – as only

54% of landholders had actually voted for the ordinance (the prior suit had gotten around this claim somehow)

58

Ryan Preston Dahl; 917 687 7146

(1) This 95% was stipulated in the original case. However, under IL such a stipulation would be given res judicata effect in subsequent litigation (though this might have come out differently under a different state law)

iii) The IL court had barred Hansberry’s claim on res judicata grounds on the theory that Hansberry was in privity with the landholders (privity ran with the land)

c) Court rejected this argument. The common interest in a particular zoning ordinance was insufficient to consider Hanesberry “adequately represented” by the previous action. Only where “the sole and common interest of the class in the litigation is either to assert a common ort or to challenge an asserted obligation” does such a preclusion hold. i) RPD: that is,

11) Adequacy of representation will be determined by (1) whether the trial court adequately determined if the representative adequately represented the class and if (2) after the termination of the suit the class representative adequately protected the interest of the class. Gonzales v. Cassidy, 474 F.2d 67 (5th Cir. 1973)a) Case was a class action challenging at TX statute suspended a drivers’ license if he failed to post

bond if he was involved in an auto accident as security for damages. P won the case, but P asked for retroactive relief only for himself and not for class members.

b) Court held that P’s failure to ask for retroactive relief for the class members precluded P from having adequately represented the class

c) Gonzales means that a court will examine whether a P will adequately represent a class twice:i) Before trial ii) After suit through collateral attacks based on res judicata

12) Individual notice is required for all Class Actions brought under Rule 23(b)(3). Eisen v. Carlisle & Jacquelin, 417 U.S. 156 (1974)a) However, Hansberry can be read to support the idea that Rule 23 class actions do not in fact

require notice and Eisen, after all, applies only to Rule 23(b)(3) actionsb) Richards v. Jefferson Cty., Ala., 517 U.S. 793 (1996) accepted the premise under Hansberry that

adequate representation could cure a lack of notice, but held that such an exception did not apply to the facts of the case in which:i) Those present were not in the same class as those absentii) Parties to the original action did not sue on behalf of their classiii) Parties did not purport to assert any claim against or on behalf of nonparties or purport to bind

any nonparties

Class Action Jurisdiction1) Filip:

a) Easiest case is Federal Question jurisdiction. If you have a class action with respect to a Federal question than the court clearly had jurisdiction over the actioni) E.g.,

(1) Antitrust(2) Securities fraud

ii) Supplemental jurisdiction could be asserted, though this does not usually occur in practice (e.g., Ps will not bring in state fraud claims in general in a securities fraud issue)

b) Diversity is more problematici) Do you look at the diversity of the citizenship of only the named plaintiff or all the plaintiffs?

(1) Ben-Hur holds that only citizenship of the named parties is required (a very pragmatic decision)

ii) Statutory minimum(1) Zahn v. Intl. Paper holds that each plaintiff must meet the statutory minimum(2) However, there is a split in the lower courts as to whether or not the § 1367 overruled

Zahn. 2) In a federal class action suit predicated on diversity jurisdiction under Rule 23(b)(3), each plaintiff

must have claim an injury in excess of the statutory threshold. Plaintiffs may not aggregate harms against a single defendant for the purposes of overcoming the statutory minimum, and all class members must meet the statutory threshold. Zahn v. Intl. Paper Co., 414 U.S. 291.

59

Ryan Preston Dahl; 917 687 7146

a) RPD: But see Stromberg Metal Works v. Press Mech., Inc., 77 F.3d 928 (7th Cir. 1996): § 1367 has modified the rule of Zahn to the extent that parties that fail to meet the diversity requirements of § 1332 (statutory minimum) may still be joined under Rule 20.

b) But compare with Rosario Ortega v. Star-Kist Foods, Inc., 370 F.3d 124 (1st Cir. 2004), specifically rejecting Stromberg and adopting majority rule to hold that a party joined through an exercise of § 1367 jurisdiction must still independently satisfy the statutory minimums to meet the requirement of § 1367(b)

3) A forum State may exercise jurisdiction over absent class-action plaintiffs even if P does not have International Shoe minimum contracts; subject to the requirement that the plaintiff has been afforded minimum due process protection (Mullane style). Phillips Petroleum Co. v. Shutts, 472 U.S. 797 (1985)a) Filip:

i) Two important issues:(1) Personal jurisdiction in the class action context(2) Choice of law issues and whether there are constitutional dimensions to choice of law

analysis that get magnified in the class action contextb) Class action was brought in KS court on behalf of 27,000 leaseholders for interest on royalties

due. 99% of the leases in question and 97% of the leaseholders had no connection to KS. State court applied KS law, finding for the class. Phillips appealed on due process and full faith and credit grounds.

c) Court held that KS could properly exercise jurisdiction if the Ps were afforded Mullane-style notice. The notice requires:i) Reasonably calculated to reach the Pii) Opportunity to be heard personally or through counseliii) Describe plaintiff’s rights and rights to e adjudicatediv) Ability to “opt out” or be excluded → this is very different from an “opt in” scenario

(1) If the default option is to opt-in, most people will likely opt in(2) You really need to make it an “opt out” option or, most likely, people will not join into

the lawsuitv) RPD: See Rule 23(c)(2) for the notice requirements for both 23(b)(1-2) and 23(b)(3) actions

d) Note: fn. 3 of the opinion implies that the holding is confined to Rule 23(b)(3)-type class actions. The Supreme Court has yet to resolve the confusion. i) Filip: Opt-out rights do not exist in (b)(1) and (b)(2) class actions (after all, it’s limited fund

cases or we’re looking to get a consistent equitable relief)ii) Fn. 3: “Our holding today is limited to those class actions which seek to bind known plaintiffs

concerning claims wholly or predominately for money judgments. We intimate no view concerning other types of class actions, such as those seeking equitable relief. Nor, of course, does our discussion of personal jurisdiction address class actions where the jurisdiction is asserted against a defendant class.”

e) However, court held that KS erred in using KS law to adjudicate the case in that it used the Due Process requirements used to establish jurisdiction to also validate its decision to use KS law (i.e., KS court failed to go through the Allstate analysis). i) KS and other possible state laws (e.g., OK, TX) were possibly in disagreementii) KS choice of law failed to determined whether the minimal requirements of “significant

contact or aggregation of contacts” required for a selection of law under Allstate Ins. Co. v. Hague All-State Ins. Co. v. Hague, 449 U.S. 302 (1981) (for State’s substantive law to be selected in a constitutionally permissible manner, that State must have a significant contact or significant aggregation of contracts creating state interests, such that choice of law is neither arbitrary nor unfair) were actually met(1) Note: On remand the KS court found that there was no conflict b/t KS law and that of any

other state, and applied KS law(2) Allstate analysis proceeds on a plaintiff-by-plaintiff basis, not on an aggregate basis

Preclusive Effect of a Class Action Judgment

60

Ryan Preston Dahl; 917 687 7146

1) The existence of a valid individual claim does not necessarily warrant that a P may maintain a class action, nor does the failure of a class action foreclose an individual claim by the representative P. Cooper v. Fed. Reserve Bank of Richmond, 467 U.S. 867 (1984)a) Filip : Res judicata and collateral estoppel apply in class actions. So you need to look even more

closely at what was actually litigated to see if classwide terms would/would not resolve individual claims that might brought in case two

b) Ps were the representatives in a class action lawsuit against he bank. However, their individual claims fell outside the class defined by the suit. The district court held that the class action suit failed but found that the Ps had individual claims for discrimination. Ps brought their individual claims and Bank claimed they were estopped.

c) Court held that the plaintiffs could bring their claims.i) “The rejection of a claim of classwide discrimination does not warrant the conclusion that no

member of the class could have a valid individual claim.”ii) Even as representative class members, the effect of prior judgment is only to

(1) Bar class members from bringing a similar class action lawsuit(2) Precluded from re-litigated questions of whether or not D engaged in the same prohibited

behavior over the relevant time periodiii) “The judgment is not, however, dispositive of the individual claims the Baxter petitioners

have alleged in their separate action.”2) Can a party who has voluntarily opted out of a class action as permitted (and indeed required) by

notice provisions of a 23(b)(3) class action, later assert collateral estoppel against a D after a verdict favorable to the P?a) Rule 23(c)(3) holds that judgment is binding upon:

i) All members of the class for Rule 23(b)(1) and Rule 23(b)(2) class actionsii) All members of Rule 23(b)(3) class actions who have failed to opt out

b) Allowing Ps who have opted out of the class action to use non-mutual estoppel defeat the purpose of Rule 23(c)(3). Wright and Miller § 1789i) “If the scope of issue preclusion is a matter of federal common law, then Parklane is not a

sufficient reason to upset the balance struck in Rule 23. Under the Rules Enabling Act, 28 U.S.C. § 2072, the Rules of Civil Procedure have the effect of statutes. A development in the common law of judgments is not a reason to undo a statute, to treat a thorough rethinking of the law as so much fluff. The revision of Rule 23 in 1966 does away with one-way intervention in class actions. It should stay done-away-with until the Supreme Court adopts a new version. Whether class members should get the benefit of a favorable judgment, despite not being bound by an unfavorable judgment, was considered and decided in 1966. That decision binds us still." Premier Elec. Constr. Co. v. National Elec. Contractors Ass'n, 814 F.2d 358, 364 (7th Cir. 1987) (Bauer, J.) (emphasis RPD)

c) However, some courts have allowed opt-outs to offensively use estoppel in this manner:i) E.g., Saunders v. Naval Air Rework Facility, Alameda, Cal., 608 F.2d 1308 (9th Cir. 1979);

Ps were permitted to estop D when they had opted out of an earlier class actionMass tort cases1) Certifications are reviewed on the basis of abuse of discretion. Castano.2) Filip

a) Products liability cases are extraordinarily dated; toxic torts are a bit dated as wellb) The mass tort issue remains very real → the big issue is tort claims which are economic losses

(e.g., fraud theory brought on a nationwide basis)3) Rule 23(b)(3)’s requirement that a common question of fact or law must predominate over individual

questions may be violated by a settlement that unfairly favors one group of the class. Amchem Prods., Inc. v. Windsor, 521 U.S. 591 (1997). Settlement is totally irrelevant to a Rule 23 certification analysisa) Prior asbestos litigation had attempted to resolve all claims of asbestos cases filed but not yet

litigated. The class action sub judice was filed by parties who had not filed actions prior to the earlier settlement. The action was filed on the same day with a proposed settlement. The settlement agreement included thati) Class would forego any and all subsequent or state claims, regardless of medical discoveriesii) There would be no inflation adjustment for payoutsiii) Latent harms would be paid as they developed

61

Ryan Preston Dahl; 917 687 7146

b) Filip: The Settlement issue changes the analysis somewhat:i) Reduces the Manageability concerns (no trial after all)ii) Heightens issues regarding:

(1) Predominance of claims(2) Similarity of the class members / potential conflicts(3) Superiority matters a great deal, but it’s not enough for the settlement to be “fair” b/c

people are not getting their day in courtc) The settlement was appealed on the basis of class certification

i) Settlement unfairly disadvantaged those with latent injuries(1) Filip: These are people w/out actual claims; they might not even know that they have a

problem(2) The lack subclasses is both important and, it seems, dispositive for the court

ii) The adequacy of representation required by 23(a)(4) was not met: the interests of the latent injury group were not fairly represented as opposed to those of the manifest injury group, nor should they have had common counsel

iii) Settlement process will be monitored by the counsel and the AFL-CIOd) In a Rule 23(b)(3) class action, a common question of law or fact must predominate over any

individual questions, and the class action must be superior to other methods of adjudication.e) Court held that this requirement could be and was violated by a settlement-only request for class

certification:i) “Federal courts . . . lack authority to substitute for Rule 23’s certification criteria . . . that if a

settlement is ‘fair’ then certification is proper.”ii) And the substantial disparity between the “exposure only” plaintiffs and the ones with actual

injury was sufficient, therefore, to violate 23(b)(3)iii) There was no adequate representation to satisfy Rule 23(a)(4)

f) O’Connor, J., concurring and dissentingi) Agrees settlement terms are required to class certification under Rule 23(b)(3)ii) However, analysis failed to consider

(1) The need for settlement of the massive litigation(2) Adequacy of representation should have properly been left to district court(3) The settlement was not unfair as a matter of law

4) Ortiz v. Fibreboard Corp., 527 U.S. 815 (1999) (NOTE: There is no opt out from 23(b)(1-2) class actions. Only 23(b)(3))a) D reached a settlement that it would pay out $1.3BN into a trust, and the settlement would be filed

as a class action under a Rule 23(b)(1)(B) limited fund settlementi) At the time, a settlement was ongoing in CA to determined if the insurers were liable for P’s

damages. The Settlement would include resolution of the suitii) Individual claimants were given a process by which to bring their claims

(1) They could seek a settlement with the trust(2) If no settlement was reached, a court action could be brought with a $500,000 limit on

recoveryiii) Class did not cover all individuals

(1) 45,000 people who had settled were not covered(2) 53,000 w/pending suits were also excluded

b) Court noted common suits brought under Rule 23(b)(1)i) Suits to reorganize fraternal benefit societiesii) Suits against a trustee (e.g., to declare a dividend or require an accounting procedure)

c) Requirements for a “limited fund” Rule 23(b)(1)(B) (limited fund) class action:i) Inadequacy of the fund to pay all maximum claimsii) The fund is devoted solely to the claimsiii) Ps are seeking recovery under a common theory

d) Court held that the extension of 23(b)(1)(B) claims into “unliquidated tort claims” was improperi) Not within the scope of the Ruleii) Serious Seventh Amendment problems for absent parties given the inability of 23(b)(1)

members to opt out

62

Ryan Preston Dahl; 917 687 7146

iii) The district court improperly extended the reach of 23(b)(1)(B)(1) No way to calculate total liability [i.e., the requested fund size] with certainty(2) No way to show the fund is insufficient(3) No way to accurately guarantee the size of the fund at all (as it was based on the sale

value of the company of certain company asset”iv) The “class” definition also

(1) Fails to include claimaints with foreseeable causes of action(2) Similarly situated Ps were receiving different payouts

(a) “It is obvious after Amchem that a class divided between holders of present and future claims . . . requires division into homogenous subclasses under Rule 23(c)(4)(B).”

v) Also, settlement would allow D to keep a substantial value that might otherwise be contributed to the common fund: “Fireboard was allowed to retain virtually its entire net worth.” → the assets are not “inadequate” as required by a 23(b)(1)(B) suit.

e) Breyer’s dissent focuses on the need for such practical settlement given the massive litigation involved

5) Costanoa) The largest class action ever attempted in Federal court

i) The class would be(1) all people who were nicotine dependent, (2) Their estates, (3) Spouses, children, survivors [etc.]

ii) The tort is fraud → claimed they weren’t told nicotine is addictive and the tobacco companies manipulated the level of nicotine (failure to inform what nicotine does, manipulating the level of the chemical)

iii) State law was being applied. b) The decision to certify is reviewed for abuse of discretion, and an abuse of discretion was found

i) District court failed to consider how the difference in state law would effect the prominence and superiority under the 23(b)(3) analysis(1) Also, court failed to show that there was some advantage in allowing the court to manage

the action on behalf of the class membersii) Court also failed to consider how a trial would actually work (process concerns)iii) A single class adjudication can’t, as a matter of law, be superior to individualized adjudication

in this case(1) Claim was based on “a new theory of liability and the existence of new evidence” hence

there is no certainty that a class action is actually more effective or required, or that the theory is in fact viable at all (hence, the “superiority” prong is not met)

(2) This same notion of “immaturity” was developed by Posner to reverse certification against pharma defendants in In re Rhone-Poulenc Rorer, Inc.(a) The plaintiffs had throught this was not a novel/immature tort – the court disagreed(b) Better to take a $1BN worth of claims a few small pieces at a time, rather than

risking a wrong judgment all together (i) Of course, the critique of this “immaturity” theory is that it runs counter to the

requirements of Rule 23 (ii) Filip: This is a major issue in class action litigation

c) “Predominance” must be analyzed in the context of whether if the issue is litigated on a classwide basis, does that resolution tend to resolve the claims of all the peoplei) i.e., a class action is not proper if the issue to be litigated, while common to the class, really

only resolves a minor element of the individual claimsii) Rather, “the district court . . . just assumed that because the common issues would play a part

in every trial, they must be significant.”iii) Importance of fraud based theories that have components of individualized reliance matters

greatly here:(1) The theory depends on being able to make millions of individualized reliance

assessements6) Filip: Important considerations in Predominance analysis:

63

Ryan Preston Dahl; 917 687 7146

a) State law variationb) Importance of the issue/fact to the overall claims (Costano)c) Similarity of class members (Windsor)

64